You are on page 1of 100

VISIONIAS™...

Inspiring Innovation
www.visionias.in
www.visionias.wordpress.com

“The significant problems we face cannot be solved at the same level of thinking we were at when we created them." - Albert Einstein
ANALYSIS / APPROACH / SOURCE / STRATEGY: GENERAL STUDIES
PRE 2022 PAPER
- TEAM VISION IAS
Observations on CSP 2022
• This year the paper appeared to be on moderately tougher side. Many questions though framed on a single topic demanded in-depth knowledge from
the students.
• The different trend followed by UPSC this year was in the framing of the options of pair type questions, which limited the students to solve questions
by elimination techniques.
• Also, this time like previous years, UPSC linked the fundamental subjects with Current Affairs and important issues which were in the news. This made
the paper more relevant and current affairs dominated.
• Static portions like Environment, Economics, S&T etc. were given due weightage. However, few portions like Modern History saw a dip and other static
portions like Geography and InternationalRelations saw a rise in the number of questions being asked as compared to the previous years.
• Questions in almost all the subjects ranged from easy to medium to difficult level.
• This year International Relations section questions gives us insight on how important it is to regularly read newspapers and follow Current Affairs
magazines. The questions were mainly directed towards basic understanding of conflicts & crisis in African and middle eastern countries. Students
were also tested on understanding of various UN mechanisms which frequently appearedin news.
• Polity questions were a mix of both fundamental topics and current affairs. This time conceptualquestions were not asked and mostly questions were
solvable by studying standard sources. Unlike previous years, Governance aspect was given more weightage eg Tea Board, Coal Controller's
Organization, etc.
• In the History section, question from Ancient and Medieval India were given more weightage and their difficulty level was also high. Art & Culture
questions were linked with current affairs directly e.g. Ramanujacharya and Somnath Temple. However, as a departure from the usual trend, UPSC
gave less weightage to Modern Indian History but questions though were easier as compared to last year.

1 www.visionias.in ©Vision IAS


• Geography this year was dominated by map-based questions, both national and international. Also, economic geography continues to remain an
essential topic owing to the government’s emphasis on infrastructure (e.g. Solar park question) and mineral resources development (e.g. question on
monazite). However, basic fundamental questions also were seen like summer solstice and clouds.
• This year questions related to Environment and Climate Change were a good mix of international initiatives (EP100, Polar Code, Climate Action
Tracker), species in news (Golden Mahseer, Ant-Fungi mutualism) and pollution related topics (Acid rain, WHO Air Quality Guidelines, Polythylene
Terephthalate). Terms frequently seen in the news (Greenwashing, Miyawaki) remain a favorite with the paper setters. Agriculture, an often
underrated subject, continues to maintain a stronghold (System of rice intensification, Crop source of methane, nitrogen-fixing plants). Some
unconventional questions testing the fundamental understandinghave also been covered (Wetlands as kidneys, Indian laws on wildlife protection).
• In Economics, continuing with the previous year trend, questions have been emphasized to test the fundamental understanding of candidates related
to concepts like Inflation, Bonds, MonetaryPolicy Instruments, NEER, etc. with a focus on their applicational aspect. Few questions from current affairs
themes like Indirect transfers and foreign e-commerce firms have also been explored. But unlike the trends followed in the previous years by UPSC,
Indian economy, agriculture and schemes questions were not seen!
• With regard to Science & Technology (including Basic Science) questions, they were by and large of moderate difficulty level. Among the easy-to-
answer questions were the ones on 'qubit', 'vaccines', and 'T cells'. In terms of coverage, questions featured from almost every expected section e.g.
space technology, computer science and information technology (as many as three questions), cosmology, biology, and biotechnology.
Nature of Question Source Type Difficulty Level
F: Fundamental, Conventional and conceptual question which is easily available
in commonly recommended books. If a current affairs source is mentioned then it EM = Essential material like basic books etc. E = Easy
indicates that you had one more reason to prepare this. RM = Reference material M = Medium
FA: Fundamental Applied question is an analytical question which requires EN = Essential News/Current Affairs D = Difficult
information + application of mind. Answer to these questions are from commonly
recommended books but not so obvious to find. RR = Random Read like random website
CA: Current Affair question which can be answered almost completely using etc.
given source alone. E : Easy , M : Medium , D : Difficult
CAA: Current Affair Applied question needs information from more than one
source to answer AND/OR application of mind (common sense and overall
understanding) of aspirant. This is why source for these questions may not be
answering the question completely.
FCA: Fundamental + Current affair. This is a F or FA question supplemented
with current affairs or a current affairs question that needs background
information for complete answer. Source to these questions might not answer the
questions in entirety.
U (Unconventional Question): Unconventional Question means the question that
is distantly related to syllabus provided by UPSC. It is neither present in even
reference sources (in addition to recommended books) nor prominently in news.
2 www.visionias.in ©Vision IAS
Vision IAS
A
L So (All India Test Series/ Current
n
Motivation e ur Affairs Magazine/PT
s Nat
QN Section Question Explanation / Current v Source ce 365/Study Material/Value
w ure
Affairs Link e Ty Addition Material/QRM-
e
l pe Quick Revision Module/Daily
r
News)
1 Current "Rapid Financing B Recent context: The Finance minister of Sri Lanka has Recently E FCA https://www.im E All India Test Series FLT: Test
Affairs Instrument" and started bailout talks with the lender of last resort (IMF) these f.org/en/About/ N 3497 (covered in explanation)
"Rapid Credit Facility" and seeking IMF’s rapid financial assistance to tackle provisions Factsheets/She Q. The facilities like 'Poverty
ets/2016/08/02
are related to the growing shortages of food, fuel and medicines. of lending Reduction and Growth
/19/55/Rapid-
provisions of lending The Rapid Financing Instrument (RFI) provides rapid were in Facility', 'Catastrophe
Financing-
by which one of the financial assistance, which is available to all member news, due Instrument#:~:t Containment and Relief Trust'
following? countries facing an urgent balance of payments need. to Sri ext=The%20Rap are often mentioned in the
(a) Asian Development The RFI was created as part of a broader reform to Lankan id%20Financing news in the context of
Bank make the IMF’s financial support more flexible to economic %20Instrument providing relief to low-income
(b) International address the diverse needs of member countries. The RFI crisis. %20(RFI,diverse countries to tackle the
Monetary Fund replaced the IMF’s previous emergency assistance %20needs%20o economic impact during the
(c) United Nations policy and can be used in a wide range of f%20member% pandemic. They are promoted
20countries.
Environment circumstances. by which of the following?
Programme Finance Financial assistance under the RFI is provided in the (a) G20
https://www.im
Initiative form of outright purchases without the need for a full- f.org/en/About/ (b) World Bank
(d) World Bank fledged program or reviews. A member country Factsheets/She (c) European Central Bank
requesting RFI assistance is required to cooperate with ets/2016/08/02 (d) International Monetary
the IMF to make efforts to solve its balance of payments /21/08/Rapid- Fund
difficulties and to describe the general economic Credit-Facility EXPLANATION
policies that it proposes to follow. Prior actions may be The Poverty Reduction and
required where warranted. https://econom Growth Facility (PRGF) is an
ictimes.indiatim
The Rapid Credit Facility (RCF) of IMF provides rapid arm of the International
es.com/news/in
concessional financial assistance to low-income Monetary Fund which lends
ternational/wor
countries (LICs) facing an urgent balance of payments ld- to the world's poorest
(BoP) need with no ex post conditionality where a full- news/technical- countries.
fledged economic program is neither necessary nor level- The PRGT has these three
feasible. The RCF was created under the Poverty discussions- concessional lending facilities:
Reduction and Growth Trust (PRGT) as part of a broader with-sri-lanka- Extended Credit Facility
reform to make the Fund’s financial support more will-continue- (ECF): Sustained medium- to
flexible and better tailored to the diverse needs of LICs, imf/articleshow long-term engagement in case
/91487174.cms
including in times of crisis. of protracted balance of
?from=mdr
Hence option (b) is the current answer. payments problems
Standby Credit Facility (SCF):
3 www.visionias.in ©Vision IAS
Financing for LICs with actual
or potential short-term
balance of payments and
adjustment needs caused by
domestic or external shocks,
or policy slippages—can also
be used on a precautionary
basis during times of
increased risk and uncertainty
Rapid Credit Facility (RCF):
Rapid financial support as a
single up-front payout for
low-income countries facing
urgent balance of payments
needs—possible repeated
disbursements over a
(limited) period in case of
recurring or ongoing balance
of payments needs.
2 Economics With reference to the C Nominal Effective Exchange Rate (NEER) is a measure This M FA https://datahel R All India Test Series: Test
Indian economy, of the value of a currency against a weighted average of question p.imf.org/knowl M 3476
consider the following several foreign currencies. The nominal exchange rate is seeks to edgebase/articl Consider the following
es/537469-
statements : the amount of domestic currency needed to purchase test the statements:
what-is-
1. An increase in foreign currency. If a domestic currency increases fundament 1. The Nominal effective
nominal-
Nominal Effective against a basket of other currencies inside a floating al effective- exchange rate (NEER) is a
Exchange Rate (NEER) exchange rate regime, NEER is said to appreciate. If the knowledge. exchange-rate- multilateral rate representing
indicates the domestic currency falls against the basket, the NEER neer the basket of foreign
appreciation of rupee. depreciates. An increase in NEER indicates an currencies, each weighted by
2. An increase in the appreciation of the local currency against the weighted https://datahel its importance to the
Real Effective basket of currencies of its trading partners. Hence p.imf.org/knowl domestic country in
Exchange Rate (REER) statement 1 is correct. edgebase/articl international trade.
es/537472-
indicates an 2. The Real Effective exchange
what-is-real-
improvement in trade Real Effective Exchange Rate (REER) is the real effective rate (REER) is interpreted as
effective-
competitiveness. exchange rate (a measure of the value of a currency exchange-rate- the quantity of domestic
3. An increasing trend against a weighted average of several foreign reer goods required to purchase
in domestic inflation currencies) divided by a price deflator or index of costs. one unit of a given basket of
relative to inflation in In simple words, a nation's nominal effective exchange https://www.in foreign goods.
other countries is likely rate (NEER), adjusted for inflation in the home country, vestopedia.com 3. The nominal exchange rate
to cause an increasing equals its real effective exchange rate (REER). An /terms/n/neer.a is the true measure of a
divergence between increase in REER implies that exports become more sp country’s international
4 www.visionias.in ©Vision IAS
NEER and REER. expensive and imports become cheaper; therefore, an https://www.fin competitiveness.
Which of the above increase indicates a loss in trade competitiveness. ancialexpress.co Which of the statements
statements are Hence statement 2 is not correct. m/opinion/getti given above is/are correct?
ng-real-about-
correct? (a) 1 and 2 only
reer-and-
(a) 1 and 2 only REER is the NEER after factoring in relative inflation (b) 2 and 3 only
exports/230203
(b) 2 and 3 only (consumer price-based index) using some measure of (c) 1 only
6/
(c) 1 and 3 only relative prices or costs; changes in the REER thus take into (d) 1, 2 and 3
(d) 1, 2 and 3 account both nominal exchange rate changes and the If the real exchange rises
inflation differential vis-à-vis trading partners. Soaring above one, this means that
inflation will impact REER, which, in turn, would inevitably goods abroad have become
push up the cost of merchandise and affect more expensive than goods at
competitiveness of Indian exports. Thus, if inflation is in an
home. The real exchange rate
increasing trend in domestic nation relative to inflation in
is often taken as a measure of
other countries, there is likely to cause an increasing
a country’s international
divergence between NEER and REER. For instance,
competitiveness.
considering NEER and REER values between April 2019 and
May 2021 in India, the NEER has been mostly declining,
whereas, REER remains in sync with the inflationary
trends—the upward biases in REER due to inflation was
already being felt in May 2021. The increasing difference
between trends of NEER and REER in the last 26 months
was due to India’s domestic inflation being higher relative
to the six major currencies considered. Hence statement 3
is correct.
3 Economics With reference to the B A central bank such as Reserve Bank of India (RBI), The M FA https://www.fin E All India Test Series: Test
Indian economy, periodically intervenes in the debt market to influence question ancialexpress.co N 3490
consider the following the interest rates and rate of inflation in the economy. If was m/money/your- With reference to the
money-central-
statements: RBI feels inflation is too high, it will sell government inspired Government Securities
banks-role-in-
1. If the inflation is too securities, and suck money out of the system. This act due to Acquisition Programme (G-
an-
high, Reserve Bank of will push up interest rates in the economy, and business fall/depreci interconnected- SAP), consider the following
India (RBI) is likely to will cut back on capital expenditure financed by loans, ation in world/2282240/ statements:
buy government reducing the demand for money. Hence statement 1 is rupee vis-a- 1. Under this program, RBI
securities. not correct. vis dollar https://www.liv purchases the Government
2. If the rupee is and rising emint.com/opin Securities from the market
rapidly depreciating, Central banks also intervene periodically in foreign inflation in ion/online- irrespective of the market
RBI is likely to sell exchange markets. If the rupee is rapidly depreciating, USA and in views/the- sentiment.
curious-case-of-
dollars in the market. RBI will sell dollars in the market. This will increase the India with 2. It may reduce borrowing
india-s-rising-
3. If interest rates in supply of dollars and the demand for rupees, causing recent cost of the government.
forex-reserves-
the USA or European the rupee price of the dollar to come down. On the interest and-falling- 3. It tends to depreciate the
Union were to fall, that contrary, if the rupee is rapidly appreciating, RBI will rates hike rupee- value of the Indian currency.
is likely to induce RBI buy dollars and inject rupees into the economy. This will in may 1163932878401 Which of the statements
5 www.visionias.in ©Vision IAS
to buy dollars. increase the demand for dollars and the supply of 2022 4.html given above is/are correct?
Which of the rupees, thereby leading to an increase in the rupee meeting of (a) 1 and 2 only
statements given price of the dollar. Hence statement 2 is correct. RBI https://www.m (b) 2 only
oneycontrol.co
above are correct ? monetary (c) 1 and 3 only
m/news/busine
(a) 1 and 2 only Interest rate movements in a foreign economy can policy (d) 1, 2 and 3
ss/rbi-may-
(b) 2 and 3 only stimulate action on the part of RBI. If interest rates in committee have-to-
(c) 1 and 3 only the US or the EU were to fall, FIIs (Foreign Institutional to curb tolerate-high-
(d) 1, 2 and 3 investors) will ramp up investments in India. The inflation. yields-prioritise-
resultant demand for rupees will cause the rupee to inflation-
appreciate. In response, RBI will buy dollars and inject experts-say-
rupees in to the system. Hence statement 3 is correct. 8502921.html
4 Current With reference to the C The Common Framework for debt treatment beyond As the M CA https://www.im E News Today: April 24th-25th
Affairs "G20 Common the DSSI (Common Framework) is an initiative endorsed pandemic f.org/en/About/ N 2022
Framework", consider by the G20, together with the Paris Club, to support, in battered FAQ/sovereign-
debt#s2q1
the following a structural manner, low income countries with global
statements: unsustainable debt. It is a way to temporarily ease the economies,
https://www.m
1. It is an initiative financing constraints for these countries and free up the Group ef.gov.it/en/G2
endorsed by the G20 scarce money that they can instead use to mitigate the of 20 0-
together with the Paris human and economic impact of the COVID-19 crisis. leading Italy/common-
Club. Hence both statements 1 and 2 are correct. economies framework.html
2. It is an initiative to launched
support Low Income measures, https://www.th
Countries with including a ehindu.com/ne
ws/internationa
unsustainable debt. temporary
l/chad-
Which of the debt
becomes-first-
statements given service country-to-ask-
above is/are correct ? suspension for-debt-
(a) 1 only for poor overhaul-under-
(b) 2 only countries. g20-common-
(c) Both 1 and 2 framework/artic
(d) Neither 1 nor 2 le33682498.ece
5 Economics With reference to the A Inflation-indexed bonds in India were issued by the IIBs were in M FA file:///C:/Users/ R News today: 16th December
Indian economy, what Reserve Bank of India (RBI) in 2013 and were news, as test/Downloads M 2021; PT 365 Economy: Page
are the advantages of benchmarked to Wholesale Price Index (WPI). RBI tries to /[IMF%20Worki 34
ng%20Papers]%
"Inflation-Indexed gauge VisionIAS AITS: FLT: 3502
20The%20Ratio
Bonds (IIBs)" ? Inflation-indexed bonds are financial instruments that demand for With reference to the Indian
nale%20and%2
1. Government can attempt to protect the bonds' purchasing power by the same 0Design%20of% economy, consider the
reduce the coupon tying interest and principal payments to an index of due to 20Inflation- following statements:
rates on its borrowing price changes. Indexed bonds include two types of increasing Indexed%20Bon 1. Floating Rate Bonds are
by way of IIBs. compensation, a real rate of return plus a compensation inflation. ds.pdf securities carrying a variable
6 www.visionias.in ©Vision IAS
2. IIBs provide for the erosion of purchasing power. Inflation coupon rate during the term
protection to the component on principal will not be paid with interest https://m.rbi.or of the security.
investors from but the same would be adjusted in the principal by g.in/Scripts/FAQ 2. Zero-Coupon Bonds are the
View.aspx?Id=9
uncertainty regarding multiplying principal with index ratio (IR). At the time of interest-bearing short-term
1
inflation. redemption, adjusted principal or the face, whichever is bonds issued by the Central
3. The interest higher, would be paid. Interest rate will be provided https://www.m Government.
received as well as protection against inflation by paying fixed coupon oneycontrol.co 3. Inflation-Indexed Bond (IIB)
capital gains on IIBs are rate on the principal adjusted against inflation. Hence m/news/trends is a bond issued by the
not taxable. statement 2 is correct. /features-2/- Sovereign (government),
Which of the Economists have argued that inflation indexed bonds 1699593.html which provides the investor
statements given could reduce government borrowing costs. If the with a constant return
above are correct ? market overestimates future inflation, government will irrespective of the level of
(a) 1 and 2 only reduce borrowing costs by issuing inflation indexed inflation in the economy.
(b) 2 and 3 only bonds rather than nominal bonds. This may occur
(c) 1 and 3 only because, for example, investors• expectations are not
(d) 1, 2 and 3 completely forward-looking or rational. Alternatively,
the government, because it is able to influence inflation
through its policies, may have better information about
the future course of inflation, or perhaps has more faith
in its commitment to contain it than the public does. In
these cases a treasury can lower its costs by issuing
indexed bonds. For example, if coupon rate of IIBs is
fixed 1.5 % above WPI (Whole sale price index) and
current WPI is 4%, so effective rate will be 5.5% (4+1.5).
In future, when WPI falls from 4% to 2%, then effective
coupon rate will become 3.5% (i.e. 2+1.5) and thus
government can reduce the coupon rates on its
borrowing by way of IIBs through reducing inflationary
trends. Hence statement 1 is correct.
Extant tax provisions will be applicable on interest
payment and capital gains on IIBs. There will be no
special tax treatment for these bonds. Hence statement
3 is not correct.
6 Economics With reference to B Consumer Protection (E-Commerce) Draft of D FCA https://consum R Monthly Current Affairs: June
foreign-owned e- Rules, 2020 define e-commerce entity (also foreign proposed eraffairs.nic.in/s M 2021(page 54)
commerce firms, owned) as any person who owns, operates or manages amendmen ites/default/file PT 365 Economy Page 92
s/E%20commer
operating in India, digital or electronic facility or platform for electronic ts to the
ce%20rules.pdf
which of the following commerce, but does not include a seller offering his Consumer
statements is/are goods or services for sale on a marketplace e- Protection https://dpiit.go
correct? commerce entity. Hence statement 1 is not correct. (E- v.in/sites/defaul
7 www.visionias.in ©Vision IAS
1. They can sell their Marketplace based model of e-commerce means commerce) t/files/pn3_201
own goods in addition providing an IT platform by an e-commerce entity on a Rules, 2020 6_0.pdf
to offering their digital & electronic network to act as a facilitator were in
https://pib.gov.i
platforms as market- between buyer and seller. On other hand, Inventory news.
n/Pressreleases
places. based model of e-commerce means an e-commerce
hare.aspx?PRID
2. The degree to which activity where inventory of goods and services is owned =1595850
they can own big by e-commerce entity and is sold to the consumers
sellers on their directly. However, FDI is not permitted in inventory
platforms is limited. based model of e-commerce.
Select the correct
answer using the code According to guidelines for FDI (Foreign Direct
given below: Investment) on E-Commerce, E-commerce entity
(a) 1 only providing a marketplace will not exercise ownership or
(b) 2 only control over the inventory i.e. goods purported to be
(c) Both 1 and 2 sold. Such an ownership or control over the inventory
(d) Neither 1 nor 2 will render the business into inventory based model.
Inventory of a vendor will be deemed to be controlled
by e-commerce marketplace entity if more than 25% of
purchases of such vendor are from the marketplace
entity or its group companies. Hence statement 2 is
correct.
7 Economics Which of the following A The real sector of an economy is the key section as This E F https://www.in R
activities constitute activities of this sector persuade economic output and is question vestopedia.com M
real sector in the represented by those economic segments that are seeks to /terms/f/financi
al_sector.asp
economy? essential for the progress of GDP of the economy. For test the
1. Farmers harvesting instance, farmers harvesting their crops or textile mills fundament
https://www.im
their crops converting raw cotton into fabrics ensure increase of al f.org/external/r
2. Textile mills economic output and in turn progress of GDP. The knowledge. egion/tlm/rr/pd
converting raw cotton sector is crucial for the sustainability of the economy f/Jan1.pdf
into fabrics because of its productive capability to meet nations’
3. A commercial bank aggregate demand. Hence statements 1 and 2 are file:///C:/Users/
lending money to a correct. test/Downloads
trading company /Relationship_B
etween_Financi
4. A corporate body On other hand, the financial sector is a section of the
al_and_Real_Se
issuing Rupee economy made up of firms and institutions that provide
ctors_Im.pdf
Denominated Bonds financial services to commercial and retail customers.
overseas The financial sector generates a good portion of its
Select the correct revenue from loans and mortgages. This sector
answer using the code comprises a broad range of industries including banks,
given below: investment companies, insurance companies, and real
8 www.visionias.in ©Vision IAS
(a) 1 and 2 only estate firms. Thus, a commercial bank lending money
(b) 2, 3 and 4 only to a trading company or a corporate body issuing
(c) 1, 3 and 4 only rupee denominated bonds overseas constitute
(d) 1, 2, 3 and 4 financial sector activities and not real sector activities.
Hence statements 3 and 4 are not correct.
8 Economics Which one of the D Indirect transfers refer to situations where when foreign Taxation M CAA https://prsindia. E
following situations entities own shares or assets in India, the shares of such Laws org/billtrack/th N
best reflects "Indirect foreign entities are transferred instead of a direct (Amendme e-taxation-laws-
amendment-
Transfers" often talked transfer of the underlying assets in India. Hence option nt) Act,
bill-2021
about in media (d) is the correct answer. 2021
recently with reference nullifies the https://econom
to India ? The origin of retrospective taxation can be traced retrospectiv ictimes.indiatim
(a) An Indian company backed to 2012, When Vodafone Ltd. was e taxation es.com/industry
investing in a foreign retrospectively taxed by the Indian tax authorities for a related to /indl-
enterprise and paying 2007 deal. The 2012 act had amended the IT act to indirect goods/svs/ceme
taxes to the foreign impose tax liability on the income earned from the sale transfers nt/adani-
country on the profits of shares of a foreign company on a retrospective basis that was holcim-deal-
and-the-legacy-
arising out of its (i.e., also applicable to the transactions done before introduced
vodafone-tax-
investment May 28, 2012). The amendments made by the 2012 Act with the
issue/articlesho
(b) A foreign company clarified that if a company is registered or incorporated Finance Act w/91619826.cm
investing in India and outside India, its shares will be deemed to be or have of 2012. s?from=mdr
paying taxes to the always been situated in India if they derive their value
country of its base on substantially from the assets located in India. As a https://www2.d
the profits arising out result, the persons who sold such shares of foreign eloitte.com/con
of its investment companies before the enactment of the Act (i.e., May tent/dam/Deloi
(c) An Indian company 28, 2012) also became liable to pay tax on the income tte/in/Documen
ts/tax/Global%2
purchases tangible earned from such sale.
0Business%20T
assets in a foreign
ax%20Alert/in-
country and sells such The Taxation Laws (Amendment) Act, 2021 nullifies the tax-gbt-alert-
assets after their value ‘retrospective taxation’ that was introduced with the indirect-
increases and transfers Finance Act of 2012. It nullifies this tax liability imposed transfer-
the proceeds to India on such persons provided they fulfil certain conditions. provisions-to-
(d) A foreign company These conditions are: apply-
transfers shares and prospectively-
such shares derive (i) if the person has filed an appeal or petition in this from-28-may-
2012-noexp.pdf
their substantial value regard, it must be withdrawn or the person must submit
from assets located in an undertaking to withdraw it,
https://www.ni
India (ii) if the person has initiated or given notice for any shithdesai.com/
arbitration, conciliation, or mediation proceedings in Content/docum
this regard, the notices or claims under such ent/pdf/Articles
9 www.visionias.in ©Vision IAS
proceedings must be withdrawn or the person must /171023_A_Indi
submit an undertaking to withdraw them, rect-Transfer-
(iii) the person must submit an undertaking to waive the Taxation-in-
India.pdf
right to seek or pursue any remedy or claim in this
regard, which may otherwise be available under any law
in force or any bilateral agreement, and
(iv) other conditions, as may be prescribed.

The Act ensures that there cannot exist any future


demand by the government for the collection of taxes
on the basis of an amendment with retrospective effect.
This Act makes the tax regime of India more predictable,
increasing the scope of foreign investment into the
country as it clarifies the stance of Indian Government
on imposition of retrospective taxation.

10 www.visionias.in ©Vision IAS


9 Economics With reference to the A Capital expenditures (CapEx) are funds used by a Capital & E F https://www.in R PT 365 Economy Page 20
expenditure made by company to acquire, upgrade, and maintain physical revenue vestopedia.com M VisionIAS Prelims Test 3490
an organisation or a assets such as property, plants, buildings, technology, or expenditur /terms/d/debtfi Consider the following:
nancing.asp#:~:
company, which of the equipment. CapEx is often used to undertake new e 1. Clothes bought by kids
text=Debt%20fi
following statements projects or investments by a company. Costs to upgrade fundament 2. Coffee machines in the
nancing%20occ
is/are correct? or purchase software, investing in new technology and al topic. urs%20when%2 cafeteria
1. Acquiring new computer equipment, are considered part of Capital 0a%20company 3. Tea leaves purchased by a
technology is capital expenditure. As they are often employed to improve %20raises%20m restaurant
expenditure. operational efficiency, increase revenue in the long oney%20by%20 Which of the above are
2. Debt financing is term, or make improvements to the existing assets of a selling,bonds%2 examples of Capital Goods?
considered capital company. Hence statement 1 is correct. C%20bills%2C% Test 3495 (AITS 2022):
expenditure, while 20or%20notes. Explanation
equity financing is When a company borrows money to be paid back at a It is a financial instrument
https://www.bu
considered revenue future date with interest it is known as debt financing. It which is a hybrid of debt &
siness-
expenditure. occurs when a firm sells fixed income products, such as standard.com/a equity financing that gives the
Select the correct bonds, bills, or notes. It could be in the form of a bout/what-is- lender the right to convert to
answer using the code secured as well as an unsecured loan. Debt financing is capital- an ownership or equity
given below: the opposite of equity financing, which entails issuing receipts#:~:text interest in the company in
(a) 1 only stock to raise money. For example, reliance can choose =Capital%20rec case of default, generally,
(b) 2 only debt financing, which entails selling fixed income eipts%20are%2 after venture capital
(c) Both 1 and 2 products, such as bonds, bills, or notes, to investors to 0receipts%20th companies and other senior
at,crucial%20pa
(d) Neither 1 nor 2 obtain the capital needed to grow and expand its lenders are paid.
rt%20of%20capi
operations. Both debt financing and equity financing are Which among the following
tal%20receipts.
considered as part of capital receipts for the company, financial instruments is best
as capital receipts are receipts that create liabilities or https://www.in described by the above
reduce financial assets. Funds from these would be used vestopedia.com passage?
by company for capital expenditure such as to grow or /ask/answers/1 (a) Mezzanine Financing
expand its operations. Hence statement 2 is not 12814/what- (b) Invoice Financing
correct. are-some- (c) Senior Debt
examples-main- (d) Debt Financing
types-capital-
expenditures-
capex.asp
10 Economics With reference to the C Household financial savings refer to currency, bank Internal M FA https://dea.gov. R
Indian economy, deposits, debt securities, mutual funds, pension funds, debt & in/sites/default/ M
consider the following insurance, and investments in small savings schemes by Household files/Final%20St
households. The net household financial savings was 11.5 atus%20paper%
statements : financial
percent of GNDI (gross national disposal income) in 2020- 202020-21.pdf
1. A share of the savings are
household financial 21. A part of this financial saving goes toward government important
borrowing. As government borrows through the issue of Page 8
savings goes towards topics and
government securities called G-secs and Treasury Bills. It
government often are in
11 www.visionias.in ©Vision IAS
borrowings. borrows from the market, small savings funds, state news. https://www.fin
2. Dated securities provident funds, external assistance and short-term ancialexpress.co
issued at market- borrowings. Any adverse movement in the household m/opinion/how
savings will have a significant bearing on banks, insurance -critical-are-
related rates in
companies and mutual/provident funds, who, in turn, are household-
auctions form a large
key investors in government securities. Hence statement 1 savings-for-
component of internal government-
debt. is correct.
borrowing/1542
The Central Government Debt includes all liabilities of
Which of the above 170/
Central Government contracted against the Consolidated
statements is/are
Fund of India (defined as Public Debt). Public debt is
correct ? https://rbidocs.
further classified into internal and external debt. Internal rbi.org.in/rdocs
(a) 1 only
debt /AnnualReport/
(b) 2 only consists of marketable debt and non-marketable debt. PDFs/0RBIAR20
(c) Both l and 2 Marketable debt comprises of Government dated 21226AD1119FF
(d) Neither 1 nor 2 securities and Treasury Bills, issued through auctions. Non- 6674A13865C9
marketable debt comprises of intermediate 88DF70B4E1A.P
Treasury Bills (14 days ITBs) issued to State DF
Governments/UTs as well as select Central Banks, special
securities issued against small savings, special securities
issued to public sector banks/EXIM Bank, securities issued
to international financial institutions, and compensation
and other bonds.
All marketable securities i.e. Dated securities and Treasury
bills are issued through auctions as per the schedule
notified through halfyearly/quarterly auction calendars. As
at end-March 2021, outstanding amounts under dated
securities stood at 71.7 lakh crore (36.3 per cent of GDP)
and accounted for 68.1 per cent of the total Public Debt.
Hence statement 2 is correct.

12 www.visionias.in ©Vision IAS


11 Polity & Consider the following B Statement 1 is correct: A committee was set up in 1961 Recently, E FCA Laxmikanth E PT 365 Polity Page 27
Governanc statements : under the chairmanship of the late H N Sanyal, the then Supreme Indian Polity M All India Test Series FLT: Test
e 1. Pursuant to the additional solicitor general. The committee made a Court held https://www.le 3500 (statement 2)
galserviceindia.
report of H.N. Sanyal comprehensive examination of the law and problems that With reference to contempt
com/article/l25
Committee, the relating to contempt of court in the light of the position No law can of court, consider the
5-Contempt-of-
Contempt of Courts obtaining in our own country and various foreign take away Court.html following statements:
Act, 1971 was passed. countries. The recommendations, which the committee court’s 1. The consent of the
2. The Constitution of made, took note of the importance given to freedom of power to https://indianex Attorney General of India is
India empowers the speech in the Constitution and of the need for punish for press.com/articl mandatory when a private
Supreme Court and the safeguarding the status and dignity of courts and contempt. e/india/sc-even- citizen wants to initiate a case
High Courts to punish interests of administration of justice. law-cant-take- of contempt of court against a
for contempt of away-power-to- person.
punish-for-
themselves. Statement 2 is correct: Article 129 of the Constitution 2. Constitution empowers the
contempt-
3. The Constitution of gives the Supreme Court the power to initiate contempt Supreme Court to initiate
7542797/
India defines Civil cases on its own, independent of the motion brought contempt cases on its own.
Contempt and Criminal before it by the Attorney General (AG) or with the 3. The Attorney General
Contempt. consent of the AG. brings in a motion before the
4. In India, the Supreme Court and High
Parliament is vested Statement 3 is not correct and Statement 4 is correct: Courts for initiating a case of
with the powers to Contempt of court seeks to protect judicial institutions criminal contempt.
make laws on from motivated attacks and unwarranted criticism, and Which of the statements
Contempt of Court. as a legal mechanism to punish those who lower its given above is/are correct?
Which of the authority. The expression ‘contempt of court’ has not Explanation: Article 129 of
statements given been defined by the Constitution. The Contempt of the Constitution gives the
above is/are correct ? Court Act, 1971 defines both civil and criminal Supreme Court the power to
(a) 1 and 2 only contempt. Hence the Parliament is vested with the initiate contempt cases on its
(b) 1, 2 and 4 powers to make laws regarding contempt of court. own, independent of the
(c) 3 and 4 only motion brought before it by
(d) 3 only the AG or with the consent of
the AG.

Explanation Test 3495:


Statement 3 and Statement 4
The expression ‘contempt of
court’ has not been defined
by the Constitution. The
Contempt of Court Act, 1971
defines both civil and criminal
contempt.

13 www.visionias.in ©Vision IAS


Civil contempt refers to the
wilful disobedience of an
order of any court.
Criminal contempt includes
any act or publication which:
‘scandalises’ the court, or
prejudices any judicial
proceeding, or
interferes with the
administration of justice in
any other manner.
‘Scandalising the Court’
broadly refers to statements
or publications which have
the effect of undermining
public confidence in the
judiciary.
The superior courts (Supreme
Court and High Courts) derive
their contempt powers from
the Constitution. The Act only
outlines the procedure in
relation to investigation and
punishment for contempt.
Therefore, deletion of the
offence from the Act will not
impact the inherent
constitutional powers of the
superior courts to punish
anyone for its contempt.
These powers will continue to
remain, independent of the
1971 Act.
12 Polity & With reference to B The role of the legal profession in society is manifold— After D FCA https://www.ep E All India Test Series Test
Governanc India, consider the its members are flag-bearers of the rule of law and they widespread w.in/journal/20 N 3480: Statement 2
e following statements : defend fundamental rights. Along with these opposition 18/2/commenta With reference to the Bar
ry/reformation-
1. Government law responsibilities, members of the legal profession have from Council of India, consider the
legal-
officers and legal firms been conferred significant power and privileges as lawyers to following statements:
profession-
are recognised as officers of the court. First, only advocates have the the new interest- 1. It is established under the
advocates, but right to access and represent others in a court of law. rules of justice.html Arbitration and Conciliation
14 www.visionias.in ©Vision IAS
corporate lawyers and No citizen, other than an advocate, has the right to conduct Act, 1996.
patent attorneys are appear, act, or plead in court. Second, in India, the term notified by https://www.in 2. The Solicitor General of
excluded from “legal professionals” refers only to those “advocates” the Bar diatoday.in/law India is the ex officio member
/story/bci-
recognition as who are law graduates and have been enrolled in state Council of of the Bar Council of India.
forms-panel-to-
advocates. bar councils (SBCs). India (BCI), 3. It is responsible for setting
review-new-
2. Bar Councils have Statement 1 is not correct: An emerging class of legal the Council rules-that- standards of legal education
the power to lay down professionals engaged with various law-related has now set prevent- in India.
the rules relating to activities—such as government law officers, corporate up a review lawyers-from- Which of the statements
legal education and lawyers, law firms, law professors, legal researchers, committee criticising- given above is/are correct?
recognition of law and patent attorneys—have been excluded from and judges-bar-
colleges. recognition as advocates. announced councils-
Which of the Third, the judiciary has clarified the otherwise that these 1821331-2021-
06-30
statements given undefined concept of “practice of law” to include all rules will
above is/are correct ? forms of legal activities, including both litigious and not be
https://www.in
(a) 1 only non-litigious work, such as appearing in court, drafting, implement diatoday.in/law
(b) 2 only giving opinions, performing transactional work, ed for now. /story/bar-
(c) Both 1 and 2 consulting, arbitrating, mediating, filing vakalatnamas The new council-india-
(d) Neither 1 nor 2 (power of attorney), and working as legal officers. Thus, rules amends-rules-
“advocates” enrolled in bar councils enjoy exclusive triggered a curb-criticism-
monopoly over the right to practise law in all courts, controvers against-judges-
tribunals, and other authorities in India. y as they 1819920-2021-
06-27
Statement 2 is correct: The Advocates Act, 1961, seek to
enacted with the objective of creating “a unified Bar for prevent
the whole country with monopoly in legal practice and lawyers
autonomy in matters of professional management,” from
conferred the Bar Council of India (BCI) and SBCs with publicly
the power to self-regulate the profession and lay down criticising
rules relating to admission and enrolment, conditions judges and
of practice, standards of professional conduct and bar
etiquette, disciplinary proceedings, legal education, councils.
recognition of law colleges, and welfare activities.
These powers were granted to the bar councils to
promote the administration of justice and uphold the
dignity of the profession in the eyes of the common
people.
13 Polity & Consider the following B An amendment of the Constitution can be initiated only The new E F Laxmikanth E All India Test Series Test 3471
Governanc statements : by the introduction of a bill for the purpose in either rules Indian Polity M Consider the following
e 1. A bill amending the House of Parliament and not in the state legislatures. triggered a statements in the context of
Constitution requires a Statement 1 is not correct: The bill can be introduced controversy the Constitutional
prior recommendation either by a minister or by a private member and does as they amendment bill:
15 www.visionias.in ©Vision IAS
of the President of not require prior permission of the president. seek to 1. It can be introduced in
India. The bill must be passed in each House by a special prevent either House of Parliament.
2. When a Constitution majority, that is, a majority (that is, more than 50 per lawyers 2. It can be introduced by a
Amendment Bill is cent) of the total membership of the House and a from minister only.
presented to the majority of two-thirds of the members of the House publicly 3. If it is duly passed by both
President of India, it is present and voting. criticising the Houses of Parliament, the
obligatory for the Statement 3 is correct: Each House must pass the bill judges and President must give his assent
President of India to separately. In case of a disagreement between the two bar to the bill.
give his/her assent. Houses, there is no provision for holding a joint sitting councils. Which of the statements
3. A Constitution of the two Houses for the purpose of deliberation and given above are correct?
Amendment Bill must passage of the bill. Explanation: The procedure
be passed by both the If the bill seeks to amend the federal provisions of the for the amendment of the
Lok Sabha and the Constitution, it must also be ratified by the legislatures Constitution as laid down in
Rajya Sabha by a of half of the states by a simple majority, that is, a Article 368 is as follows:
special majority and majority of the members of the House present and An amendment of the
there is no provision voting. Constitution can be initiated
for joint sitting. After duly passed by both the Houses of Parliament and only by the introduction of a
Which of the ratified by the state legislatures, where necessary, the bill for the purpose in either
statements given bill is presented to the president for assent. House of Parliament and not
above are correct ? Statement 2 is correct: The president must give his in the state legislatures.
(a) 1 and 2 only assent to the bill (24th Constitutional Amendment Act, The bill can be introduced
(b) 2 and 3 only 1967). He can neither withhold his assent to the bill nor either by a minister or by a
(c) 1 and 3 only return the bill for reconsideration of the Parliament. private member and does not
(d) 1, 2 and 3 After the president’s assent, the bill becomes an Act require prior permission of
(i.e., a constitutional amendment act) and the the president.
Constitution stands amended in accordance with the The bill must be passed in
terms of the Act." each House by a special
majority, that is, a majority
(that is, more than 50 per
cent) of the total
membership of the House
and a majority of two-thirds
of the members of the House
present and voting.
Each House must pass the bill
separately. In case of a
disagreement between the
two Houses, there is no
provision for holding a joint
sitting of the two Houses for
16 www.visionias.in ©Vision IAS
the purpose of deliberation
and passage of the bill.
If the bill seeks to amend the
federal provisions of the
Constitution, it must also be
ratified by the legislatures of
half of the states by a simple
majority, that is, a majority of
the members of the House
present and voting.
After duly passed by both the
Houses of Parliament and
ratified by the state
legislatures, where necessary,
the bill is presented to the
president for assent.
The president must give his
assent to the bill. He can
neither withhold his assent to
the bill nor return the bill for
reconsideration of the
Parliament.
After the president’s assent,
the bill becomes an Act (i.e., a
constitutional amendment
act) and the Constitution
stands amended in
accordance with the terms of
the Act."
14 Polity & Consider the following B Statement 1 is not correct: The Council of Ministers is a Recently, E F Laxmikanth E All India Test Series FLT Test
Governanc statements : constitutional body. Article 74 of the Constitution the Union Indian Polity M 3504
e 1. The Constitution of provides that there shall be a Council of Ministers with Governmen Consider the following
India classifies the the Prime Minister at the head to aid and advise the t expanded statements:
ministers into four President who shall, in the exercise of his functions, act its Council 1. The central council of
ranks viz. Cabinet in accordance with such advice. of Ministers ministers is a constitutional
Minister, Minister of However, its classification is not mentioned in the to 77 by body.
State with Constitution. The classification of the central council of inducting 2. The size and classification
Independent Charge, ministers into a three-tier body (Cabinet Ministers, new of the central council of
Minister of State and Ministers of State, and Deputy Ministers) is based on ministers ministers are mentioned in
Deputy Minister. British parliamentary conventions. and the Indian Constitution.
17 www.visionias.in ©Vision IAS
2. The total number of Statement 2 is correct: The 91st Amendment, 2003 to promoting Which of the following is
ministers in the Union the Constitution introduced Article 75(1A), which states junior correct with reference to the
Government, including that the Union Council of Ministers cannot exceed more ministers. above statements?
the Prime Minister, than 15 percent of the number of MPs in the Lok Sabha. (a) Both Statement 1 and
shall not exceed 15 Statement 2 are correct and
percent of the total Statement 2 is the correct
number of members in explanation for Statement 1.
the Lok Sabha. (b) Both Statement 1 and
Which of the Statement 2 are correct but
statements given Statement 2 is not the correct
above is/are correct ? explanation for Statement 1.
(a) 1 only (c) Statement 1 is correct but
(b) 2 only Statement 2 is not correct.
(c) Both 1 and 2 (d) Statement 1 is not correct
(d) Neither 1 nor 2 but Statement 2 is correct.
Explanation: The 91st
Amendment to the
Constitution introduced
Article 75(1A), which states
that the Union Council of
Ministers cannot exceed
more than 15 percent of the
number of MPs in the Lok
Sabha.

The Council of Ministers is a


constitutional body. Article 74
of the Constitution provides
that there shall be a Council
of Ministers with the Prime
Minister at the head to aid
and advise the President who
shall, in the exercise of his
functions, act in accordance
with such advice.
However, its classification is
not mentioned in the
Constitution. But the
maximum size of council of
ministers (15% of the number
18 www.visionias.in ©Vision IAS
of MPs in Lok Sabha) was
mentioned.
The classification of the
central council of ministers
into a three-tier body (Cabinet
Ministers, Ministers of State,
and Deputy Ministers) is
based on British
parliamentary conventions.
15 Polity & Which of the following B Option 1 is not correct: Under Article 352, the President This E F Laxmikanth E All India Test Series: Test
Governanc is/are the exclusive can declare a national emergency when the security of question Indian Polity M 3493; Test 3470: Option 1
e power(s) of Lok Sabha? India or a part of it is threatened by war or external seeks to Consider the following
1. To ratify the aggression or armed rebellion. test the statements regarding National
declaration of The President can declare a national emergency even fundament Emergency:
Emergency before the actual occurrence of war or armed rebellion al 1. Approval of parliament is
2. To pass a motion of or external aggression Parliamentary approval and knowledge. necessary for the revocation
no-confidence against duration. of the National Emergency.
the Council of 2. Maximum time period for
Ministers The proclamation of emergency must be approved by the operation of the National
3. To impeach the both the houses of parliament within one month from Emergency is three years.
President of India the date of its issue.If approved by both the houses, the Which of the statements
Select the correct Emergency continues for 6 months and can be extended given above is/are correct?
answer using the code to an indefinite period with an approval of the
given below: Parliament for every six months.
(a) 1 and 2
(b) 2 only Every resolution approving the proclamation of Test 3703 Open Test/
(c) 1 and 3 emergency or its continuance must be passed by either Abhyaas(2022):
(d) 3 only House of Parliament by a special majority. Consider the following
statements:
Option 2 is correct: Article 75 of the Constitution says 1. Prior to 1962 there were
that the council of ministers shall be collectively Parliamentary Constituencies
responsible to the Lok Sabha. It means that the ministry where two members were
stays in office so long as it enjoys the confidence of the elected from the same
majority of the members of the Lok Sabha. In other constituency.
words, the Lok Sabha can remove the ministry from 2. More than a hundred no-
office by passing a no-confidence motion. The motion confidence motions have
needs the support of 50 members to be admitted. been passed in Lok Sabha
since independence.
Option 3 is not correct: Article 61 of the Indian 3. Since independence the
Constitution states that when a President is to be voter turnout percentage in
19 www.visionias.in ©Vision IAS
impeached for violation of the Constitution, the charge general Lok Sabha elections
shall be preferred by either House of Parliament. has been increasing
Either Lok Sabha or Rajya Sabha can initiate the continuously.
process for the removal of the President of India. Which of the statements
given above is/are correct?
Test 3499: Option 3
With reference to the
Impeachment procedure of
the President of India and the
USA, consider the following
statements:
1. While the ground for the
impeachment of the Indian
President is the Violation of
the Constitution, for the
President of the USA there
are no specific grounds.
2. Only the Lower House of
the Legislature can initiate the
process for the impeachment
of the President of India as
well as the USA.
Which of the statements
given above is/are correct?
16 Polity & With reference to anti- B To tackle the scourge of political defection, in 1969, a Anti- E F Laxmikanth E PT 365 Polity Page 36.
Governanc defection law in India, committee chaired by Home Minister Y B Chavan defection Indian Polity M Vision IAS Test Series: Test
e consider the following examined the issue of defection. It observed that after laws are 3493
statements: the 1967 general elections, defections changed the frequently With reference to the Anti-
1. The law specifies political scene in India as 176 of 376 independent in the Defection Law in India
that a nominated legislators later joined a political party. news. consider the following
legislator cannot join The next legislative attempt, in 1978, allowed Recently, statements:
any political party independent and nominated legislators to join a the 1. Anti-Defection Law was
within six months of political party once. Calcutta enacted on the
being appointed to the With rising public opinion for an anti-defection law, High Court recommendation of the Y.B
House. immediately after securing a clear majority in 1984, the has given Chavan committee.
2. The law does not Rajiv Gandhi's government proposed the new anti- West 2. A legislator’s speech and
provide any defection bill in the Parliament. After marathon Bengal conduct inside the legislature
time-frame within debates, both the Lok Sabha and Rajya Sabha Assembly cannot be used to disqualify
which the presiding unanimously approved the bill and through the 52nd Speaker a him under the Anti-Defection
officer has to decide a amendment to the Constitution of India, the Tenth deadline to Law.
20 www.visionias.in ©Vision IAS
defection case. Schedule was inserted into the constitution. pass an 3. Under this law, the
Which of the The Tenth Schedule contains the following provisions order in the presiding officer of legislature
statements given with respect to the disqualification of members of defection has to decide on a
above is/are correct ? Parliament and the state legislatures on the ground of case. disqualification case brought
(a) 1 only defection: before him within six months.
(b) 2 only Which of the statements
(c) Both 1 and 2 Members of Political Parties: A member of a House given above is/are not
(d) Neither 1 nor 2 belonging to any political party becomes disqualified for correct?
being a member of the House,
-if he voluntarily gives up his membership of such
political party or
-if he votes or abstains from voting in such House
contrary to any direction issued by his political party
without obtaining prior permission of such party and
such act has not been condoned by the party within 15
days.
-In the case of ‘voluntarily giving up his membership,'
the Supreme Court has interpreted that in the absence
of a formal resignation by the member, the giving up of
membership can be inferred by his conduct. In other
judgments, members who have publicly expressed
opposition to their party or support for another party
were deemed to have resigned.

Thus, a legislator’s speech and conduct inside and


outside the legislature can lead to disqualification under
the anti-defection law.
Independent Members: An independent member of a
House (elected without being set up as a candidate by
any political party) becomes disqualified to remain a
member of the House if he joins any political party after
such election.
Statement 1 is not correct: Nominated Members: A
nominated member of a House becomes disqualified for
being a member of the House if he joins any political
party after the expiry of six months from the date on
which he takes his seat in the House. This means that he
may join any political party within six months of taking
his seat in the House without inviting this
disqualification.
21 www.visionias.in ©Vision IAS
Exceptions:
The above disqualification on the ground of defection
does not apply in the following two cases:
If a member goes out of his party as a result of a merger
of the party with another party. A merger takes place
when two-thirds of the members of the party have
agreed to such a merger.
If a member, after being elected as the presiding officer
of the House, voluntarily gives up the membership of his
party or rejoins it after he ceases to hold that office.
This exemption has been provided in view of the dignity
and impartiality of this office.
Statement 2 is correct: The Anti-Defection Law does not
specify a time period for the Presiding Officer to decide
on a disqualification plea. Given that courts can
intervene only after the Presiding Officer has decided on
the matter, the petitioner seeking disqualification has
no option but to wait for this decision to be made.

22 www.visionias.in ©Vision IAS


17 Polity & Consider the following D Statement 1 is not correct: Article 76 of the constitution This M F Laxmikanth E All India Test Series: Test
Governanc statements : provides for the office of Attorney General of India and question Indian Polity M 3479
e 1. Attorney General of he is the highest law officer in the country. The Solicitor seeks to Consider the following
India and Solicitor General of India assists Attorney General to fulfil his test the statements:
General of India are official duties and responsibilities. fundament 1. A minister who is not a
the only officers of the In the performance of his official duties, the Attorney al member of either House,
Government who are General (AG) has the right of audience in all courts in knowledge. cannot participate in a joint
allowed to participate the territory of India. Further, he has the right to speak sitting of both the Houses.
in the meetings of the and to take part in the proceedings of both the Houses 2. A minister belonging to the
Parliament of India. of Parliament or their joint sitting and any committee Lok Sabha can participate in
2. According to the of the Parliament of which he may be named a the proceedings of Rajya
Constitution of India, member, but without a right to vote. He enjoys all the Sabha without being entitled
the Attorney General privileges and immunities that are available to a to vote.
of India submits his member of Parliament. The Solicitor General of India is 3. The attorney general of
resignation when the not allowed to participate in the meetings of the India can speak and take part
Government which Parliament. in the proceedings of either
appointed him resigns. house but not in a joint sitting
Which of the Statement 2 is not correct: The term of office of the AG of both the Houses.
statements given is not fixed by the Constitution. Further, the Which of the statements
above is/are correct ? Constitution does not contain the procedure and given above is/are correct?
(a) 1 only grounds for his removal. He holds office during the Test 3470: Consider the
(b) 2 only pleasure of the president. This means that he may be following statements
(c) Both 1 and 2 removed by the president at any time. He may also quit regarding the Attorney
(d) Neither 1 nor 2 his office by submitting his resignation to the president. General of India:
Conventionally, he resigns when the government 1. Article 76 of the Indian
(council of ministers) resigns or is replaced, as he is constitution provides for the
appointed on its advice. office of the Attorney General
of India.
2. He is subordinate to and
assists the Solicitor General of
India.
Which of the statements
given above is/are correct?
18 Polity & With reference to the C Mandamus literally means ‘we command’. It is a This E F Laxmikanth E All India Test Series: Test
Governanc writs, issued by the command issued by the court to a public official asking question Indian Polity M 3470:
e Courts in India, him to perform the official duties that he has failed or seeks to Consider the following
consider the following refused to perform. test the statements:
statements : Statement 2 is not correct: It can also be issued against fundament 1. Only an aggrieved person
1. Mandamus will not any public body, a corporation, an inferior court, a al can seek a Quo-Warranto
lie against a private tribunal, or government for the same purpose. knowledge. writ.
23 www.visionias.in ©Vision IAS
organisation unless it is Statement 1 is correct: The writ of mandamus cannot 2. Mandamus cannot be
entrusted with a public be issued: issued against a private
duty. - against a private individual or body. individual.
2. Mandamus will not - to enforce departmental instruction that does not 3. Certiorari can be issued
lie against a Company possess statutory force. even against administrative
even though it may be - when the duty is discretionary and not mandatory to authorities.
a Government enforce a contractual obligation. Which of the statements
Company. - against the president of India or the state governors. given above is/are correct?
3. Any public minded - against the chief justice of a high court acting in a
person can be a judicial capacity.
petitioner to move the Statement 3 is correct: Quo Warranto means ‘by what
Court to obtain the authority or warrant’. It is issued by the court to enquire
writ of Quo Warranto. into the legality of the claim of a person to a public
Which of the office. Hence, it prevents illegal usurpation of public
statements given office by a person.
above are correct ? The writ can be issued only in case of a substantive
(a) 1 and 2 only public office of a permanent character created by a
(b) 2 and 3 only statute or by the Constitution. It cannot be issued in
(c) 1 and 3 only cases of ministerial office or private office. Any
(d) 1, 2 and 3 interested person and not necessarily the aggrieved
person can seek Quo Warranto writ.
19 Current With reference to B The Union Cabinet, chaired by Prime Minister Shri The scheme M CA https://ndhm.g E VisionIAS Monthly Current
Affairs Ayushman Bharat Narendra Modi has approved the national roll-out of was ov.in/faq M Affairs Magazine November
Digital Mission, Central Sector Scheme, Ayushman Bharat Digital recently 2021: Page 89
consider the following Mission (ABDM) of Ministry of Health and Family launched. Vision IAS Feb MCAR
statements : Welfare, Government of India, with a budget of Question:
1. Private and public Rs.1,600 crore for five years. The National Health Consider the following
hospitals must adopt Authority (NHA) will be the implementing agency of statements about Ayushman
it. Ayushman Bharat Digital Mission (ABDM). Bharat Digital Mission
2. As it aims to achieve (ABDM):
universal health Statement 1 is not correct: Participation in ABDM is 1. It aims to provide digital
coverage, every citizen voluntary including for citizens. Participation of a health IDs for all Indian
of India should be part healthcare facility or an institution is also voluntary citizens.
of it ultimately. and shall be taken by the respective management 2. It creates interoperability
3. It has seamless (government or private management). However, once within the digital health
portability across the the management decides to register the respective ecosystem.
country. healthcare facility/institution in ABDM, it is essential for 3. National Health Authority
Which of the all the healthcare professionals serving the said (NHA) is implementing agency
statements given facility/institution to register in Healthcare Professionals for it.
above is/are correct ? Registry so that the institution can become fully Which of the statements
24 www.visionias.in ©Vision IAS
(a) 1 and 2 only integrated with the National Digital Health Ecosystem given above is/are correct?
(b) 3 only (NDHE).
(c) 1 and 3 only Statement 2 is not correct and statement 3 is correct:
(d) 1, 2 and 3 Ayushman Bharat Digital Mission will connect the
digital health solutions of hospitals across the country
with each other. Based on the foundations laid down in
the form of Jan Dhan, Aadhaar and Mobile (JAM) trinity
and other digital initiatives of the government,
Ayushman Bharat Digital Mission (ABDM) is creating a
seamless online platform through the provision of a
wide-range of data, information and infrastructure
services, duly leveraging open, interoperable,
standards-based digital systems while ensuring the
security, confidentiality and privacy of health-related
personal information.

Under the ABDM, citizens will be able to create their


ABHA (Ayushman Bharat Health Account) numbers, to
which their digital health records can be linked. This
will enable creation of longitudinal health records for
individuals across various healthcare providers, and
improve clinical decision making by healthcare
providers. The mission will improve equitable access to
quality healthcare by encouraging use of technologies
such as telemedicine and enabling national portability
of health services.
20 Polity & With reference to A Under Article 94, the speaker resigns from his office by The post of E F E Vision IAS Test Series FLT:
Governanc Deputy Speaker of Lok writing to the Deputy Speaker. Deputy M 3499 and Open Test 3374
e Sabha, consider the Statement 4 is not correct: Article 93 provides for “The Speaker, (2022):
following statements: House of the People shall, as soon as may be, choose for the first Consider the following
1. As per the Rules of two members of the House to be respectively Speaker time, has statements with regards to
Procedure and and Deputy Speaker thereof and, so often as the office been the election of deputy
Conduct of Business in of Speaker and Deputy Speaker becomes vacant, the vacant for speaker in Lok Sabha: (3499)
Lok Sabha, the election House shall choose another member to be Speaker or more than 1. He is elected by the Lok
of Deputy Speaker Deputy Speaker, as the case may be.” a year after Sabha itself from amongst its
shall be held on such Statement 1 is correct: Being an important the members.
date as the Speaker constitutional office, the election is usually held on constitution 2. The date of election of the
may fix. next sitting after Speaker selection as President fixes of 17th Lok Deputy Speaker is fixed by the
2. There is a date for Speaker election and after being elected the Sabha. This Speaker.
mandatory provision Speaker fixes the date for Deputy Speaker election. is the first 3. The election should be held
25 www.visionias.in ©Vision IAS
that the election of a Statement 2 is not correct: There is no mandatory time that within six months after the
candidate as Deputy provision that the election of a candidate as Deputy the Lok election of speaker.
Speaker of Lok Sabha Speaker of Lok Sabha shall be from either the principal Sabha has Open Test 3374
shall be from either opposition party or the ruling party. It is by convention functioned The post of Deputy Speaker,
the principal that position of Deputy Speaker is offered to for over a for the first time, has been
opposition party or the opposition party in India. year vacant for more than a year
ruling party. Statement 3 is correct: The Deputy Speaker has the without after the constitution of 17th
3. The Deputy Speaker same power as of the Speaker when presiding over the having a Lok Sabha. In this context,
has the same power as sitting of the House and no appeal lies against his Deputy consider the following
of the Speaker when rulings. Speaker. A statements regarding Deputy
presiding over the Speaker:
sitting of the House 1. He is appointed after the
and no appeal lies election of the Speaker has
against his rulings. taken place.
4. The well established 2. He can preside over the
parliamentary practice joint sitting if the Speaker is
regarding the absent from such a sitting.
appointment of Deputy 3. A member wishing to give
Speaker is that the notice of a resolution for the
motion is moved by removal of the Deputy
the Speaker and duly Speaker shall do so in writing
seconded by the Prime to the Speaker.
Minister.
Which of the
statements given
above are correct ?
(a) 1 and 3 only
(b) 1, 2 and 3
(c) 3 and 4 only
(d) 2 and 4 only
21 Environme Among the following B Paddy fields are anthropogenic sources of atmospheric This E F https://indianex R All India Test Series: Test
nt crops, which one is the nitrous oxide (N2O) and methane (CH4), which have question press.com/articl M 3477
most important been reckoned as 273 and 80-83 times more powerful seeks to e/opinion/colu "It is a Greenhouse gas. It is
mns/for-
anthropogenic source than CO2 in driving temperature increase in 20 years. test the released in large amounts in
climate-smart-
of both methane and Paddies are a potential source of anthropogenic nitrous fundament paddy fields, coal mines, from
agriculture-
nitrous oxide ? oxide (N2O) emission. In paddies, both the soil and the al cop26-budget- rotting garbage dumps. It is
(a) Cotton (b) Rice rice plants emit N2O into the atmosphere. The rice plant knowledge. 7771379/ associated with chemical
(c) Sugarcane(d) Wheat in the paddy is considered to act as a channel between reactions leading to the
the soil and the atmosphere for N2O emission. destruction of ozone. It is also
used as biogas." It is
26 www.visionias.in ©Vision IAS
(a) Nitrous Oxide
(b) Methane
(c) Chlorofluorocarbon
(d) Carbon dioxide
22 Environme "System of Rice D System of Rice Intensification (SRI) was first developed This E FA https://indianex E VisionIAS Open Test 2021,
nt Intensification" of in Madagascar in the 1980s and since then several technique press.com/articl N (3543)
cultivation, in which countries in the world have been practising it, including is in the e/explained/pu In which of the following ways
njab-paddy-
alternate wetting and India. It promises to save 15 to 20% ground water, news for is the System of Rice
sowing-
drying of rice fields is improves rice productivity. sometime Intensification (SRI) different
technique-
practised, results in : in the water-labour- from traditional rice
1. Reduced seed Option 1 is correct: Under SRI 2kg seed is required to context of costs-7938766/ cultivation methods?
requirement grow a nursery for one acre against 5kg seed required in rice 1. Traditional methods
2. Reduced methane the traditional method. cultivation require continuous flooding of
production in India. fields whereas in SRI, there is
3. Reduced electricity Option 2 is correct: SRI is a holistic approach to alternate wetting and drying.
consumption sustainable rice cultivation. By minimizing water use 2. Traditional methods are
Select the correct and alternating wet and dry conditions, it minimizes less labour-intensive while SRI
answer using the code methane production. is initially labour-intensive.
given below : 3. In SRI 8-12 days old
(a) 1 and 2 only Option 3 is correct: With less consumption of water, seedlings are transplanted as
(b) 2 and 3 only adoption of SRI saves total energy inputs. A study found compared to 3-4 week old in
(c) 1 and 3 only in Vietnam has also proved that applying SRI methods traditional methods.
(d) 1, 2 and 3 can save around 23% of energy inputs, while increasing Select the correct answer
energy outputs by 11%. using the code given below.
23 Current Which one of the B Lake Faguibine is an isolated lake in Mali, west of This lake D CA https://thewire. E
Affairs following lakes of West Timbuktu (Tombouctou). It lies north of the Niger River recently in/environment N
Africa has become dry in the Macina depression. Lake Faguibine in northern dried up /chart-climate-
change-the-
and turned into a Mali is dry and has been since the 1970s. Over seven and turned
great-displacer
desert ? years, droughts in the 1970s dried up the lakes. Then into a
(a) Lake Victoria sand filled the channels connecting the lakes to the desert due https://www.un
(b) Lake Faguibine River Niger, with the result that when rain finally to which ep.org/news-
(c) Lake Oguta returned the water could no longer reach the lakes. inhabitants and-
(d) Lake Volta were forced stories/story/be
to leave nefits-galore-if-
that place. malis-lake-
faguibine-
system-were-
revived-0

27 www.visionias.in ©Vision IAS


24 Geography Gandikota canyon of C Gandikota is a village and historical fort on the right This M F https://www.th R VisionIAS Abhyaas Test 2019
South India was bank of the Pennar river, in Kadapa district, Andhra question ehindu.com/life M (2880)
created by which one Pradesh, India. The fort was the centre of power for seeks to -and- This place derives its name
style/travel/gan
of the following rivers. various dynasties, such as the Kalyani Chalukyas, test the from a 12th-century fort,
dikotas-
(a) Cauvery Pemmasani Nayakas, and the Golconda Sultanate. fundament constructed by the Western
majestic-
(b) Manjira al canyon-and- Chalukyan Kings. It also
(c) Pennar knowledge. belums-dark- derives its name from a gorge
(d) Tungabhadra caves-make-for- through which the river
a-great- Pennar flows in the Erramala
weekend- hills. The Mylavaram Dam is
trip/article2469 situated near the fort and its
6541.ece backwaters are key in
supplying water to this gorge
which is also popularly known
as "India's Grand Canyon".
Which of the following places
is best described in the above
passage?
(a) Mysuru (b) Vijayawada
(c) Gandikota (d) Konark
Answer: Gandikota; river
Pennar flows
25 Geography Consider the following B Pair 1 is not correctly matched: Namcha Barwa or This E F Standard E All India Test Series: Test
pairs : Namchabarwa is a mountain peak lying in Tibet in the question Reference Atlas M 3495:
Peak Mountains region of Pemako. It is the easternmost section of the seeks to Arrange the following
1. Namcha Barwa — Himalaya in southeastern Tibet and northeastern India. test the Himalayan peaks from West
Garhwal Himalaya The Garhwal Himalayas are mountain ranges located in the mapping to East.
2. Nanda Devi — Indian state of Uttarakhand. knowledge. 1. Namcha Barwa
Kumaon Himalaya Pair 2 is correctly matched: Nanda Devi is the second- 2. Nanda Devi
highest mountain in India, after Kangchenjunga. It is
3. Nokrek — Sikkim 3. Kanchenjunga
situated in the Chamoli district of Uttarakhand. Kumaun
Himalaya 4. Nanga Parbat
Himalayas, west-central section of the Himalayas in
Which of the pairs Select the correct answer
northern India, extends 200 miles (320 km) from the Sutlej
given above is/are using the code given below.
River east to the Kali River. It rises to 7,817 metres at
correctly matched ? Nanda Devi, the range’s highest peak, and to 7,756 metres (a) 1-2-3-4
(a) 1 and 2 at Kamet, near the Chinese border. (b) 1-2-4-3
(b) 2 only Pair 3 is not correctly matched: The Nokrek Peak is located (c) 2-4-3-1
(c) 1 and 3 in the northeast of India on the Tura Range, which forms (d) 4-2-3-1
(d) 3 only part of the Meghalaya Plateau. The entire area is
mountainous and Nokrek is the highest peak of the Garo
hills, rising up 1,412 metres.
28 www.visionias.in ©Vision IAS
26 Geography The term "Levant" A The Levant is an old term referring to countries of the Syria, M F https://www.bri R News Today- 15th-16th
often heard in the eastern Mediterranean. Some scholars include in it Lebanon tannica.com/pla M October, 2021, Places in news
news roughly Cyprus and a small part of Turkey. But basically the and ce/Levant PT 365 IR- Page 25, Lebanon,
corresponds to which Levant has throughout history meant Syria, Lebanon Palestine Places in news."
of the following and Palestine. This means Jordan, the West Bank (now region
regions ? under Israeli occupation) and Israel itself are part of the often
(a) Region along the Levant. remains in
eastern Mediterranean the news.
shores (b) Region along
North African shores
stretching from Egypt
to Morocco (c) Region
along Persian Gulf and
Horn of Africa (d) The
entire coastal areas of
Mediterranean Sea

29 www.visionias.in ©Vision IAS


27 Geography Consider the following C Afghanistan is bounded to the east and south by Afghanistan M CAA Standard E PT 365 IR- Places in News
countries : Pakistan (including those areas of Kashmir administered was Reference Atlas N from Afghanistan on world
1. Azerbaijan by Pakistan but claimed by India), to the west by Iran, frequently map- Page 25; News Today-
2. Kyrgyzstan and to the north by the Central Asian states of in the 28th Jan, 2022; Monthly
3. Tajikistan Turkmenistan, Uzbekistan, and Tajikistan. news. Current Affairs April 2021,
4. Turkmenistan Page 19
5. Uzbekistan
Which of the above Test 3374 - All India Open
have borders with Test 1(2022)
Afghanistan ?
(a) 1, 2 and 5 only Recently, “6+2+1” group was
(b) 1, 2, 3 and 4 only in news, is related to:
(c) 3, 4 and 5 only (a) mediate a peace process in
(d) 1, 2, 3, 4 and 5 Nagorno-Karabakh conflict.
(b) keeping a watch over the
Iran's nuclear programme.
(c) achieving carbon neutrality
by 2030 in US, China and
India.
(d) establish peace and
stability in Afghanistan.
Explanation mentions: It
includes neighbouring
countries: Iran, Pakistan,
Tajikistan, Turkmenistan and
Uzbekistan.
28 Geography With reference to B Statement 1 is correct: Monazite is a primarily reddish- This E FA https://pib.gov.i E VisionIAS AITS Test 3486 -
India, consider the brown phosphate mineral that contains rare-earth question n/newsite/Print M Which of the following
following statements: elements. Due to variability in composition, monazite is seeks to Release.aspx?re statements is not correct with
lid=112033
1. Monazite is a source considered a group of minerals. test the respect to the atomic
of rare earths. fundament minerals in India?
2. Monazite contains Statement 2 is correct: In addition to the rare-earth al (a) The production of Uranium
thorium. elements, thorium (Th) can be extracted from monazite knowledge. is confined to the mines of
3. Monazite occurs ore. Thorium is a slightly radioactive metal, identical to Kadapa district of Andhra
naturally in the entire uranium. Pradesh
Indian coastal sands in (b) India possesses the
India. Statement 3 is not correct: Atomic Minerals Directorate world’s largest monazite
4. In India, for Exploration and Research (AMD), a constitute unit of reserves which is the principal
Government bodies Department of Atomic Energy (DAE) has estimated the source of thorium
only can process or presence of 11.93 million tonnes of monazite resources
30 www.visionias.in ©Vision IAS
export monazite. in the beach sand mineral placer deposits along the (c) The beach sands of Kerala
Which of the coastal tracts of India. <Insert Table> in Palghat and Quilon districts
statements given contain rich deposits of
above are correct ? Statement 4 is correct: Monazite is a mineral mainly monazite
(a) 1, 2 and 3 only containing rare earths and thorium-a prescribed (d) Uranium is found in
(b) 1, 2 and 4 only substance to be handled by the Department of Atomic igneous and metamorphic
(c) 3 and 4 only Energy (DAE). Accordingly, Indian Rare Earths Ltd. (IREL) rocks in India
(d) 1,2, 3 and 4 wholly owned by the Govt. of India, under the
administrative control of the Dept. of Atomic Energy
(DAE) utilises monazite mainly for production of rare
earth compounds, and thorium, as needed in the
Department of Atomic Energy.
29 Geography In the northern B The summer solstice, also known as estival solstice or This E F https://www.na E News Today- 20th and 21st
hemisphere, the midsummer, occurs when one of Earth's poles has its question tionalgeographi M June, 2021; PT 365 Science
longest day of the year maximum tilt toward the Sun. It happens twice yearly, seeks to c.com/science/a and Technology- Page 47
rticle/what-is-
normally occurs in the : once in each hemisphere. During the Northern test the
summer-winter-
(a) First half of the Hemisphere's summer solstice—which always falls fundament
solstice-answer-
month of June around June 21—the Southern Hemisphere gets its al might-surprise-
(b) Second half of the winter solstice. knowledge. you
month of June GC Leong
(c) First half of the
month of July
(d) Second half of the
month of July
30 Geography Consider the following B Pair 1 is not correctly matched: It is located at the This M F http://www.wii E PT365, Environment- Page 84
pairs : northwest Himalayan biogeopgraphic province of question envis.nic.in/Dat M
Wetland/Lake Kashmir, back of the snow-draped Pir Panchal. seeks to abase/ramsar_ All India Test Series: Test
wetland_sites_8
Location test the 3473
224.aspx
1. Hokera Wetland - Pair 2 is correctly matched: Renuka Wetland is located fundament Consider the following pairs:
Punjab in Himachal Pradesh. It is a natural wetland with al Ramsar Site State
2. Renuka Wetland - freshwater springs and inland subterranean karst knowledge. 1. Deepor Beel : Assam
Himachal Pradesh formations. 2. Hokera wetland : Jammu
3. Rudrasagar Lake - and Kashmir
Tripura Pair 3 is correctly matched: Rudrasagar Lake, also 3. Rudrasagar Lake : Uttar
4. Sasthamkotta -. known as Twijilikma, is a lake located in Melaghar, Pradesh
Tamil Nadu Lake Tripura, India. The Government of India's Ministry of Which of the pairs given
How many pairs given Environment and Forest has identified Rudrasagar as above are correctly matched?
above are correctly one of the wetlands of National Importance for Test 3487:
matched ? conservation and sustainable use based on its bio- Which of the following
(a) Only one pair diversity and socio economic importance. wetlands of North Eastern
31 www.visionias.in ©Vision IAS
(b) Only two pairs Pair 4 is not correctly matched: Sasthamcotta Lake or India are included in the
(c) Only three pairs Sasthamkotta Lake, also categorized as a wetland, is the Ramsar list of wetlands?
(d) All four pairs largest fresh water lake in Kerala. Sasthamkotta lake is 1. Manas
also known as the Queen of Lakes. 2. Deepor Beel
3. Kaziranga
4. Rudrasagar
Select the correct answer
using the code given below.
Test 3490
In the context of recently
seen in news Renukaji Dam
Project, consider the
following statements:
1. The Renukaji Dam Project
has been constructed over
River Giri in Himachal
Pradesh.
2. The cost of the water and
power component of the
project has been equally
shared between the Centre
and the basin states.
Which of the statements
given above is/are correct?
31 S&T Consider the following: D Open-source software (OSS) is computer software that All the four M FCA PIB and the R News Today 6th July 2021
1. Aarogya Setu is released under a license in which the copyright holder platforms websites of the R
2. CoWIN grants users the rights to use, study, change, and were platforms
3. DigiLocker distribute the software and its source code to anyone recently
4. DIKSHA and for any purpose. developed,
Which of the above are DIKSHA: DIKSHA (Digital Infrastructure for Knowledge two of
built on top of open- Sharing) is a national platform for school education, an them were
source digital initiative of National Council for Education Research and created and
platforms? Training (NCERT), Ministry of Education. DIKSHA was have
(a) 1 and 2 only developed based on the core principles of open always
(b) 2, 3 and 4 only architecture, open access, open licensing diversity, been talked
(c) 1, 3 and 4 only choice and autonomy as outlined in the Strategy and about
(d) 1, 2, 3 and 4 Approach Paper for the National Teacher Platform during the
released in 2017. Hence option 4 is correct. COVID 19
DigiLocker is an initiative by the government to offer peak
Indian citizens a free platform to store and access period.
32 www.visionias.in ©Vision IAS
important documents. The platform uses several open
source technologies to deliver a mass solution and
contributes back to the ever-growing community.
Hence option 3 is correct.
COWIN: CoWIN is an Indian government web portal for
COVID-19 vaccination registration, owned and operated
by India's Ministry of Health and Family Welfare. It
displays booking slots of COVID-19 vaccine available in
the nearby areas and can be booked on the website.
Initially it was not an open-source digital platform, but
in July, 2019 it was made open source. Hence option 2
is correct.
Aarogya Setu: On 2nd April 2020, India launched
Aarogya Setu mobile App for helping augment the
efforts of limiting the spread of COVID19, with an
objective of enabling Bluetooth based contact tracing,
mapping of likely hotspots and dissemination of
relevant information about COVID19. It is open source
now. Hence option 1 is correct.
32 S&T With reference to Web D The concept of Web3, also called Web 3.0, is used to Web 3.0 is M FCA https://indianex E Monthly Current Affairs
3.0, consider the describe a potential next phase of the internet. The the future, press.com/articl N December 2021, Page 96;
following statements: model, a decentralised internet to be run on blockchain the e/technology/te News Today- 27th&28th
ch-news-
1. Web 3.0 technology technology, would be different from the versions in use, upcoming March, 2022; Updated PT
technology/tec
enables people to Web 1.0 and Web 2.0. Hence, statement 2 is correct. third 365, Page 99
h-indepth-
control their own data. Web3 enables peer to peer (seller to buyer) generation understanding- VisionIAS Open Test -3699
2. In Web 3.0 world, transactions by eliminating the role of the intermediary. of the web-3- Consider the following
there can be In web3, users will have ownership stakes in platforms internet. It 7833206/ statements regarding Web
blockchain based social and applications unlike now where tech giants control has always 3.0:
networks. the platforms. Web3 will deliver a “decentralized and been in the 1. It is based on blockchain
3. Web 3.0 is operated fair internet where users control their own data”. news. technology.
by users collectively Hence, statements 1 and 3 are correct. 2. It enables peer to peer
rather than a transactions by eliminating
corporation. the role of the intermediary.
Which of the 3. In Web 3.0, most of the
statements given data in the internet is handled
above are correct ? by very few tech companies.
(a) 1 and 2 only Which of the statements
(b) 2 and 3 only given above is/are correct?
(c) 1 and 3 only
(d) 1, 2 and 3
33 www.visionias.in ©Vision IAS
33 S&T With reference to D Software-as-a-Service (SaaS) is a software licensing SaaS is M FCA https://azure.m R News Today 15th April 2022.
"Software as a Service model. It allows access to software on a subscription often seen icrosoft.com/en R
(SaaS)", consider the basis using external servers. As its important feature, in the -
in/overview/wh
following statements: SaaS allows each user to access programs via the news.
at-is-saas/
1. SaaS buyers can Internet. Thus, SaaS allows data to be accessed from
customise the user any device (computer or mobile phones) with an https://www.so
interface and can internet connection and a web browser. The user need ftwareadvice.co
change data fields. not install the software on his/her computer. In simple m/resources/sa
2. SaaS users can words, software-as-a-Service (SaaS)—also known as as-10-faqs-
access their data cloud based software—is now mainstream. software-
through their mobile Today’s web-based software (SaaS) is flexible enough to service/
devices. be modified for specific business uses but also individual
3. Outlook, Hotmail users. Buyers can customize the user interface (UI) to
and Yahoo! Mail are change the look and feel of the program, as well as
forms of SaaS. modify specific areas, such as data fields, to alter what
Which of the data appears. Several business process features can also
statements given be turned off and on at will. Hence statements 1 and 2
above are correct ? are correct.
(a) 1 and 2 only
(b) 2 and 3 only Any web-based email service is classified as SaaS,
(c) 1 and 3 only examples a web-based email service such as Outlook,
(d) 1, 2 and 3 Hotmail or Yahoo! Hence statement 3 is correct.
34 S&T Which one of the C Recently (in October 2021), China was suspected to It was in D FCA https://www.fin E Monthly Current Affairs
following statements have tested Fractional Orbital Bombardment System. the news. ancialexpress.co N Magazine October 2021, Page
best reflects the idea A Fractional Orbital Bombardment System (FOBS) is a m/defence/chin 45
as-fractional-
behind the "Fractional warhead delivery system that uses a low earth orbit
orbital-
Orbital Bombardment towards its target destination. Just before reaching the
bombardment-
System" often talked target, it deorbits through a retrograde engine burn. system-impact-
about in media ? The flight path of the warhead from the system does on-indias-
(a) A hypersonic not reveal the target location till the payload hits the nuclear-
missile is launched into destination. Hence option (c) is the correct answer. deterrence-
space to counter the posture/235647
asteroid approaching 1/
the Earth and explode
it in space.
(b) A spacecraft lands
on another planet after
making several orbital
motions.

34 www.visionias.in ©Vision IAS


(c) A missile is put into
a stable orbit around
the Earth and deorbits
over a target on the
Earth.
(d) A spacecraft moves
along a comet with the
same speed and places
a probe on its surface.
35 S&T Which one of the B Quantum computing holds the promise to solve some of Quantum E FCA https://www.th E News Today 29th and 30th
following-is the our planet's biggest challenges - in the areas of Computing ehindu.com/sci- N May, 2022.; Monthly Current
context in which the environment, agriculture, health, energy, climate, is a cutting tech/technolog Affairs August 2021, Page 79.
y/google-builds-
term "qubit" is materials science, and others we haven't encountered edge VisionIAS Prelims FLT: 3500
qubit-game-to-
mentioned ? yet. For some of these problems, classical computing is technology Consider the differences
teach-students-
(a) Cloud Services increasingly challenged as the size of the system grows. and has quantum- between Qubits and Binary
(b) Quantum Just as bits are the fundamental object of information always computing/artic bits:
Computing in classical computing, qubits (quantum bits) are the been in the le65331917.ece 1. Qubits are typically
(c) Visible Light fundamental object of information in quantum news. subatomic particles such as
Communication computing. While a bit, or binary digit, can have a value electrons or photons, while a
Technologies either 0 or 1, a qubit can have a value that is either 0, 1 bit represents a stream of
(d) Wireless or a quantum superposition of 0 and 1. electrical or optical pulses.
Communication Hence option (b) is the correct answer. 2. While bits can hold only a
Technologies position of 0 or 1, qubits can
hold a superposition of all
possible states.
36 S&T Consider following D Short Range Devices (SRD) are radio devices that offer FASTAG, E F https://www.et R PT 365 Science and
communication the a low risk of interference with other radio services, CCTV and si.org/technolog R Technology, Updated
technologies: usually because their transmitted power, and hence LAN are ies/short-range- Material, Page 100
devices
1. Closed-circuit their range, is low. The definition 'Short Range Device' part of our
Television may be applied to many different types of wireless daily life.
2. Radio Frequency equipment, including various forms of:
Identification Access control (including door and gate openers)
3. Wireless Local Area Alarms and movement detectors
Network Closed-circuit television (CCTV)
Which of the above are Cordless audio devices, including wireless microphones
considered Short- Industrial control
Range Local Area Networks
devices/technologies? Medical implants
(a) 1 and 2 only Metering devices
(b) 2 and 3 only Remote control
35 www.visionias.in ©Vision IAS
(c) 1 and 3 only Radio frequency identification (RFID)
(d) 1, 2 and 3 Hence option (d) is the correct answer.
37 S&T Consider the following D Biofilm is a thin usually resistant layer of Not D FCA https://www.th E
statements : microorganisms (as bacteria) that forms on and coats covered in ehindu.com/sci- N
1. Biofilms can form on various surfaces (as of catheters or water pipes). These any of the tech/science/mi
croplastics-in-
medical implants cells are frequently embedded within a self-produced essential
sewage-
within human tissues. matrix of extracellular polymeric substance (EPS). newspapers
become-hubs-
2. Biofilms can form on Biofilms can form on just about any imaginable surface: , but has for-drug-
food and food metals, plastics, natural materials (such as rocks), been in the resistant-
processing surfaces. medical implants, kitchen counters, contact lenses, the news. bacteria-
3. Biofilms can exhibit walls of a hot tub or swimming pool, human and animal Since, study/article34
antibiotic resistance. tissue, including medical devices and implants. Hence presence of 176654.ece
Which of the statement 1 is correct. biofilms is
statements given They are a constant concern in food processing not https://www.sci
encedirect.com
above are correct? environments. Biofilms of Lactobacillus curvatus could uncommon,
/science/article
(a) 1 and 2 only lead to a defect caused by the formation of calcium the topic
/pii/S00220302
(b) 2 and 3 only lactate crystals in Cheddar cheese and they can be cannot be 98758345
(c) 1 and 3 only present on equipment surfaces (tough to remove). called
(d) 1, 2 and 3 Hence statement 2 is correct. 'unconventi https://pubmed
onal'. .ncbi.nlm.nih.go
Bacteria form biofilms, a kind of matrix, during infection v/16353112/
in plants and animals. The biofilm shields the bacteria
from antibiotics and helps bacteria survive harsh
conditions such as extreme temperature or stress. Thus
they can exhibit antibiotic resistance. Hence statement
3 is correct.
38 Basic Consider the following C Probiotics are live microorganisms (bacteria and yeasts Growing M CAA https://indianex E News Today 12th Feb 2022.
Science statements in respect can form an important part of the human mycobiome as consciousn press.com/articl N Abhyaas Test 2881 (2019)
of probiotics: probiotics.) that are intended to have health benefits ess about e/lifestyle/healt Consider the following
h/probiotics-
1. Probiotics are made when consumed or applied to the body. They can be healthy statements:
help-improve-
of both bacteria and found in yogurt and other fermented foods, dietary eating/diet 1. Prebiotic foods contain live
gut-profile-
yeast. supplements, and beauty products. Hence statement 1 ary habits benefits- microorganisms that help
2. The organisms in is correct. and the 6386487/ human health whereas
probiotics are found in Although people often think of bacteria and other increase in https://www.int Probiotic foods contain fiber
foods we ingest but microorganisms as harmful “germs,” many are actually the echopen.com/c compounds that induce
they do not naturally helpful. Some bacteria help digest food, destroy popularity hapters/39620 growth of microorganisms.
occur in our gut. disease-causing cells, or produce vitamins. Many of the of the 2. Prebiotics resist digestion in
3. Probiotics help in microorganisms in probiotic products are the same as or the small intestine and reach
the digestion of milk similar to microorganisms that naturally live in our the colon where they are
sugars. bodies. Hence statement 2 is not correct. fermented by gut microflora.
36 www.visionias.in ©Vision IAS
Which of the There is evidence that probiotics can alleviate 3. Fermented foods like
statements given symptoms of lactose intolerance. This can occur by Yogurt and Buttermilk are rich
above is/are correct ? increased hydrolysis (breaking down or digestion) of sources of Probiotics.
(a) 1 only lactose (milk sugar) in the dairy product and in the small Which of the statements
(b) 2 only intestine. It can also be achieved by manipulation of the given above is/are correct?
(c) 1 and 3 colonic metabolism. Hence statement 3 is correct.
(d) 2 and 3
39 S&T In the context of B Covishield and Covaxin were the first vaccines The E CA https://indianex E News Today, 13th and 14th
vaccines manufactured permitted to be administered to people in India in the importance press.com/articl N April, 2021; 6th Jan, 2022
to prevent COVID-19 first half of 2021. Sputnik came weeks later. of the topic e/india/booster Weekly Focus- Universal
-must-be-
pandemic, consider the Covishield: It is not an mRNA-based vaccine. Covishield cannot be immunisation.
different-
following statements : (produced by Serum Institute of India) is a recombinant, overstated. Vision IAS Prelims Test
vaccine-expert-
1. The Serum Institute replication-deficient chimpanzee adenovirus vector body-7669635/ Series: Test 3471
of India produced encoding the SARS-CoV-2 Spike (S) glycoprotein. Consider the following
COVID-19 vaccine Following administration, the genetic material of part of statements regarding the
named Covishield corona virus is expressed which stimulates an immune adenovirus based vaccine:
using mRNA platform. response. Hence statement 1 is not correct. By using 1. They work by triggering an
2. Sputnik V vaccine is elimination technique, one can reach option (b). Hence immune response by putting
manufactured using option (b) is the correct answer. a weakened or inactivated
vector based platform. Covaxin: It is an indigenous, inactivated pathgen type germ into our bodies.
3. COVAXIN is an vaccine developed and manufactured in Bharat 2. Recently, Sputnik V has
inactivated pathogen Biotech's BSL-3 (Bio-Safety Level 3) high containment been approved for emergency
based vaccine. facility. The vaccine is developed using Whole-Virion use in India which is an
Which of the Inactivated Vero Cell derived platform technology. adenoviral vector-based
statements given Hence statement 3 is correct. vaccine.
above are correct? Sputnik V: It is an adenovirus viral vector vaccine for Which of the statements
(a) 1 and 2 only COVID-19 developed by the Gamaleya Research given above is/are correct?
(b) 2 and 3 only Institute of Epidemiology and Microbiology in Russia. It (a) 1 only (b) 2 only
(c) 1 and 3 only is the world's first registered combination vector (c) Both 1 and 2
(d) 1, 2 and 3 vaccine for the prevention of COVID-19, having been (d) Neither 1 nor 2
registered on 11 August 2020 by the Russian Ministry of 3443 (2021 Abhyaas)
Health. Hence statement 2 is correct. In the explanation to the
following question
Recently CSIR has successfully
created a new agonist
molecule with indigenous
chemicals for the production
of COVAXIN. In this context,
which of the following
statements best describes an
37 www.visionias.in ©Vision IAS
agonist molecule?
(a) It is a molecule that binds
to specific receptors and
cause a process in the cell to
become more active.
(b) It is a large protein
molecule embedded in the
cell wall or membrane where
a specific drug molecule
attaches itself.
(c) It is the active live virus in
a vaccine that generates a
response from the human
immune system.
(d) It is a polynucleotide that
does not have the ability to
trigger the host immune
system.
40 S&T If a major solar storm C Solar flares occur when magnetic energy builds up in Intense M FA https://www.na E News Today- 23-24th Jan,
(solar-flare) reaches the solar atmosphere and is released suddenly. These solar flares sa.gov/mission_ N 2022; VisionIAS Monthly
the Earth, which of the outbursts are intrinsically linked to the solar cycle — an posing a pages/sunearth Current Affairs Magazine Jan
/news/flare-
following are the approximately 11-year cycle of solar activity driven by threat to 2022, 7.5.10; PT 365, Science
impacts.html
possible effects on the the sun's magnetic field. These are large explosions specific and Technology, Page 39
Earth ? from the surface of the sun that emit intense bursts of regions of https://www.liv Vision IAS Prelims Test
1. GPS and navigation electromagnetic radiation. According to NASA, harmful Asia was in emint.com/scie Series: Test 3500
systems could fail. radiation from a flare cannot pass through Earth’s the news. nce/news/stron In the context of solar flares,
2. Tsunamis could atmosphere to physically affect humans on the ground. g-solar-flares- consider the following
occur at equatorial The solar flares, even the intense ones, do not pose an emitted-from- statements:
regions. immediate threat to us, at least those who are within sun-towards- 1. Solar flares are associated
3. Power grids could be the Earth's atmosphere. Flares are not fires but earth-how-it- with solar magnetic storms
may-impact-us-
damaged. outbursts of electromagnetic radiation. Thus they will known as Coronal Mass
1163557489335
4. Intense auroras not cause forest fires. Hence statement 5 is not correct. Ejection.
8.html
could occur over much Tsunami is a purely seismic phenomenon. Earthquakes, 2. The harmful radiations
of the Earth. volcanic eruptions and other underwater explosions https://www.sp from such a flare cannot pass
5. Forest fires could (including detonations, landslides, glacier calvings, ace.com/solar- through Earth's atmosphere
take place over much meteorite impacts and other disturbances) above or flares-effects- to physically affect humans on
of the planet. below water all have the potential to generate a classification- the ground.
6. Orbits of the tsunami and not solar flares. Hence statement 2 is not formation Which of the statements
satellites could be correct. By elimination technique, one can easily reach given above is/are correct?
https://www.sp
disturbed. option (c). Hence option (c) is the correct answer.
38 www.visionias.in ©Vision IAS
7. Shortwave radio Strong M-class and X-class solar flares and can trigger ace.com/solar-
communication of the coronal mass ejections — a large release of plasma and flares-effects-
aircraft flying over magnetic field from the sun. This behavior can disrupt classification-
formation
polar regions could be Earth's magnetosphere and result in geomagnetic
interrupted. storms. Such geomagnetic storms can lead to auroras
https://www.hi
Select the correct closer to the equator than is possible during calm ndustantimes.c
answer using the code conditions. Hence statement 4 is correct. om/science/hug
given below : Known as a coronal mass ejection or CME these solar e-solar-flare-
(a) 1, 2, 4 and 5 only explosions propel bursts of particles and electromagnetic over-india-
(b) 2, 3, 5, 6 and 7 only fluctuations into Earth's atmosphere. Those fluctuations southeast-asia-
(c) 1, 3, 4, 6 and 7 only could induce electric fluctuations at ground level that could radio-gps-
(d) 1, 2, 3, 4, 5, 6 and 7 blow out transformers in power grids. A CME's particles blackout-likely-
can also collide with crucial electronics onboard a satellite what-you-
and disrupt its systems. The following impacts were should-know-
predicted in April 2022 owing to the huge solar flares over 1016504444152
South Asia and India. 50.html
Expected high-frequency communication blackouts,
satellite anomalies, GPS scintillations, airline
communication impacts.

39 www.visionias.in ©Vision IAS


41 Environme "Climate Action A The Climate Action Tracker is an independent scientific Recently, M CA https://indianex E
nt Tracker, which analysis that tracks government climate action and data from press.com/articl N
monitors the emission measures it against the globally agreed Paris Climate e/world/climate
-change/united-
reduction pledges of Agreement aim of "holding warming well below 2°C, Action
nations-cop26-
different countries is a: and pursuing efforts to limit warming to 1.5°C." A Tracker
summit-carbon-
(a) Database created collaboration of two organisations, Climate Analytics reveals emission-
by coalition of research and NewClimate Institute, the CAT has been providing emissions 7604479/
organisations this independent analysis to policymakers since 2009. pathways
(b) Wing of CAT quantifies and evaluates climate change mitigation for the
"International Panel of targets, policies and action. It also aggregates country world’s 10
Climate Change" action to the global level, determining likely biggest
(c) Committee under temperature increases during the 21st century using the polluters.
"United Nations MAGICC climate model. CAT further develops sectoral They
Framework Convention analysis to illustrate required pathways for meeting the account for
on Climate Change" global temperature goals. more than
(d) Agency promoted two-thirds
and financed by United CAT tracks 39 countries and the EU covering around of
Nations Environment 85% of global emissions greenhouse
Programme and World CAT covers all the biggest emitters and a representative gas
Bank sample of smaller emitters covering about 85% of global emissions.
emissions and approximately 70% of global population.
42 Environme Consider the following B Statement 1 is correct: The Climate Group is a non- Recently, D CAA https://www.th R
nt statements : profit organisation that works with business and Dalmia eclimategroup.o M
1. "The Climate Group" government leaders around the world to address Cement rg/ep100-faqs
is an international non- climate change. The group has programmes focussing and JSW
https://indiacsr.
profit organization that on renewable energy and reducing greenhouse gas Cement –
in/mahindra-
drives climate action emissions. two leading mahindra-
by building large Indian becomes-first-
networks and runs Statement 2 is not correct and Statement 3 is correct: cement company-to-
them. EP100 is a global initiative led by the international non- companies join-ep100-
2. The International profit Climate Group, bringing together over 120 energy - campaign-led-
Energy Agency in smart businesses committed to measuring and committed by-the-climate-
partnership with the reporting on energy efficiency improvements. The to the group/
Climate Group Climate Group’s global EP100 initiative, delivered in Climate
https://www.th
launched a global partnership with the Alliance to Save Energy, brings Group’s
eclimategroup.o
initiative "EP100". together leading companies improving their energy flagship rg/our-
3. EP100 brings productivity; using less energy to achieve higher business work/news/indi
together leading economic output. initiatives as-road-cop26-
companies committed RE100, summit-
to driving innovation in EV100 and mobilising-

40 www.visionias.in ©Vision IAS


energy efficiency and Statement 4 is correct: Mahindra & Mahindra Ltd., the EP100. business-
increasing world’s largest manufacturer of tractors, has signed up commitment-
competitiveness while to EP100. The Indian-based company has committed to and-action-
climate
delivering on emission doubling their energy productivity by 2030, a core
reduction goals. requirement for any business signing on to the
4. Some Indian campaign. Recently, Dalmia Cement and JSW Cement –
companies are two leading Indian cement companies - committed to
members of EP100. the Climate Group’s flagship business initiatives RE100,
5. The International EV100 and EP100. Dalmia Cement committed to EV100,
Energy Agency is the while they are already members of RE100 and EP100.
Secretariat to the JSW Cement, committed to all the three campaigns in
"Under2 Coalition". one go.
Which of the
statements given Statement 5 is not correct: It is the largest global
above are correct? network of state and regional governments committed
(a) 1, 2, 4 and 5 to reducing emissions in line with the Paris Agreement.
(b) 1, 3 and 4 only
(c) 2, 3 and 5 only
(d) 1, 2, 3, 4 and 5
43 Environme "If rainforests and D Tropical rainforests are often called the “lungs of the Wetlands E FA https://science. E VisionIAS Test 3477
nt tropical forests are the planet” because they generally draw in carbon dioxide and their thewire.in/envir M Which of the following are
lungs of the Earth, then and breathe out oxygen. Natural wetlands have often functions is onment/explain potential benefits of coastal
ed-wetlands-
surely wetlands been referred to as "earth's kidneys" because of their an marshes and wetlands?
why-are-they-
function as its high and long-term capacity to filter pollutants from the important
important/
kidneys." Which one of water that flows through them. and 1. They act as migration areas
the following functions fundament for birds and animals.
of wetlands best Aquatic plants can uptake large amounts of metals from al topic. 2. They purify and recharge
reflects the above water and/or sediment through active and passive groundwater.
statement? absorption, with this absorption capacity of metals 3. They prevent
(a) The water cycle in through different organs such as roots, stems, and contamination of wells in the
wetlands involves leaves, making these plants suitable for heavy metal coastal areas.
surface runoff, subsoil alterations in the aquatic environment. 4. They protect the coastlines
percolation and from storms and cyclones.
evaporation. Select the correct answer
(b) Algae form the using the code given below.
nutrient base upon (a) 1, 2, 3 and 4
which fish, (b) 1, 2 and 4 only
crustaceans, molluscs, (c) 2, 3 and 4 only
birds, reptiles and (d) 1 and 3 only
mammals thrive.
41 www.visionias.in ©Vision IAS
(c) Wetlands play a Explanation Mentions:
vital role in Coastal Wetlands can improve
maintaining water quality by removing
sedimentation balance pollutants from surface
and soil stabilization. waters. Three pollutant
(d) Aquatic plants removal processes provided
absorb heavy metals by wetlands are particularly
and excess nutrients. important: sediment trapping,
nutrient removal, and
chemical detoxification.
They also enhances or
protects water quality
through chemical action, by
the removal of nutrients, by
the retention or removal of
sediments or organic matter,
or by moderating the adverse
water quality effects of soil
erosion or stormwater runoff.

Monthly Current Affairs, Feb


2022, Page 70
44 Environme In the context of WHO B Statement 1 is correct: According to the guidelines, The Recently M CAA https://www.w E News Today 23rd March,
nt Air Quality Guidelines, annual average for PM2.5 should not exceed 5 the World ho.int/news- N 2022; Monthly Current
consider the following micrograms per cubic metre of air, while the 24-hour Health room/fact- Affairs November 2021, Page
sheets/detail/a
statements : average should not exceed 15 micrograms per cubic Organisatio 63; PT 365 Environment Page
mbient-
1. The 24-hour mean metre. n’s new air 29
(outdoor)-air-
of PM 2.5 should not quality quality-and-
exceed 15 µg/m3 and Statement 2 is not correct: Ozone at ground level is one guidelines, health Vision IAS Test Series: Test
annual mean of PM2.5 of the major constituents of photochemical smog. It is Global Air 3495: Partially solved
should not exceed 5 formed by the reaction with sunlight (photochemical Quality 3495
µg/m3. reaction) of pollutants such as nitrogen oxides (NOx) Guidelines The World Health
2. In a year, the highest from vehicle and industry emissions and volatile organic (AQGs) Organisation’s new air quality
levels of ozone compounds (VOCs) emitted by vehicles, solvents and were guidelines, Global Air Quality
pollution occur during industry. As a result, the highest levels of ozone released in Guidelines (AQGs) released in
the periods of pollution occur during periods of sunny weather. 2021. 2021 recommend air quality
inclement weather. (Inclement weather is unpleasant, especially with cold levels for which of the
3. PM10 can penetrate wind and rain.) following pollutants?
the lung barrier and 1. Carbon Dioxide
enter the bloodstream. 2. Ozone
42 www.visionias.in ©Vision IAS
4. Excessive ozone in Statement 3 is not correct: While particles with a 3. Carbon Monoxide
the air can trigger diameter of 10 microns or less, (≤ PM10) can penetrate 4. Nitrogen Dioxide
asthma. and lodge deep inside the lungs, the even more health- Select the correct answer
Which of the damaging particles are those with a diameter of 2.5 using the code given below.
statements given microns or less, (≤ PM2.5). PM2.5 can penetrate the Explanation mentions: The
above are correct ? lung barrier and enter the blood system. WHO’s new air quality
(a) 1, 3 and 4 guidelines, Global Air Quality
(b) 1 and 4 only Statement 4 is correct: Excessive ozone in the air can Guidelines (AQGs) has
(c) 2, 3 and 4 have a marked effect on human health. It can cause redefined the threshold of
(d) 1 and 2 only breathing problems, trigger asthma, reduce lung safe air.
function and cause lung diseases. This is the first revision after
the last updation in 2005.
WHO’s new guidelines
recommend air quality levels
for 6 pollutants, where
evidence has advanced the
most on health effects from
exposure.
These are Particulate Matter
(PM 2.5 AND PM 10), ozone
(O₃), nitrogen dioxide (NO₂)
sulfur dioxide (SO₂) and
carbon monoxide (CO).
- The annual average for
PM2.5 should not exceed 5
micrograms per cubic metre
of air, while the 24-hour
average should not exceed 15
micrograms per cubic metre.
45 Current With reference to C Statement 1 is correct: Grown in the foothills of the First time in D FCA https://www.th E
Affairs "Gucchi" sometimes Himalayas, Gucchi mushrooms are the costliest among India, ehindubusinessl N
mentioned in the fungi in the world. They are also known as Morel world's ine.com/econo
my/agri-
news, consider the Mushrooms, and are a rare variety that cannot be costliest
business/gi-tag-
following statements: cultivated commercially. mushroom
sought-for-
1. It is a fungus. (gucchi) indias-costliest-
2. It grows in some Statement 2 is correct: Gucchi mushrooms grow in was mushroom/artic
Himalayan forest clusters on logs in decaying wood and are mostly cultivated le33594977.ece
areas. cultivated in parts of India like Himachal Pradesh, artificially
3. It is commercially Uttarakhand and Jammu and Kashmir. in Himachal https://www.tri
cultivated in the Pradesh. buneindia.com/

43 www.visionias.in ©Vision IAS


Himalayan foothills of Statement 3 is not correct: The fresh morel mushroom Also news/himachal/
north-eastern India. season is limited to a barely few weeks in the spring. It recently in first-time-in-
Which of the is collected from the wild habitats in northwestern Jan. 2022 india-worlds-
costliest-
statements given Himalayas by the locals" GI tag was
mushroom-
above is/are correct? sought for
cultivated-
(a) 1 only gucchi. artificially-
(b) 3 only 217432
(c) 1 and 2
(d) 2 and 3
46 Environme With reference to A Polyethylene terephthalate (PET or PETE), a strong, stiff PET is D FA https://econom E
nt polythylene synthetic fibre and resin and a member of the polyester commonly ictimes.indiatim N
terephthalate, the use family of polymers. PET is spun into fibres for used in es.com/news/h
ow-to/how-are-
of which is so permanent-press fabrics and blow-molded into everyday
plastic-pet-
widespread in our daily disposable beverage bottles. food and
bottles-
lives, consider the Statement 1 is correct: They are often used in durable- beverage recycled-into-
following statements: press blends with other fibres such as rayon, wool, and packaging. clothing/articles
1. Its fibres can be cotton, reinforcing the inherent properties of those how/92016415.
blended with wool and fibres. cms
cotton fibres to Statement 2 is not correct: Plastic packaging is
reinforce their dangerous to human health and the country liquor and https://www.fr
properties. country made foreign liquor cannot be sold in such eepressjournal.i
n/mumbai/plast
2. Containers made of bottles.
ic-industry-
it can be used to store Statement 3 is correct: PET bottles are made of one of
opposes-ban-
any alcoholic beverage. the few polymers that can be recycled into the same on-alcohol-sale-
3. Bottle made of it can form – a new beverage bottle – again and again. During in-pet-bottles
be recycled into other the process the PET is generally blended in a ratio of
products. virgin to recycled, to give strength to the material for https://www.liv
4. Articles made of it use in a new product. Some other everyday items emint.com/Opi
can be easily disposed recycled plastic bottles can be made into are: Plastic nion/95WEIYfc
of by incineration packaging, Clothing and shoes, Carpets and soft GMwJa6t84oEf
HL/PET-bottles-
without causing furnishings, Furniture, Automotive parts etc.
used-for-soft-
greenhouse gas Statement 4 is not correct: Globally, in this year alone,
drinks-and-
emissions. researchers estimate that the production and juices-are-
Which of the incineration of plastic will pump more than 850 million unsafe.html
statements given tonnes of greenhouse gases into the atmosphere.
above are correct? Producing a 16 oz. PET bottle generates more than 100
(a) 1 and 3 times the toxic emissions to air and water than making
(b) 2 and 4 the same size bottle out of glass.
(c) 1 and 4
(d) 2 and 3
44 www.visionias.in ©Vision IAS
47 Environme Which of the following A Mahseer roughly translates as mahi – fish and sher – Recently E F https://indianex E PT 365 Environment Page 57
nt is not a bird? tiger, and hence is also referred as tiger among fish. It the THE press.com/articl N Monthly Current Affairs June
(a) Golden Mahseer is a large cyprinid and known to be the toughest among BLUE- e/india/blue- 2021, Page 98"
finned-
(b) Indian Nightjar the fresh water sport fish. FINNED
mahseer-out-of-
(c) Spoonbill Mahseer Vision IAS Test Series: Test
iucn-red-list-
(d) White Ibis The Indian nightjar is a small nightjar which is a was moved 7343295/ 3479 (Major types of
resident breeder in open lands across South Asia and to the ‘least Mahseer covered in
Southeast Asia. The first bird to be called a "nightjar" concern’ explanation)
was the European Nightjar (Caprimulgus europaeus) status in
which was so named in the 17th-century because it was the The Blue-Finned Mahseer,
active at night and made a jarring noise (night + jar = Internation which was on the
nightjar). al Union for International Union for
Conservatio Conservation of Nature’s
Spoonbills are a genus, Platalea, of large, long-legged n of (IUCN) red list as
wading birds. The spoonbills have a global distribution, Nature’s ‘endangered', has now moved
being found on every continent except Antarctica. The (IUCN) red to the ‘least concern’ status.
genus name Platalea derives from Ancient Greek and list of In this context consider the
means "broad", referring to the distinctive shape of the endangered following statements
bill. species. regarding mahseer:
However, 1. They prefer clean and fast
White Ibises are wetland birds. They use freshwater the Golden flowing waters.
marshes, coastal estuaries, mangroves, flooded Mahseer 2. They are only found in
pastures, mudflats, and swamps. continues Himalayan waters in India.
to remain Which of the statements
on that list. given above is/are correct?
(a) 1 only
(b) 2 only
(c) Both 1 and 2
(d) Neither 1 nor 2

Test 3971
Which of the following types
of Mahseer are found in
India?
1. Golden Mahseer
2. Red Finned Mahseer
3. Blue Fin Mahsee
4. Orange Finned Mahseer
Select the correct answer
using the code given below.
45 www.visionias.in ©Vision IAS
(a) 1 and 2 only
(b) 3 and 4 only
(c) 1, 2 and 4 only
(d) 1, 2, 3 and 4
48 Environme Which of the following A Alfalfa, also called lucerne, is a perennial flowering plant Nitrogen M FA https://ncert.ni R Vision IAS Test Series: Test
nt are nitrogen-fixing in the legume family Fabaceae. It is cultivated as an fixation and c.in/ncerts/l/ke M 3487
plants? important forage crop in many countries around the nitrogen- bo112.pdf Consider the following
1. Alfalfa world. Symbiotic N2 fixation by alfalfa provides fixing plants statements about the
2. Amarnath substantial amounts of nitrogen (N) to livestock is a nitrogen cycle:
3. Chickpea operations, subsequent crops, and soil organic matter. fundament 1. Nitrogen in the atmosphere
4. Clover Hence option 1 is correct. al topic. cannot be directly used as a
5. Purslane (Kulfa) nutrient by any plants or
6. Spinach Amarnath: Amaranthus is a cosmopolitan genus of animals.
Select the correct annual or short-lived perennial plants collectively 2. Ammonia can be a direct
answer using the code known as amaranths. Hence option 2 is not correct. source of nitrogen for some
given below: plants.
(a) 1, 3 and 4 only Chickpea: The chickpea or chick pea is an annual legume 3. The nitrates present in the
(b) 1, 3, 5 and 6 only of the family Fabaceae. Hence option 3 is correct. soil are reduced to nitrous
(c) 2, 4, 5 and 6 only oxide gas by the process of
(d) 1, 2, 4, 5 and 6 Clover: Nitrogen is "fixed" in clovers through a symbiotic ammonification.
relationship with Rhizobium bacteria that infects the Which of the statements
plant's roots. Hence option 4 is correct. given above is/are correct?
(a) 1, 2 and 3
Purslane: Purslane is a green, leafy vegetable that can (b) 1 and 2 only
be eaten raw or cooked. It is known scientifically as (c) 3 only
Portulaca oleracea, and is also called pigweed, little (d) None
hogweed, fatweed and pusley. This succulent plant
contains about 93% water. It has red stems and small, Explanation mentions:
green leaves. Hence option 5 is not correct. Nitrogen fixing bacteria feed
off the root nodules of certain
Spinach: Spinach is a leafy green flowering plant native plant species such as beans,
to central and western Asia. It is of the order peas and alfalfa while they fix
Caryophyllales, family Amaranthaceae. Hence option 6 nitrogen.
is not correct.
49 Current "Biorock technology" is A Biorock technology is an innovative process originally This E CA "https://www.gl E VisionIAS Abhyaas Test 3444
Affairs talked about in which invented in 1976 by the late architect to produce technology obalcoral.org/bi N (2021)
one of the following natural building materials in the sea. Biorock was in the orock-coral- Which of the following are the
reef-marine-
situations? technology has been successfully applied to fish and news. applications of Biorock
habitat-
(a) Restoration of shellfish mariculture as well as to growing limestone technology?
restoration/
damaged coral reefs breakwaters to protect islands and coastal areas from 1. Tidal power generation
46 www.visionias.in ©Vision IAS
(b) Development of erosion and rising sea levels. It is a unique method that https://www.th 2. Erosion control measures
building materials allows coral reefs, and other marine ecosystems ehindu.com/sci- 3. Coral reef restoration
using plant residues including seagrass, salt marsh, mangrove, and oyster tech/science/in 4. Extraction of rare earth
dia-begins-
(c) Identification of reefs to survive and recover. metals
coral-
areas for Hence option (a) is the correct answer. 5. Shoreline protection
restoration-in-
exploration/extraction gulf-of- Select the correct answer
of shale gas kachchh/article using the code given below.
(d) Providing salt licks 30645770.ece" (a) 1, 2 and 4 only
for wild animals in (b) 3, 4 and 5 only
forest/protected areas (c) 2, 3 and 5 only
(d) 1, 2, 3, 4 and 5"
50 Current The "Miyawaki C Urban forests are created through Miyawaki, an Miyawaki E CA https://indianex E Weekly Focus- Nature-
Affairs method" is well known afforestation method based on the work of Japanese technique press.com/articl N Positive Cities
for the: botanist Akira Miyawaki in the 1980s. The technique has been in e/cities/mumba MCAR (Monthly Current
i/urban-forests-
(a) Promotion of compresses layers of a forest – shrubs, trees, canopies news Affairs - March) Test 3463
miyawaki-
commercial farming in – on small plots of land, turning them into tiny forests. frequently. Which of the following are the
technique-
arid and semi-arid In the Miyawaki technique, various native species of helps-create- benefits of afforestation
areas plants are planted close to each other so that the greens dense-green- through Miyawaki technique?
(b) Development of receive sunlight only from the top and grow upwards patches-in-one- 1. High Carbon-dioxide
gardens using than sideways. As a result, the plantation becomes year-7218624/ absorption as compared to
genetically modified approximately 30 times denser, grows 10 times faster conventional forest. 2. Faster
flora and becomes maintenance-free after a span of 3 years. regeneration 3. High ability to
(c) Creation of mini Hence option (c) is the correct answer. induce rain
forests in urban areas
(d) Harvesting wind
energy on coastal
areas and on sea
surfaces.
51 Modern In the Government of C Montagu-Chelmsford or Montford Reforms. Based on This M F Modern-India- E All India Test Series Test
History India Act 1919, the these, the Government of India Act, 1919 was enacted. question Bipan-Chandra M 3475-Explanation:
functions of Provincial Main Features: seeks to Old NCERT page In the context of Montague-
263-Chapter-
Government were The Provincial Legislative Councils were enlarged and test the Chelmsford reforms, which of
Struggle for
divided into "Reserved the majority of their members were to be elected. fundament the statements are correct?
Swaraj
and Transferred" The Act introduced dyarchy for the executive at the al 1. Dyarchy was introduced at
subjects. Which of the level of the provincial government. knowledge. the level of the Provincial
following were treated The Dyarchy system provided more powers to the government.
as "Reserved" Provincial governments. Under this system some 2. Majority of the members of
subjects? subjects, such as Land revenue administration, famine the Provincial Legislative
1. Administration of relief, irrigation, administration of justice, law and Councils were to be
Justice order, newspapers, borrowing, forests etc., were called nominated.
47 www.visionias.in ©Vision IAS
2. Local Self- 'reserved' subjects and remained under the direct 3. Despite the reforms, the
Government control of the Governor; Central Government had
3. Land Revenue others such as education, public health & sanitation, unrestricted control over the
4. Police public works, agriculture, fisheries, religious provincial governments.
Select the correct endowments, local self governments, medical services 4. Congress leader
answer using the code etc, were called 'transferred' subjects and were to be Surendranath Banerjea was in
given below: controlled by ministers responsible to the legislatures. favor of accepting the
(a) 1, 2 and 3 In other words, the subjects which were considered of reforms.
(b) 2, 3 and 4 key importance for the welfare of the masses and for Select the correct answer
(c) 1, 3 and 4 maintaining peace and order in the state were classified using the code given below.
(d) 1, 2 and 4 as reserved, while subjects in which there was more Explanation:
local interest were treated as transferred. The Act introduced dyarchy
Hence option (c) is the correct answer. for the executive at the level
of the provincial government.
The Dyarchy system provided
more powers to the Provincial
governments. Under this
system some subjects, such
as finance and law and order,
were called 'reserved'
subjects and remained under
the direct control of the
Governor; others such as
education, public health, and
local self-government, were
called 'transferred' subjects
and were to be controlled by
ministers responsible to the
legislatures. The Governor
could overrule the ministers
on any grounds that he
considered special.
52 Medieval In medieval India, the B Coins were called by popular names such as : anna, This D F https://www.hi R
History term "Fanam" referred cash, dhinglo, dokdo, doudou, dub, escudo, fanam, question ndustantimes.c R
to: faruqi, karshapara, kas, kon, mohur, naya paisa, pagoda, seeks to om/india/it-s-
all-about-the-
(a) Clothing panam, pice, pie, rupia, suvarna, tanga, and tanka. test the
money/story-
(b) Coins The fanam was a small coin used in south India fundament
KCUOWCc5xpD
(c) Ornaments between the 9th and 19th centuries. These small gold al AQr5fC0QsdM.h
(d) Weapons coins weighed as little as 0.3 grams and had a diameter knowledge. tml
of less than a centimetre.
48 www.visionias.in ©Vision IAS
Hence option (b) is the correct answer. https://www.fir
stpost.com/busi
ness/suvarna-
mohur-a-
journey-
through-indias-
numismatic-
history-
through-133-
rare-coins-
5889281.html

https://timesofi
ndia.indiatimes.
com/city/benga
luru/tales-of-
the-
rupee/articlesh
ow/71130395.c
ms

49 www.visionias.in ©Vision IAS


53 Modern Consider the following D Hindustan Republican Association/Army or HRA (later This M F Pg 245-246, Old E All India Test Series Test
History freedom fighters: renamed Hindustan Socialist Republican Association or question NCERT (Bipin M 3495:
1. Barindra Kumar HSRA). The HRA was founded in October 1924 in Kanpur seeks to Chandra) In this historic case, the whole
Ghosh by Ramprasad Bismil, Jogesh Chandra Chatterjee and test the Anushilan group was put on
A Brief History
2. Jogesh Chandra Sachin Sanyal. fundament trial and was charged with
of India Pg 284
Chatterjee The first revolutionary groups in Bengal were organised al "Conspiracy" or "waging war
3. Rash Behari Bose in 1902 in Midnapore (under Jnanendranath Basu) and knowledge. against the King". The case
Who of the above in Calcutta (the Anushilan Samiti founded by Promotha took nearly one year to
was/were actively Mitter, and including Jatindranath Banerjee, Barindra complete and came to a close
associated with the Kumar Ghosh and others.) in 1909. In this case,
Ghadar Party? Rash Behari Bose was one of the most prominent Aurobindo Ghosh was
(a) 1 and 2 leaders of the Indian National Movement, especially defended by Chittaranjan das
(b) 2 only during the extremist phase. He was one of the and was acquitted but
(c) 1 and 3 masterminds behind the Delhi conspiracy case, prominent leaders like
(d) 3 only participated in the Ghadar movement, and established Hemchandra Kanungo, Barin
the Indian Independence league while in Japan. Ghosh, etc were convicted.
Which of the following cases
is being described in the
above passage?
(a) Alipore Bomb Case
(b) Lahore Conspiracy Case
(c) Kakori train robbery Case
(d) Delhi Conspiracy Case

3475-
He was one of the
masterminds of the Delhi
conspiracy which involved the
throwing of a bomb on
Viceroy Hardinge. He went to
Punjab to assume the
leadership of the Ghadar
movement. He also
established the Indian
Independence League while in
Japan.

Which of the following


leaders is being to in the
passage given above?
50 www.visionias.in ©Vision IAS
(a) Sachin Sanyal
(b) Rash Behari Bose
(c) Sohan Singh Bhakna
(d) Lala Har Dayal
54 Modern With reference to the B The Cripps mission was an attempt in late March 1942 This M F India's Struggle E All India Test Series: Test
History proposals of Cripps by the British government to secure full Indian question for M 3483
Mission, consider the cooperation and support for their efforts in World War seeks to Independence Which of the following
(Bipan
following statements: II. The mission was headed by Sir Stafford Cripps, a test the statements is not correct
Chandra)- Page
1. The Constituent senior left-wing politician and government minister in fundament regarding the Cripps Mission?
484-486
Assembly would have the War Cabinet, who had actively supported the Indian al (a) It proposed a plan for a
members nominated national movement. knowledge. Constituent Assembly.
by the Provincial The main proposals of the mission were as follows: (b) It provided that the Indian
Assemblies as well as - An Indian Union with a dominion status would be set Union would be free to decide
the Princely States. up; it would be free to decide its relations with the its relations with the
2. Any Province, which Commonwealth. Commonwealth and free to
is not prepared to - After the end of the war, a Constituent Assembly participate in the United
accept the new would be convened to frame a new constitution. Nations.
Constitution would Members of this assembly would be partly elected by (c) It proposed separate
have the right to sign a the provincial assemblies through proportional constitution for provinces
separate agreement representation and partly nominated by the princes. unwilling to join the Indian
with Britain regarding Hence statement 1 is not correct. dominion.
its future status. - The British government would accept the new (d) It provided the right of
Which of the constitution subject to two conditions: self-determination to the
statements given any province not willing to join the Union could have a Muslims and the creation of
above is/are correct? separate constitution and form a separate Union, and Pakistan.
(a) 1 only the new constitution- making body and the British Explanation: proposed a plan
(b) 2 only government would negotiate a treaty to effect the for a Constituent Assembly;
(c) Both 1 and 2 transfer of power and to safeguard racial and religious the new constitution-making
(d) Neither 1 nor 2 minorities. Hence statement 2 is correct. body and the British
In the meantime, defence of India would remain in government would negotiate
British hands and the governor-general’s powers would a treaty to effect the transfer
remain intact. of power and to safeguard
racial and religious minorities.
provided if any province not
willing to join the Union
could have a separate
constitution and form a
separate Union.

51 www.visionias.in ©Vision IAS


3543(Open Test):
Consider the following
statements about the Cripps
Mission 1942:
1. It proposed for the
Constitution-making body
elected by the members of
the Lower House of the
Provincial Legislatures by the
system of proportional
representation.
2. Radical Democratic Party
led by M.N. Roy was the only
party who supported it.
Which of the statements
given above is/are correct?
Explanation:
A new constitution would be
drafted after the war by a
Constituent Assembly
comprising the Indians
elected by the members of
the Lower House of the
Provincial Legislatures by the
system of proportional
representation
55 Art & With reference to B The Nettipakarana (Pali: -pakaraṇa, also called This D F A History of R
Culture Indian history, consider Nettippakarana or just Netti) is a Buddhist scripture. question Ancient and M
the following texts: Nettipakarana is a guide to help those who already seeks to Early Medieval
India: From the
1. Nettipakarana understand the teaching present it to others. The Netti test the
Stone Age to
2. Parishishtaparvan methods were taught by the Buddha's disciple Kaccana fundament
the 12th
3. Avadanashataka (also Katyayana or Kaccayana). al Century-
4. Trishashtilakshana Avadana texts: The emergences of the Avadana may knowledge. Upinder singh-
Mahapurana have been compiled between second Century A.D to the This Page no.29
Which of the above are 11th Century A.D. The earlier product of Avadana texts question
Jaina texts? like Avadana-sataka, Divyavadana etc., which are of can be
(a) 1, 2 and 3 Hinayanic character and the later Avadanas like solved https://tipitaka.
(b) 2 and 4 only Suvarnavarnavadana, Kalpadrumavadanamala, through fandom.com/wi
ki/Nettipakaran
(c) 1, 3 and 4 Vratavadanamala are earmarked as Mahayanic in elimination
a
(d) 2, 3 and 4 character. Avadanas are non canonical Buddhist texts as
52 www.visionias.in ©Vision IAS
about previous lives and karma, example Avadanasha https://shodhga
Divyavadana(4th century) and Avadanashataka (2nd taka are nga.inflibnet.ac.
century) written in sanskrit. It deal wth stories and tales buddhist in/bitstream/10
603/156514/7/
related to Buddha. texts.
07_chapter%20
The Parishishtaparvan also known as the
3.pdf
Sthaviravalicharitra is a 12th-century Sanskrit
mahakavya by Hemachandra which details the histories
of the earliest Jain teachers.
Mahapurana or Trishashthilkshana Mahapurana is a
major Jain text composed largely by "Acharya" Jinasena
during the rule of Rashtrakuta ruler Amoghavarsha and
completed by his pupil Gunabhadra in the 9th century
CE. Mahapurana consists of two parts. The first part is
Ādi purāṇa written by "Acharya" Jinasena. The second
part is Uttarapurana which is the section composed by
Gunabhadra.
Hence option (b) is the correct answer.
56 Art & With reference to C Āryadeva (3rd century), was a Mahayana Buddhist This D F https://www.ox R
Culture Indian history, consider monk, a disciple of Nagarjuna and a Madhyamaka question fordbibliographi R
the following pairs: philosopher. is a central figure in the development of seeks to es.com/view/do
cument/obo-
Historical person early Indian Madhyamaka philosophy. test the
9780195393521
Known as Dignaga was the early 6th century Indian Buddhist fundament
/obo-
1. Aryadeva – Jaina Monk and was also the founder of Indian logic, Hetu al 9780195393521
scholar Vidya. He was born in Simhavakta near Kanchipuram. knowledge. -0065.xml
2.Dignaga – Buddhist Sri Ranganathamuni, popularly known as Sriman
scholar Nathamuni, was a Vaishnava theologian who collected https://www.bri
3. Nathamuni – and compiled the Nalayira Divya Prabandham. tannica.com/bio
Vaishnava scholar Considered the first of Sri Vaishnava āchāryās, graphy/Dignaga
How many pairs given Nathamuni is also the author of Yogarahasya, and
https://www.hi
above are correctly Nyāyatattva.
malayanacadem
matched?
y.com/media/b
(a) None of the pairs ooks/south-
(b) Only one pair india-bhakti-
(c) Only two pairs saints_ei/web/c
(d) All three pairs h01.html
57 Medieval With reference to B Statement 1 is not correct as Changez Khan reached This D F Class 7 NCERT E
History Indian history, consider Indus during the reign of Iltutmish in 1221 AD,chasing question Our Past -II, M
the following the Khwarizmi prince Jalaluddin. seeks to Chapter 3rd,
The Delhi
statements: test the
Sultans -Page
1. The first Mongol fundament
53 www.visionias.in ©Vision IAS
invasion of India Statement 2 is correct :The fourth Mongol invasion al no. 40-41
happened during the during Ala-ud-din Khalji reign took place only after some knowledge.
reign of Jalal-ud-din months of Ala-ud-din’s return from Chittor in 1303 A.D. The Mongols
and Delhi
Khalji. The Mongols numbering 1,20,000 horsemen under the
Sultanate-
2. During the reign of command of Targhi moved so swiftly that provincial
Satish Chandra,
Ala-ud-din Khalji, one governors could not get time to reach Delhi to help the Medieval India
Mongol assault Sultan. NCERT, Class
marched up to Delhi Besides, a large part of the army of Ala-ud-din had left 11. Old NCERT
and besieged the city. for Telingana campaign and the army left at Delhi was page 51-53
3. Muhammad-bin- insufficient and weak after its tough battle at Chittor.
Tughlaq temporarily Therefore, Ala-ud-din was not in a position to face the
lost portions of north- Mongols in an open battle.
west of his kingdom to He retired to the fort of Siri and took up defensive
Mongols. position. The Mongols plundered the environs of Delhi
Which of the and besieged the fort for two months. But as they were
statements given ignorant of the art of siege-warfare, they failed to
above is/are correct? capture the fort and withdrew.
(a) 1 and 2
(b) 2 only Hence statement 3 is not correct:The next major
(c) 1 and 3 Mongol invasion took place after the Khiljis had been
(d) 3 only replaced by the Tughlaq dynasty in the Sultanate. In
1327 the Mongols invaded under Tarmashirin.
Historians have differed regard Muhammad Tughlaq’s
policy. According to one version, he defeated the
Mongols and himself chased them to drive them out.
However according to another version, Muhammad
Tughlaq bribed the Mongols and they went back.
58 Medieval With reference to D The Sayyids were claimed to descent from the Prophet This D F History of R
History Indian history, who of through his daughter Fatima. They Commanded special question Medieval India M
the following were respect in Muslim society. seeks to by V.D Mahajan
page no. 364 (
known as "Kulah- Even the Timur protected the life of Sayyids during his test the
The Delhi
Daran"? invasion in India. Although his policy was one of general fundament
sultanate)
(a) Arab merchants slaughter. al
(b) Qalandars The sayyids put on a pointed cap (kulah) and they were knowledge.
(c) Persian known as ‘Kulah Daran’ during Delhi sultanate.
calligraphists
(d) Sayyids

54 www.visionias.in ©Vision IAS


59 Modern With reference to B In 1602, the United East India Company of the This D F Modern-India- E All India Test Series: Test
History Indian history, consider Netherlands was formed and given permission by the question Bipan-Chandra M 3482:
the following Dutch government to trade in the East Indies including seeks to Old NCERT page Explanation: In the context of
49-54
statements: India. The Dutch founded their first factory in test the European conquest of India,
1. The Dutch Masaulipatam in Andhra Pradesh in 1605. fundament where did the Dutch establish
established their They went on to establish trading centers in different al their first factory?
factories/warehouses parts of India and thus became a threat to the knowledge. (a) Surat
on the east coast on Portuguese. They captured Nagapatam near Madras (b) Masulipatam
lands granted to them (Chennai) from the Portuguese and made it their main (c) Bombay
by Gajapati rulers. stronghold in South India. (d) Chandernagore
2. Alfonso de The Hindu Gajapati rulers (c. 1435 – 1541 CE) ruled over
Albuquerque captured Kalinga (Odisha), large parts of Andhra Pradesh and In 1602, the United East India
Goa from the Bijapur West Bengal, and the eastern and central parts of Company of the Netherlands
Sultanate. Madhya Pradesh and Jharkhand. Gajapati means “king was formed and given
3. The English East with an army of elephants”. In c. 1435 CE, Kapilendra permission by the Dutch
India Company Deva founded the Gajapati dynasty and last ruler of this government to trade in the
established a factory at dynasty was Prataparudra Deva. Hence statement 1 is East Indies including India.
Madras on a plot of not correct. The Dutch founded their first
land leased from a Alfonso de Albuquerque served as viceroy of factory in Masaulipatam in
representative of the Portuguese India from 1509 to 1515. He was the real Andhra Pradesh in 1605.
Vijayangara empire. founder of the Portuguese power in the East, a task he
Which of the completed before his death. He secured for Portugal the 3482-(Explanation)
statements given strategic control of the Indian Ocean by establishing With reference to the British
above are correct? bases overlooking all the entrances to the sea. East India Company, which
(a) 1 and 2 only one of the following events
(b) 2 and 3 only Albuquerque acquired Goa from the Sultan of Bijapur happened the earliest?
(c) 1 and 3 only in 1510 with ease; the principal port of the Sultan of (a) The East India Company
(d) 1, 2 and 3 Bijapur became “the first bit of Indian territory to be got the golden Farman from
under the Europeans since the time of Alexander the the Sultan of Golconda.
Great”. Hence statement 2 is correct. (b) Sir Thomas Roe, the
ambassador of King James I,
British’s East India Company had entered the then arrived at Jahangir’s court.
Madras and entered a treaty with the native chieftains (c) A permanent factory of
of the land to choose the site of their choice to settle East India Company is
and trade. Company established a factory in Madras in established at Surat.
1639 on land leased from representatives of (d) The East India Company
Vijayanagara Empire called the Nayakas. established its first factory in
Originally an uninhabited land, Francis Day and his the south in Masulipatnam.
superior Andrew Cogan of the British East India Explanation:
Company can be considered the founders of Madras, In 1616 the Company
55 www.visionias.in ©Vision IAS
finally zeroed in on this coastal town and began established its first factory in
construction of St George Fort and houses for their the south in Masulipatnam.
residence on 23 April 1640. Hence statement 3 is In 1632 the Company got the
correct. golden Farman from the
Sultan of Golconda ensuring
the safety and prosperity of
their trade.
In 1633 the Company
established its first factory in
east India in Hariharpur,
Balasore (Odisha).
In 1639 the Company gets the
lease of Madras from a local
king.
60 Ancient According to Kautilya's D The Arthashastra states that a man could be a slave This D F http://indiansag R
History Arthashastra, which of either by birth, by voluntarily selling himself, by being question a.com/others/sl R
the following are captured in war, or as a result of a judicial punishment. seeks to avery.html
correct? Slavery was a recognized institution and the legal test the
1. A person could be a relationship between master and slave was clearly fundament
slave as a result of a defined e.g. if a female slave bore her master a son, al
judicial punishment. not only was she legally free but the child was entitled knowledge.
2. If a female slave to legal status of the master's son.
bore her master a son,
she was legally free.
3. If a son born to a
female slave was
fathered by her
master, the son was
entitled to the legal
status of the master's
son.
Which of the
statements given
above are correct?
(a) 1 and 2 only
(b) 2 and 3 only
(c) 1 and 3 only
(d) 1, 2 and 3

56 www.visionias.in ©Vision IAS


61 Economics Consider the following A Tight monetary policy implies the Central Bank (or Recent rate M FA https://thewire. E All India Test Series: Test
statements: authority in charge of Monetary Policy) is seeking to hikes by in/economy/the N 3496
1. Tight monetary reduce the demand for money and limit the pace of Federal -us-feds- In the context of US Federal
aggressive-
policy of US Federal economic expansion. Central banks engage in tight reserve and Reserve, which of the
monetary-
Reserve could lead to monetary policy when an economy is accelerating too announcem following best describes taper
policy-
capital flight. quickly or inflation is rising too fast and usually involves ent of fed tightening-and- tantrum, a term sometimes
2. Capital flight may increasing interest rates. tampering new- seen in the news?
increase the interest leading headwinds-for- (a) It is a situation in
cost of firms with Tight monetary policy of US Federal Reserve means FIIs/FPIs to emerging- monetary policy that sees
existing External hiking the federal funds rate–the rate at which banks sell markets interest rates falling below
Commercial lend to each other–increases borrowing rates and slows financial 0%.
Borrowings (ECBs). lending. Rate increases make borrowing less attractive assets in https://www.bu (b) It refers to a shift in
siness-
3. Devaluation of as interest payments increase. It affects all types of emerging monetary policy that led to
standard.com/a
domestic currency borrowing including personal loans, mortgages, and economies decrease in the purchase of
rticle/finance/e
decreases the currency interest rates on credit cards. Faster Fed rate could like India. cb-inflows-to- the treasury bonds by US Fed.
risk associated with rattle financial markets and tighten financial conditions moderate-on- (c) It is when monetary policy
ECBs. globally especially in emerging market economies like rising-rates- becomes ineffective due to
Which of the India. For instance, an aggressive monetary tightening depreciating- very low interest rates.
statements given would raise US yields and strengthen the US dollar rupee-bank-of- (d) Increase in capital inflows
above are correct? against EM currencies like rupee. As a result, US- based baroda- in emerging markets due to
(a) 1 and 2 only foreign portfolio investors/Foreign institutional 122052400327_ Quantitative Easing by US
1.html
(b) 2 and 3 only investors investing in countries like India would pull Federal Reserve.
(c) 1 and 3 only money out from here and invest in "safe heaven" US Explanation
https://m.rbi.or
(d) 1, 2 and 3 assets (Treasury bonds) and thus leading significant g.in/scripts/Pub In 2013, as the world was
capital flight from India. Hence statement 1 is correct. licationsView.as coming out of a global
px?id=20994 financial crisis, the United
The sudden stops and reversal of capital flows will lead States Federal Reserve said it
to depreciation pressures on emerging market https://www.in would gradually reduce
currencies like rupee. When foreign investors invest in vestopedia.com quantitative easing instituted
equities, bonds and other financial assets in EMEs, they /terms/t/tightm after the Lehman Brothers
onetarypolicy.as
measure financial returns in the US dollar and other bankruptcy in 2008. This
p#:~:text=Tight
foreign currencies. If the EM currency depreciates would involve slowing the
%20monetary%
against the US dollar, it decreases the value of their 20policy%20is% purchase of treasury bonds
investments in dollar terms and, therefore, they may 20an,prices%E2 and hence reducing the
engage in distress sales of funds. This capital flight may %80%94is%20ri money being pumped into the
increase interest cost of firms with a large stock of sing%20too%20 US economy.
foreign currency debt as rising US dollar would increase fast. The announcement led to a
the debt-servicing costs (in local currencies) for firms. sudden sell-off in global
Hence statement 2 is correct. stocks and bonds. This
triggered capital outflows and
57 www.visionias.in ©Vision IAS
Devaluation of domestic currency increases the currency depreciation in many
currency risk associated with ECBs (as mostly foreign emerging market economies
currency denominated). For instance, if at the time of that received large capital
raising loan through ECBs, 1 dollar was equal to Rs 75 inflows. This episode earned
and in future with depreciation/devaluation of domestic the nickname taper tantrum.
currency, 1 dollar becomes Rs 80. In this case, Hence, option (b) is the
companies/firms done borrowing through ECBs would correct answer.
have to pay back more as converting more rupee with Vision IAS Open test (2022) -
their dollar equivalent and in turn increases their 3375
currency risk. Hence statement 3 is not correct. Consider the following
statements with reference to
External Commercial
Borrowings (ECBs):
1. ECBs can be raised in Indian
Rupees (INR).
2. ECB proceeds cannot be
utilised for investment in the
capital market.
3. Increase in ECBs will result
in the decline of forex
reserves.
Which of the statements
given above is/are correct?
(a) 1 only (b) 1 and 2 only
(c) 2 and 3 only (d) 1 and 3
only
62 Geography Consider the following C Today the major tea growing states are Assam, West This M FA https://www.te R
States: Bengal, Tripura, Tamil Nadu, Kerala and Karnataka. question aboard.gov.in/p M
1. Andhra Pradesh Himachal Pradesh and Uttarakhand are also traditional seeks to df/Production_
data_for_2021_
2. Kerala tea growing states, albeit to a less significant extent. test the
and_2021_22_p
3. Himachal Pradesh fundament
df223.pdf
4. Tripura In Himachal Pradesh, tea is grown in the Mandi and al
How many of the Kangra districts over an area of 2,063 hectares. Kangra, knowledge. https://www.in
above are generally known as "the valley of gods," is famous lor its distinct diatea.org/tea_
known as tea- flavoured tea. Tripura is categorized as a traditional tea- growing_region
producing States? growing State with about 54 Tea Estates, 21 Tea s
(a) Only one State processing factories, and more than 2500 small tea
(b) Only two States growers.
(c) Only three States
(d) All four States
58 www.visionias.in ©Vision IAS
63 Economics Consider the following B Credit rating agencies in India are regulated by SEBI. Recently, M FCA https://www.se E Monthly Current Affairs
statements: The SEBI (Credit Rating Agencies) Regulations, 1999 SAT sets bi.gov.in/sebi_d N Magazine March 2022 page
1. In India, credit rating govern the credit rating agencies and provide for aside Rs 1- ata/faqfiles/oct- no 27
2021/16349026
agencies are regulated eligibility criteria for registration of credit rating cr penalty
64371.pdf
by Reserve Bank of agencies, monitoring and review of ratings, imposed on Test 2880 (2019 Abhyaas)
India. requirements for a proper rating process, avoidance of Brickwork https://www.bu Consider the following
2. The rating agency conflict of interest and inspection of rating agencies by Ratings by siness- statements with reference to
popularly known as SEBI, amongst other things. Hence statement 1 is Sebi standard.com/a credit rating agencies in India:
ICRA is a public limited correct. rticle/companie 1. Credit rating agencies
company. s/sat-sets-aside- primarily rate the lending
3. Brickwork Ratings is ICRA Limited (formerly Investment Information and rs-1-cr-penalty- abilities of investors.
an Indian credit rating Credit Rating Agency of India Limited) was set up in imposed-on- 2. In India, credit rating
brickwork-
agency. 1991 by leading financial/investment institutions, agencies have to be
ratings-by-sebi-
Which of the commercial banks and financial services companies as registered with the Securities
122032801153_
statements given an independent and professional investment 1.html and Exchange Board of India.
above are correct. Information and Credit Rating Agency. Today, ICRA and 3. CRISIL is the only credit
(a) 1 and 2 only its subsidiaries together form the ICRA Group of https://www.bri rating agency of India.
(b) 2 and 3 only Companies (Group ICRA). ICRA is a Public Limited ckworkratings.c Which of the statements
(c) 1 and 3 only Company, with its shares listed on the Bombay Stock om/Aboutus.as given above is/are correct?
(d) 1, 2 and 3 Exchange and the National Stock Exchange. Hence px (a) 1 only
statement 2 is correct. (b) 2 only
https://www.icr
(c) 2 and 3 only
a.in/Home/Profi
Brickwork Ratings (BWR), is a SEBI registered Indian (d) 1, 2 and 3
le#:~:text=ICRA
Credit Rating Agency founded by bankers, credit rating %20is%20a%20
professionals, former regulators as well as professors, is Public%20Limit
committed to promoting Financial Literacy. Brickwork ed,and%20the%
Ratings has Canara Bank, a leading Public Sector Bank, 20National%20S
as its promoter and strategic partner. Hence statement tock%20Exchan
3 is correct. ge.

64 Economics With reference to the B Central Government notified the amendment to the Restructuri E FCA https://banksbo E All India Test Series: Test
'Banks Board Bureau Nationalised Banks (Management and Miscellaneous ng of Banks ardbureau.org.i N 3504
(BBB)', which of the Provisions) Scheme, 1980 providing the legal framework Board n/bureau- Consider the following
profile/
following statements for composition and functions of the Banks Board Bureau was statements regarding Bank
are correct? Bureau on March 23, 2016. The Bureau accordingly in news as Board Bureau (BBB):
https://econom
1. The Governor of RBI started functioning from April 01, 2016 as an its ictimes.indiatim 1. It will advise the
is the Chairman of BBB. autonomous recommendatory body. members es.com/news/e government on top-level
2. BBB recommends extended conomy/policy/ appointments at Public Sector
for the selection of The board comprises the Chairman, three ex-officio two-year government-to- Banks (PSBs) and Financial
heads of Public Sector members i.e. Secretary, Department of Public term soon-approve- Institutions (FIs).
recast-of-banks-
59 www.visionias.in ©Vision IAS
Banks. Enterprises, Secretary of the Department of Financial concluded board- 2. It has been set up as a
3. BBB helps the Public Services and Deputy Governor of the Reserve Bank of on April 10. bureau/articles three member body.
Sector Banks in India and three expert members. The Governor of RBI is how/91574167. 3. Only a retired governor of
cms
developing strategies not its chairman. Hence statement 1 is not correct. RBI can become the Chairman
and capital raising The Banks Board Bureau is mandated to select and of the Bureau.
https://econom
plans. appointment Board members for various financial ictimes.indiatim Which of the statements
Select the correct institutions in public sector like Public sector banks, es.com/news/c given above is/are correct?
answer using the code insurance companies. It is also required to undertake ompany/corpor (a) 1 and 2 only
given below: activities in the sphere of governance in these ate- (b) 1 only
(a) 1 and 2 only institutions. The functions of the Bureau outlined in the trends/banks- (c) 2 and 3 only
(b) 2 and 3 only Section 7(C) of the Scheme and in subsequent board-bureau- (d) 1, 2 and 3
(c) 1 and 3 only amendments as per orders of Appointment Committee recommends-
names-for-
(d) 1, 2 and 3 of the Cabinet are :-
chiefs-of-psu-
a) To recommend the selection and appointment of
banks/articlesh
Board of Directors in Mandated Institutions (Whole ow/90275752.c
Time Directors and Chairman). Hence statement 2 is ms
correct.
b) To advise the Central Government on matters
relating to appointments, confirmation or extension of
tenure and termination of services of the Directors of
mandated institutions.
c) To advise the Central Government on the desired
management structure of mandated institutions, at the
level of Board of Directors and senior management.
d) To advise the Central Government on a suitable
performance appraisal system for mandated
institutions.
e) To build a data bank containing data relating to the
performance of mandated institutions and its officers.
f) To advise the Central Government on the formulation
and enforcement of a code of conduct and ethics for
managerial personnel in mandated institutions.
g) To advise the Central Government on evolving
suitable training and development programs for
managerial personnel in mandated institutions.
h) To help the banks in terms of developing business
strategies and capital raising plan. Hence statement 3
is correct.
i) Any other work assigned by the Government in
consultation with Reserve Bank of India.
60 www.visionias.in ©Vision IAS
65 Economics With reference to C A convertible bond is a fixed-income corporate debt Bonds like D FA https://www.in R
Convertible Bonds, security that yields interest payments, but can be corporate vestopedia.com M
consider the following converted into a predetermined number of common bonds, /terms/c/conve
rtiblebond.asp#
statements: stock or equity shares. It offers investors a type of convertible
:~:text=Howeve
1. As there is an option hybrid security that has features of a bond, such as bonds aim
r%2C%20conver
to exchange the bond interest payments, while also having the option to own to test tible%20bonds
for equity, Convertible the underlying stock. fundament %20tend%20to,
Bonds pay a lower rate al all%20companie
of interest. Issuing convertible bonds can help companies minimize understandi s%20offer%20c
2. The option to negative investor sentiment that would surround equity ng. onvertible%20b
convert to equity issuance. Further, issuing convertible bonds can also onds.
affords the bondholder help provide investors with some security in the event
https://www.ec
a degree of indexation of default. A convertible bond protects investors'
onomist.com/fi
to rising consumer principal on the downside, but allows them to
nance-and-
prices. participate in the upside should the underlying company economics/202
Which of the succeed. However, convertible bonds tend to offer a 1/07/10/why-
statements given lower coupon rate or rate of return in exchange for the convertible-
above is/are correct? value of the option to convert the bond into common bonds-are-the-
(a) 1only stock. Companies benefit since they can issue debt at asset-class-for-
(b) 2 only lower interest rates than with traditional bond the-times
(c) Both 1 and 2 offerings. However, not all companies offer convertible
(d) Neither 1 nor 2 bonds. Hence statement 1 is correct.

The option to convert to equity affords the bondholder


a degree of indexation to rising consumer prices. As
indexation will ensure that prices are adjusted with
inflation over a period of time. With the help of
indexation, bondholders will be able to lower their long-
term capital gains (as their investment will be adjusted
with inflation) even when converting bonds into equity,
which brings down their taxable income. Hence
statement 2 is correct.
66 Internatio Consider the following: D The Asian Infrastructure Investment Bank (AIIB) is a The E CA https://www.liv E PT 365 2022 International
nal 1. Asian Infrastructure multilateral development bank with a mission to organisatio emint.com/new N Relations (pg no 41, 45, 50)
Relations Investment Bank improve social and economic outcomes in Asia and ns/groupin s/india/india- VISIONIAS All India Test
may-not-alter-
2. Missile Technology beyond. Headquartered in Beijing, It commenced gs Series: Test 3493
rules-of-
Control Regime operations in January 2016. India is a founding member mentioned Which of the following
engagement-
3. Shanghai of AIIB with the second-highest voting share after in the with-aiib- statements regarding the
Cooperation China. question - 1159598548115 Asian Infrastructure
Organisation The Missile Technology Control Regime (MTCR) is an SCO, MTCR, 7.html Investment Bank (AIIB) are
61 www.visionias.in ©Vision IAS
India is a member of informal political understanding among states that seek AIIB https://www.th correct?
which of the above? to limit the proliferation of missiles and missile frequently ehindubusinessl 1. AIIB is the second largest
(a) 1 and 2 only technology. The regime was formed in 1987. India appeared in ine.com/news/i multilateral development
ndia-in-mtcr-
(b) 3 only became an official member of the Missile Technology news. bank by membership after the
club-can-now-
(c) 2 and 3 only Control Regime (MTCR) in 2018. World Bank.
export-
(d) 1, 2 and 3 The Shanghai Cooperation Organisation (SCO) is a arms/article641 2. China is the largest
transcontinental political, economic, security, and 24008.ece shareholder with 26.61%
military alliance founded in 2001. On 9 June 2017, at the https://mea.gov voting shares in the bank
historic summit in Astana, India and Pakistan officially .in/Portal/Forei followed by Japan.
joined SCO as full-fledged members. The SCO currently gnRelation/SCO 3. AIIB offers only sovereign
comprises Nine Member States (China, India, _MULTI_Brief_f loans to its member
Kazakhstan, Kyrgyzstan, Russia, Pakistan, Tajikistan, eb_2020.pdf countries.
Uzbekistan and Iran). Select the correct answer
Hence option (d) is the correct answer. using the code given below.
Explanation: China is the
largest shareholder with
26.61 % voting shares in the
bank followed by India (7.6%),
Russia (6.01%), and Germany
(4.2 %).
Test 3488:
Which of the following
statements with respect to
Shanghai Cooperation
Organization (SCO) are
correct?
1. It originated from Shanghai
five grouping of 1996.
2. Afghanistan is the latest
country to become the
permanent member of the
grouping.
3. The objective of SCO is to
fight terrorism, separatism
and extremism, bolster trade
and economic growth, and
advance people-to-people
ties.
Select the correct answer
using the code given below.
62 www.visionias.in ©Vision IAS
Explanation: The SCO
currently comprises eight
Member States (China, India,
Kazakhstan, Kyrgyzstan,
Russia, Pakistan, Tajikistan
and Uzbekistan).
67 Internatio Consider the following C Vietnam is a one-party communist state, not a multi party Vietnam M CAA https://www.bb E Monthly Current Affairs
nal statements: parliamentary democracy. Hence statement 2 is not was in c.com/news/wo N September 2021 page 31, 34
Relations 1. Vietnam has been correct. news as it rld-asia-pacific-
Vietnam’s open economic policy of recent years integrating 16567315
one of the fastest has taken
into global supply chains has made the growth success https://www.ec
growing economies in decisive
story possible in recent years. Vietnam's export-led onomist.com/fi
the world in the recent steps to nance-and-
years. growth strategy and global integration are among the key limit both economics/202
2. Vietnam is led by a factors behind the country's remarkable achievements in the health 1/08/30/the-
growth and poverty. Between 2002 and 2021, GDP per
multi-party political and economy-that-
capita increased 3.6 times, reaching almost US$3,700.
system. economic covid-19-could-
Poverty rates declined sharply from over 32 percent in not-stop
3. Vietnam's economic 2011 to below 2 percent today. Thanks to its solid
fallout.
growth is linked to its https://www.w
foundations, the economy has proven resilient through orldbank.org/en
integration with global different crises, the latest being COVID-19. Vietnam was /country/vietna
supply chains and one of only a few countries to post GDP growth in 2020 m/overview#1
focus on exports. when the pandemic hit. Vietnam is projected to be the
4. For a long time fastest-growing internet economy in Southeast Asia in the
Vietnam's low labour next 10 years. Hence statements 1 and 3 are correct.
costs and stable Thanks to an abundance of low-wage labour, Vietnam’s
exchange rates have manufacturing sector grew at a compound annual growth
attracted global in the last decade. As the rest of East Asia developed and
manufacturers. wages there rose, global manufacturers were lured by
5. Vietnam has the Vietnam's low labour costs and stable exchange rate.
most productive e- Hence statement 4 is correct.
service sector in the According to Asian Development Bank Report, e-services
Indo-Pacific region. including digital financial services are at a very nascent
Which of the stage in Vietnam. Vietnamese fintech companies raised a
total of US$375 million in the first nine months of 2021,
statements given
putting the country at the 3rd position in fintech
above are correct?
investment amount across ASEAN, behind only Singapore
(a) 2 and 4
and Indonesia. Countries like Philippines, Singapore and
(b) 3 and 5
Indonesia are the forefront in e-commerce productivity.
(c) 1, 3 and 4 Hence statement 5 is not correct.
(d) 1 and 2

63 www.visionias.in ©Vision IAS


68 Economics In India, which one of D The Reserve Bank of India (RBI) is vested with the This E F https://www.rbi E PT 365 2022 Economy
the following is responsibility of conducting monetary policy. This question .org.in/scripts/F N All India Test Series 3490
responsible for responsibility is explicitly mandated under the Reserve seeks to S_Overview.asp (AITS 2022)
x?fn=2752
maintaining price Bank of India Act, 1934. The primary objective of test the In the explanation to the
stability by controlling monetary policy is to maintain price stability while fundament following question.
https://www.th
inflation? keeping in mind the objective of growth. Price stability al ehindubusinessl
(a) Department of is a necessary precondition to sustainable growth. knowledge, ine.com/opinio Recently Consumer Price
Consumer Affairs as mainting n/inflation- Index (CPI) has been declining
(b) Expenditure The MPC (Monetary Policy Committee) determines the price targeting-has- but the Wholesale Price Index
Management policy interest rate required to achieve the inflation stability by been-a- (WPI) has been rising, with
Commission target. The Reserve Bank’s Monetary Policy Department controlling success/article3 reference to inflation and its
(c) Financial Stability (MPD) assists the MPC in formulating the monetary inflation is 4208320.ece targeting in India, consider
and Development policy. Views of key stakeholders in the economy, and prerogative the following statements:
https://www.hi
Council analytical work of the Reserve Bank contribute to the of RBI. 1. Increase in inflation
ndustantimes.c
(d) Reserve Bank of process for arriving at the decision on the policy repo om/ht- indicates a decrease in
India rate. The Financial Markets Operations Department insight/econom purchasing power of a given
(FMOD) of RBI operationalises the monetary policy, y/inflation- currency over time.
mainly through day-to-day liquidity management targeting-and- 2. The Reserve Bank of India
operations. The Financial Markets Committee (FMC) the-indian- (RBI) uses Consumer Price
meets daily to review the liquidity conditions so as to economy- Index (CPI) for inflation
ensure that the operating target of the weighted 1016268509711 targeting in India.
01.html
average call money rate (WACR) is aligned with the repo Which of the statements
rate. given above is/are correct?
There are several direct and indirect instruments that (a) 1 only
are used by RBI for implementing monetary policy for (b) 2 only
price stability by keeping check on inflation such as (c) Both 1 and 2
Repo rate, Reverse repo, Open market operations, (d) Neither 1 and 2
Liquidity adjustment facility, etc. For example, RBI does
it by increasing the repo rate in the economy, if inflation
is high. Increase in interest rate is believed to bring
down the inflation level in the economy by curtailing
the amount of economic activity whereas decreasing
the interest rate increases the inflation level of the
economy by increasing the economic activity. Hence
option (d) is the correct answer.
69 Current With reference to Non- A Non-fungible tokens, commonly known as NFTs, are Recently in M FCA https://indianex E Monthly Current Affairs April
Affairs Fungible Tokens unique cryptographic tokens that exist on a blockchain Budget press.com/articl N 2021, News Today 25th
(NFTs), consider the and cannot be replicated—having a unique 2022-23, e/explained/eve January 2022
ryday-
following statements: identification code and metadata. Hence statement 2 is Governmen All India Test Series: Test
explainers/what
1. They enable the correct. t included 3493
-are-non-
64 www.visionias.in ©Vision IAS
digital representation Anything that can be converted into a digital form can capital fungible-tokens- With reference to Non-
of physical assets. be an NFT. Everything from one’s drawings, photos, gains from nft-7783662/ fungible tokens (NFTs),
2. They are unique videos, GIF, music, in-game items, selfies, and even a virtual consider the following
https://www.in
cryptographic tokens tweet can be turned into an NFT. Hence statement 1 is digital statements:
vestopedia.com
that exist on a correct. assets like 1. NFT is a digital asset that
/non-fungible-
blockchain. NFTs and cryptocurrencies are very different from each cryptocurre tokens-nft- exists on a blockchain with a
3. They can be traded other. Cryptocurrency is a currency and is fungible, ncies and 5115211#:~:tex unique digital signature.
or exchanged at meaning that it is interchangeable. For instance, if you non- t=NFTs%20are% 2. These can be bought with
equivalency and hold one crypto token, say one Ethereum, the next fungible 20unique%20cr cryptocurrencies or cash.
therefore can be used Ethereum that you hold will also be of the same value. tokens yptographic%20 3. All kinds of digital objects
as a medium of But NFTs are non-fungible, that means the value of one (NFTs) with tokens,reducing including texts and tweets on
commercial NFT is not equal to another. Every art is different from 30% %20the%20pro twitter can be turned into an
bability%20of%
transactions. other, making it non fungible, and unique. Thus, unlike income tax. NFT.
20fraud.
Which of the cryptocurrencies, they cannot be traded or exchanged Which of the statements
statements given at equivalency and therefore cannot be used as a https://www.liv given above is/are correct?
above are correct? medium of commercial transactions. Hence statement 3 emint.com/mon (a) 1 and 3 only
(a) 1 and 2 only is not correct. ey/personal- (b) 2 only
(b) 2 and 3 only finance/tax- (c) 1 and 2 only
(c) 1 and 3 only implications-of- (d) 1, 2 and 3
(d) 1, 2 and 3 minting-nfts-
1165010880981
3.html
70 Geography Consider the following C Pair 1 is not correctly matched: Raja Lakhamagouda This D F Standard E Vision IAS Open Test
pairs: dam, also known as Hidkal dam, is a dam constructed question Reference Atlas M 3375(2022)
Reservoirs States across the Ghataprabha River in the Krishna River basin. seeks to
1. Ghataprabha – It is situated at Hidkal village in Hukkeri Taluk of test the Recently, Project Lion was
Telangana Belagavi district in North Karnataka. fundament announced for the
2.Gandhi Sagar – Pair 2 is correctly matched: Gandhisagar Dam is located al conservation of Asiatic Lion.
Madhya Pradesh in the Mandsaur, districts of the state of Madhya knowledge. Which of the following sites
3. Indira Sagar – Pradesh. The Dam is constructed on the Chambal River. has been identified under the
Andhra Pradesh Pair 3 is not correctly matched: The Indira Sagar Dam is plan?
4. Maithon – the largest dam in India, in terms of volume of water 1. Gandhi Sagar Wildlife
Chhattisgarh stored in the reservoir. It is located on the Narmada Sanctuary
How many pairs given River at the town of Narmada Nagar, Punasa in the 2. Sitamata Wildlife Sanctuary
above are not correctly Khandwa district of Madhya Pradesh in India. 3. Mukundra Hills Tiger
matched? Pair 4 is not correctly matched: Maithon is located on Reserve
(a) Only one pair the banks of river Barakar. The Maithon Dam is located 4. Narayan Sarovar Sanctuary
(b) Only two pairs about 48 kms from the Coal City of Dhanbad, Jharkhand. Select the correct answer
(c) Only three pairs using the code given below.
(d) All four pairs

65 www.visionias.in ©Vision IAS


71 Economics In India, which one of C Labour Bureau has been engaged in collection, Labour M F https://labour.g R
the following compiles compilation, analysis and dissemination of labour Bureau ov.in/sites/defa M
information on statistics at All India and State level on different facets releases ult/files/Annual
_Report_20202
industrial disputes, of labour since its inception in the year 1920. It releases annual
1_English.pdf
closures, annual publication titled, ‘Industrial Disputes, publication
retrenchments and lay- Closures, Retrenchments and Lay-Offs in industries in titled, http://www.lab
offs in factories India’. It provides statistics on work-stoppages, ‘Industrial ourbureaunew.
employing workers? closures, retrenchments and lockouts which is useful in Disputes, gov.in/UserCont
(a) Central Statistics making policies for maintaining cordial and Closures, ent/ID_Review_
Office harmonious relations between the management and Retrenchm 2014.pdf?pr_id
(b) Department for the worker. This report is also aimed at meeting the ents and =FfR6mn4XUVE
Promotion of Industry demand for historical data and information on industrial Lay-Offs in %3D
and Internal Trade conflicts and their causes by the planners, policy makers industries
https://econom
(c) Labour Bureau and decision makers in industry and government. Hence in India’.
ictimes.indiatim
(d) National Technical option (c) is the correct answer. es.com/news/e
Manpower conomy/indicat
Information System ors/3557-
workers-
affected-by-lay-
offs-across-
india-between-
january-and-
june-labour-
ministry-
data/articlesho
w/78232681.cm
s?from=mdr
72 Polity & In India, what is the A Coal controller organization (earlier Coal Recently, D FA http://www.coa E
Governanc role of the Coal Commissioner), established in 1916, is one of the oldest governmen lcontroller.gov.i N
e Controller's offices in Indian Coal sector. Main aim behind setting up t n/pages/display
/5-
Organization (CCO)? this office was to have Government control to inaugurate
functionsrespon
1. CCO is the major adequately meet the coal requirement during First d an
sibilities
source of Coal World War. Acute scarcity of coal necessitated extended
Statistics in promulgation of Colliery Control Order, 1944 for office of https://energy.
Government of India. effective control on production, distribution and pricing Coal economictimes.
2. It monitors progress of coal. Subsequently, it was revised by a more Controller's indiatimes.com/
of development of comprehensive order in 1945. Later in 1996, distribution Organizatio news/coal/exte
Captive Coal/Lignite and pricing of coal was deregulated. Thereafter, Colliery n, a nded-office-of-
blocks. Control Order, 2000 superseded the previous order. subordinate coal-controllers-
organization-
3. It hears any Finally, the Colliery Control Rules, 2004, was published office
inaugurated/89
objection to the by Government of India in August, 2004. under the
66 www.visionias.in ©Vision IAS
Government's Functions of Coal Controller’s Organization Ministry of 128579
notification relating to (i) Under Colliery Control Rules, 2004: Coal.
acquisition of coal- (a) To lay down procedure and standard for sampling of
bearing areas. coal.
4. It ensures that coal (b) Inspection of collieries so as to ensure the
mining companies correctness of the class, grade or size of coal.
deliver the coal to end (c) To issue directives for the purpose of declaration and
users in the prescribed maintenance of grades of coal of a seam mined in a
time. colliery.
Select the correct (d) To act as the appellate authority in case of dispute
answer using the code between consumers and owner arising out of
given below: declaration of grade and size of coal.
(a) 1, 2 and 3 (e) To regulate disposal of stock of coal or the expected
(b) 3 and 4 only output of coal in the colliery.
(c) 1 and 2 only (f) Quality surveillance with respect to maintenance of
(d) 1, 2 and 4 grade, loading of coal in wagons/ trucks according to
laid down procedures regarding grades and sizes.
(g) To grant opening / re-opening permission of coal
mine, seam or a section of seam or to subdivide a mine.
(ii) Under Collection of Statistics Act, 2008:
(a) Coal Controller has been made the statistical
authority with respect to coal and lignite statistics.
Hence statement 1 is correct.
(b) Entrusted the responsibility of carrying out Annual
Coal & Lignite survey and publishing of Provisional Coal
Statistics and Coal Directory of India.
(c) Submission of monthly coal data to different
ministries of central and state Govt., national and
international organization.
(d) Collection of Statistics relating to coal washeries.
(iii) Under Coal Bearing Area (Acquisition and
Development) Act, 1957:
Coal Controller is the competent authority under this
act to hear any objection to the Central Government’s
Notification relating to acquisition of coal bearing land
and to furnish his reports to Central Govt. Hence
statement 3 is correct.
(iv) Under Coal Mines (Conservation & Development)
Act, 1974 and Coal Mines (Conservation and
Development) Amendment Rules, 2011:
67 www.visionias.in ©Vision IAS
(a) Assessment and collection of excise duty levied on all
raw coal raised and dispatched.
(b) Providing financial support to the coal operators for-
-Ensuring the conservation of coal resources: Stowing in
UG mines.

-Undertaking the development of coal mines in a


scientific manner.

-Undertaking research in relation to conservation of


coal, development of coal mines and utilization of coal.

-Protective works including blanketing with


incombustible material, N2 & CO2 flushing, filling up of
subsided areas, cutting of trenches etc.

-Infrastructure development such as Road / Rail


infrastructures in coalfields.

(v) Commissioner of Payments

(a) Coal Controller functions as the Commissioner of


Payment to settle the claim cases of colliery owners of
pre-nationalisation period under the Coking Coal Mines
(Nationalisation) act, 1972, the Non-coking Coal Mines
(Nationalisation) Act, 1973 and also for the Schedule-I
Coal Mines as per the Coal Mines (Special Provision) Act,
2015

(vi) Others:
Monitoring of Coal & Lignite blocks. Hence statement 2
is correct.
Monitoring of opening of Escrow account as per
approved MCP.

In above roles and responsibility, it is not explicitly


mentioned that CCO ensures that coal mining
companies deliver the coal to end users in the
prescribed time. Hence option (a) is the correct answer.

68 www.visionias.in ©Vision IAS


73 Polity & If a particular area is A The Indian Constitution provides autonomy to tribal This M F Laxmikant:India E
Governanc brought under the areas in matters of governance under the Fifth and Sixth question n Polity M
e Fifth Schedule of the Schedules, which is further fortified by the Samatha v. seeks to Chapter: Fifth
and Sixth
Constitution of India, State of Andhra Pradesh & Ors (1997) judgment where test the
Schedule
which one of the the Supreme Court declared that the transfer of tribal fundament
following statements land to private parties for mining was null and void al
best reflects the under the Fifth Schedule. knowledge.
consequence of it? Special Provisions for Fifth Schedule Areas:
(a) This would prevent -The Union Government shall have executive powers to
the transfer of land of give directions to the States as to the administration of
tribal people to non- the Scheduled Areas.
tribal people. - Provision for establishment of a Tribes Advisory
(b) This would create a Council (TAC) in any State having Scheduled Areas.
local self-governing - The Governor may make regulations:
body in that area. # prohibit or restrict the transfer of land by or among
(c) This would convert members of the Scheduled tribes in such area;
that area into a Union #regulate the allotment of land to members of the STs
Territory. in such area;
(d) The State having # regulate the carrying on of business as money-lender
such areas would be by persons who lend money to members of the STs in
declared a special such area.
Category State. - At present, 10 States namely Andhra Pradesh,
Chhattisgarh, Gujarat, Himachal Pradesh, Jharkhand,
Madhya Pradesh, Maharashtra, Odisha, Rajasthan and
Telangana have Fifth Scheduled Areas.
74 Current Consider the following B Statement 1 is not correct: The India Sanitation Recently, D CA https://www.ne E PT 365 2022 Economy page
Affairs statements: Coalition was launched on June 25, 2015, at FICCI, New India windianexpress. N no 98
1. The India Sanitation Delhi. ISC is a multi-stakeholder platform that brings Sanitation com/cities/delhi
/2022/feb/23/c
Coalition is a platform together the private sector, government, financial Coalition
ontest-for-
to promote sustainable institutions, civil society groups, media, donors/bi- (ISC) has
promoting-
sanitation and is lateral/multilateral, experts etc. to work in the partnered hand-hygiene-
funded by the sanitation space to drive sustainable sanitation through with 2422771.html
Government of India a partnership model. UNICEF
and the World Health India to https://pib.gov.i
Organization. Statement 2 is correct: Established in 1976, the organise a n/PressRelease
2. The National National Institute of Urban Affairs (NIUA), is a premier national Page.aspx?PRID
Institute of Urban Institute and appointed as an apex body to support and competitio =1800831
Affairs is an apex body guide the Government of India in its urban development n for
http://urbanriv
of the Minister of plans. identifying
ers.niua.org/no
Housing and Urban and de/46
69 www.visionias.in ©Vision IAS
Affairs in Government promoting
of India and provides ‘Product
innovative solutions to Innovations
address the challenges & Business
of Urban India. Solutions
Which of the on
statements given designing,
above is/are correct? setting up,
(a) 1 only and
(b) 2 only operating
(c) Both 1 and 2 Hand
(d) Neither 1 nor 2 Washing
with Soap
Stations’ in
the wake of
increasing
pandemic
resilience.

Recently,
World
Economic
Forum and
the
National
Institute of
Urban
Affairs sign
a MoU to
collaborate
on a jointly
designed
‘Sustainabl
e Cities
India
program’.

70 www.visionias.in ©Vision IAS


75 Environme Which one of the C Central Ground Water Authority (CGWA) was This M FA http://cgwb.gov R Covered in Current Affairs
nt following has been constituted under sub-section (3) of Section 3 of the question .in/aboutcgwa.h M Magazine February 2022
constituted under the Environment (Protection) Act, 1986 for the purposes of seeks to tml
Environment regulation and control of ground water development test the PT 365 2022 365 Updated
(Protection) Act, 1986? and management in the country. fundament page no 67
(a) Central Water al
Commission knowledge. MCAR February 2022: Test
(b) Central Ground 3987
Water Board
(c) Central Ground With reference to guidelines
Water Authority issued by the Central Ground
(d) National Water Water Authority (CGWA) to
Development Agency regulate the extraction of
groundwater, consider the
following statements:
1. Annual water audits are
compulsory for industrial
users.
2. It is mandatory for mining
projects to get a No objection
certificate (NoC) for
withdrawal of groundwater.
3. Domestic consumers and
farmers do not need NoC for
drawing water up to a limit.
Which of the statements
given above is/are correct?
(a) 1 only (b) 3 only
(c) 1 and 3 only (d) 1, 2 and 3
76 Current With reference to the A The United Nations Credentials Committee is a United M CA https://www.th E
Affairs "United Nations committee of the United Nations General Assembly Nations ehindubusinessl N
Credentials whose main purpose is to report to the Assembly Credentials ine.com/news/
world/un-
Committee", consider regarding the credentials of the body's representatives. Committee
committee-to-
the following UN Credentials Committee is appointed at the was in
review-taliban-
statements: beginning of each regular session of the General news credential-
1. It is a committee set Assembly. It consists of nine members, who are recently as issue-to-meet-
up by the UN Security appointed by the General Assembly on the proposal of the in-
Council and works the President. The Assembly meets in regular sessions Committee november/articl
under its supervision. from September to December each year, and had to e36788305.ece
2. It traditionally meets thereafter as required. It discusses specific issues review https://www.un

71 www.visionias.in ©Vision IAS


in March, June and through dedicated agenda items or sub-items, which credentials .org/en/ga/cred
September every year. lead to the adoption of resolutions. Hence statements 1 of entials/credenti
3. It assesses the and 2 are not correct. Afghanistan als.shtml
credentials of all UN ’s Taliban
members before The credentials of representatives and the names of and
submitting a report to members of the delegation of each Member State are Myanmar’s
the General Assembly submitted to the Secretary-General and are issued junta
for approval. either by the Head of the State or Government or by the governmen
Which of the Minister for Foreign Affairs. The Committee is t to
statements given mandated to examine the credentials of represent
above is/are correct? representatives of Member States and to report to the their
(a) 3 only (b) 1 and 3 General Assembly thereon. Hence statement 3 is countries at
(c) 2 and 3 (d) 1 and 2 correct. the UN.
77 Environme Which one of the A The International Code for Ships Operating in Polar India M CAA https://www.im R
nt following statements Waters or Polar Code is an international regime recently o.org/en/OurW M
best describes the adopted by the International Maritime Organization in unveiled its ork/Safety/Page
s/polar-
'Polar Code'? 2014. The Code sets out regulations for shipping in the Arctic
code.aspx
(a) It is the polar regions, principally relating to ice navigation and policy.
international code of ship design. https://indianex
safety for ships press.com/articl
operating in polar e/india/india-
waters. arctic-policy-
(b) It is the agreement climate-change-
of the countries mineral-
around the North Pole resources-
7825518/
regarding the
demarcation of their
territories in the polar
region.
(c) It is a set of norms
to be followed by the
countries whose
scientists undertake
research studies in the
North Pole and South
Pole.
(d) It is a trade and
security agreement of
the member countries
of the Arctic Council.
72 www.visionias.in ©Vision IAS
78 Internatio With reference to the D Observer status is a privilege granted by some Recently, E CA https://www.th E Monthly Current Affairs
nal United Nations General organizations to non-members to give them an ability to UNGA ehindu.com/ne N October 2021 (Pg. 66),
Relations Assembly, consider the participate in the organization's activities. The United granted ws/national/ind Monthly Current Affairs
ia-introduces-
following statements: Nations General Assembly grants Permanent Observer observer December 2021 (Pg. 27)
draft-
1. The UN General Status to non-member states, international status to
resolution-in-
Assembly can grant organisations and other entities. Internation unga-for-
observe status to the According to UN website, "observer status would be al Solar granting-
non-member States. confined to states and intergovernmental Alliance. observer-status-
2. Inter-governmental organisations whose activities cover matters of interest for-
organisations can seek to the Assembly”. The Sixth Committee of the General international-
observer status in the Assembly considers all applications for observer status solar-
UN General Assembly. before they are considered in the plenary session. alliance/article3
7016157.ece
3. Permanent Hence statements 1 and 2 are correct.
https://ask.un.o
Observers in the UN The Permanent Observers may participate in the
rg/faq/14519
General Assembly can sessions and workings of the General Assembly and
maintain mission at the maintain missions at the U.N. headquarters. Hence
UN headquarters statement 3 is correct.
Which of the In December, 2021, United Nations General Assembly
statements given granted Observer status to International Solar Alliance
above are correct?
(a) 1 and 2 only
(b) 2 and 3 only
(c) 1 and 3 only
(d) 1, 2 and 3
79 Polity & With reference to the D The genesis of the Tea Board India dates back to 1903 The 70- M FCA https://econom E
Governanc "Tea Board" in India, when the Indian Tea Cess Bill was passed. The present year-old ictimes.indiatim N
e consider the following Tea Board set up under section 4 of the Tea Act 1953 Tea Act is es.com/news/e
conomy/policy/
statements: was constituted on 1st April 1954. It has succeeded the headed for
commerce-
1. The Tea Board is a Central Tea Board and the Indian Tea Licencing an
ministry-
statutory body. Committee which functioned respectively under the amendmen proposes-
2. It is a regulatory Central Tea Board Act,1949 and the Indian Tea Control t aimed at repeal-of-
body attached to the Act, 1938 which were repealed. The activities of the two overhauling decades-old-
Minister of Agriculture previous bodies had been confined largely to regulation the Tea tea-coffee-
and Farmers Welfare. of tea cultivation and export of tea as required by the Board of spices-rubber-
3. The Tea Board's International Tea Agreement then in force, and India amid acts/articlesho
Head Office is situated promotion of tea Consumption. falling tea w/89217798.cm
s
in Bengaluru. Statement 1 is correct and statement 2 is not correct: exports
https://www.liv
4. The Board has The present Tea Board is functioning as a statutory worsened
emint.com/eco
overseas offices at body of the Central Government under the Ministry of by the nomy/tea-
Dubai and Moscow. Commerce. The Board is constituted of 31 members Ukraine board-of-india-
73 www.visionias.in ©Vision IAS
Which of the (including Chairman) drawn from Members of crisis. set-for-
statements given Parliament, tea producers, tea traders, tea brokers, overhaul-as-
above are correct? consumers, and representatives of Governments from exports-falter-
1164780287077
(a) 1 and 3 the principal tea producing states, and trade unions.
0.html
(b) 2 and 4 The Board is reconstituted every three years.
(c) 3 and 4 Statement 3 is not correct: Its head office is located at
(d) 1 and 4 Kolkata (West Bengal).
Statement 4 is correct: Currently Tea Board has two
overseas offices located at Dubai, and Moscow. All
these foreign offices of the Board are designed to
undertake the various promotional measures to boost
up export of Indian tea. These offices also act as a
liaison office for interaction between importers of
Indian tea of the respective regions as well as Indian
Exporters.

74 www.visionias.in ©Vision IAS


80 Current Which one of the A Greenwashing is when a company purports to be The term E FCA https://www.th E Monthly Current Affairs
Affairs following best environmentally conscious for marketing purposes but 'greenwashi ehindu.com/sci- N February 2022 (Pg. 29)
describes the term actually isn’t making any notable sustainability efforts. ng' is tech/energy- "Test 3375 - All India Open
and-
"greenwashing"? frequently Test 2 (2022)
environment/ca
(a) Conveying a false seen in the “A trash bag is labelled
lling-out-the-
impression that a news. greenwashing- 'recyclable'. Trash bags are
company's products trend/article30 not ordinarily separated from
are eco-friendly and 411299.ece other trash at the landfill or
environmentally sound incinerator, so they are highly
(b) Non-inclusion of unlikely to be used again for
ecological/environmen any purpose. The claim is
tal costs in the Annual deceptive since it asserts an
Financial Statements of environmental benefit where
a country. no meaningful benefit
(c) Ignoring the exists.""
disastrous ecological Which of the following best
consequences while describes the above
undertaking situation?
infrastructure (a) Greenwashing
development. (b) Evergreening
(d) Making mandatory (c) Green marketing
provision for (d) None"
environmental costs in
a government
project/programme.
81 Geography Consider the following D Statement 1 is not correct: Low, thick clouds primarily To test the D FA http://sites.iiser R
statements: reflect solar radiation and cool the surface of the Earth. basic pune.ac.in/~p.s M
1. High clouds primarily fundament ubramanian/Clo
uds_energy_cyc
reflect solar radiation Statement 2 is not correct: High, thin clouds primarily al
le.pdf
and cool the surface of transmit incoming solar radiation; at the same time, knowledge
the Earth. they trap some of the outgoing infrared radiation
2. Low clouds have a emitted by the Earth and radiate it back downward,
high absorption of thereby warming the surface of the Earth.
infrared radiation
emanating from the
Earth's surface and
thus cause warming
effect.
Which of the
statements given
75 www.visionias.in ©Vision IAS
above is/are correct?
(a) 1only (b) 2 only
(c) Both 1 and 2
(d) Neither 1 nor 2
82 Internatio Consider the following C The Bidibidi Refugee Settlement is located in the West In 2021, D CA https://weareali E
nal statements: Nile Area of Uganda, and is home to over 270,000 Kenya has ght.org/welcom N
Relations 1. Bidibidi is a large South Sudanese refugees — the second largest refugee announced e-bidi-bidi-
refugee-
refugee settlement in settlement in the world. The Bidibidi area was a small that it will
settlement/
north-western Kenya. village before becoming a refugee settlement which was close the
https://www.na
2. Some people who opened in August 2016. Since then, the Uganda Dadaab tionalgeographi
fled from South Sudan Government and other NGOs have worked to create a Camp in c.com/magazin
civil war live in Bidibidi. settlement rather than a camp to host and contain the June 2022. e/article/how-
3. Some people who influx of the growing number of the asylum seekers Bidibidi bidibidi-uganda-
fled from civil war in from South Sudan. Hence statement 1 is not correct Refugee refugee-camp-
Somalia live in Dadaab and statement 2 is correct. settlement became-city
refugee complex in Dadaab Refugee Camp is located in Dadaab, Kenya. was in https://www.do
ctorswithoutbor
Kenya. The first camp was established in 1991, when refugees news due
ders.org/latest/
Which of the fleeing the civil war in Somalia started to cross the to the work
kenya-urgent-
statements given border into Kenya. A second large influx occurred in of solutions-
above is/are correct? 2011, when some 130,000 refugees arrived, fleeing Internation needed-
(a) 1 and 2 drought and famine in southern Somalia. Humanitarian al refugees-
(b) 2 only assistance at the camp has waned in last few months Organisatio dadaab-camps-
(c) 2 and 3 following Kenya's announcement that it will close the n for close
(d) 3 only camps in June 2022. Hence statement 3 is correct. Migration
at the
settlement.
83 Internatio Consider the following C Organization of Turkic States (then called the The Eighth M CA https://www.tu E
nal countries: Cooperation Council of Turkic Speaking States - Turkic Summit of rkkon.org/en/tu N
Relations 1. Armenia Council) was established in 2009 as an the rk-konseyi-
hakkinda
2. Azerbaijan intergovernmental organization, with the overarching Cooperatio
3. Croatia aim of promoting comprehensive cooperation among n Council of
4. Romania Turkic States. Its four founding member states are Turkic
5. Uzbekistan Azerbaijan, Kazakhstan, Kyrgyzstan and Türkiye. In Speaking
Which of the above are October 2019, Uzbekistan joined as a full member. States was
members of the Hence option (c) is the correct answer. held in
Organization of Turkic November,
States? 2021.
(a) 1, 2 and 4
(b) 1 and 3 (c) 2 and 5
(d) 3, 4 and 5
76 www.visionias.in ©Vision IAS
84 Current Consider the following B Statement 1 is not correct: Bhadla Solar Park is the Major M CA https://www.liv E Monthly Current Affairs July
Affairs statements: world's largest solar park located in India which is infrastructu emint.com/indu N 2021 (Pg. 100), News Today
1. Gujarat has the spread over a total area of 14,000 acres in Bhadla, re related stry/energy/indi 25th Feb 2022
as-largest-
largest solar park in Phalodi tehsil, Jodhpur district, Rajasthan, India. news.
floating-solar-
India. All India Test Series - Test
power-plant-
2. Kerala has a fully Statement 2 is correct: Kerala's Cochin International commissioned- 3477:
solar powered Airport Ltd (CIAL) is the first airport in the world that in-andhra-
International Airport. would be running fully on solar power. The Cochin pradesh- India’s single largest solar
3. Goa has the largest airport became fully powered by solar energy back in 1163177412385 park by NTPC (National
floating solar 2015 after it started drawing power from a dedicated 3.html Thermal Power Corporation)
photovoltaic project in solar plant – the 15 megawatts (MW) photovoltaic is set to come up in the state
India. power station built by the Cochin International Airport https://www.hi of
ndustantimes.c
Which of the Limited. The entrepreneurial vision won the airport in (a) Gujarat
om/india-
statements given 2018 the much-coveted 'Champion of the Earth' award, (b) Rajasthan
news/fully-
above is/are correct? which is the highest environmental honour granted by solar-powered- (c) Tamil Nadu
(a) 1 and 2 the United Nations. cochin-airport- (d) Uttar Pradesh
(b) 2 only leaves-anand-
(c) 1 and 3 Statement 3 is not correct: Bharat Heavy Electricals mahindra-
(d) 3 only Limited (BHEL) has successfully commissioned India's impressed-see-
largest Floating Solar PV plant. Located at NTPC tweet-
Simhadri in Andhra Pradesh, the 25 MW floating SPV 1016386153810
22.html
project covers an area of 100 acres.
85 Internatio With reference to the D The United Nations Convention on the Law of the Sea UNCLOS is E FCA https://www.un E Monthly Current Affairs May
nal United National (UNCLOS) is an international agreement that establishes frequently .org/depts/los/c N 2021 (Pg. 8), Monthly Current
Relations Convention on the Law a legal framework for all marine and maritime activities. appeared in onvention_agre Affairs April 2021 (Pg. 88), PT
news due to ements/texts/u
of Sea, consider the Every State has the right to establish the breadth of its 365 IR (Pg. 55)
disputes in nclos/unclos_e.
following statements: territorial sea up to a limit not exceeding 12 nautical
South China pdf
1. A coastal state has miles, measured from baselines determined in Sea. https://indianex
the right to establish accordance with this Convention. Hence statement 1 is It was also in press.com/articl
the breadth of its correct. news in the e/opinion/colu
territorial sea up to a Subject to this Convention, ships of all States, whether context of mns/why-india-
limit not exceeding 12 coastal or land-locked, enjoy the right of innocent need to needs-to-
nautical miles, passage through the territorial sea. Hence statement 2 strengthen strengthen-its-
measured from is correct. India's maritime-laws-
maritime and-regulatory-
baseline determined in The exclusive economic zone shall not extend beyond
laws and mechanisms-
accordance with the 200 nautical miles from the baselines from which the
regulatory 7379949/
convention. breadth of the territorial sea is measured. Hence mechanisms
2. Ships of all states, statement 3 is correct. related to
whether coastal or maritime
land-locked, enjoy the sector.

77 www.visionias.in ©Vision IAS


right of innocent
passage through the
territorial sea.
3. The Exclusive
Economic Zone shall
not extend beyond 200
nautical miles from the
baseline from which
the breadth of the
territorial sea is
measured.
Which of the
statements given
above are correct?
(a) 1 and 2 only
(b) 2 and 3 only
(c) 1 and 3 only
(d) 1, 2 and 3

78 www.visionias.in ©Vision IAS


86 Internatio Which one of the B The Senkaku Islands are a group of uninhabited islands Senkaku E CA https://theprint E News Today 3rd May 2022
nal following statements in the East China Sea. They are known in mainland Islands are .in/world/two- N
Relations best reflects the issue China as the Diaoyu Islands. The islands are the focus of frequently chinese-ships-
enter-japans-
with Senkaku Islands, a territorial dispute between Japan and China. China mentioned
territorial-
sometimes mentioned claims the discovery and ownership of the islands from in news in
waters-near-
in the news? the 14th century, while Japan maintained ownership of the context senkaku-
(a) It is generally the islands from 1895 until its surrender at the end of of Japan- islands/948666/
believed that they are World War II. Hence option (b) is the correct answer. China
artificial islands made relations
by a country around and
South China Sea. growing
(b) China and Japan proximity
engage in maritime between
disputes over these India and
islands in East China Japan.
Sea.
(c) A permanent
American military base
has been set up there
to help Taiwan to
increase its defence
capabilities.
(d) Through
International Court of
Justice declared them
as no man's land, some
South-East Asian
countries claim them.
87 Current Consider the following C There are recurring claims that China is building, or Countries D CA https://econom E PT 365 International
Affairs pairs: attempting to create, a string of military bases around mentioned ictimes.indiatim N Relations (Pg. 26, 70, 71)
Country Important the world. In early December, 2021, media reports in the news es.com/news/d
efence/china-is-
reason for being in the alleged that China was trying to build its first Atlantic frequently
struggling-to-
news recently military base in Equatorial Guinea. Hence pair 1 is not appeared in
establish-
1. Chad – Setting up of correctly matched. last 8 military-
permanent military On 5 September 2021, President of Guinea was months. bases/articlesho
base by China. captured by the country's armed forces in a coup. w/88268005.cm
2. Guinea – Suspension Military leaders detain president, declaring dissolution s?from=mdr
of Constitution and of government and suspension of constitution. Also https://constitu
Government by The African Union (AU) declared that it was suspending tionnet.org/ne
military Guinea's membership in the bloc after a military junta ws/guinea-

79 www.visionias.in ©Vision IAS


3. Lebanon – Severe overthrew the government. Hence pair 2 is correctly military-
and prolonged matched. leaders-detain-
economic depression According to the World Bank Lebanon Economic president-
declaring-
4. Tunisia – Suspension Monitor (LEM), Lebanon is suffering from a dangerous
dissolution-
of Parliament by depletion of resources, including human capital, with
government-
President brain drain becoming an increasingly desperate option. and-suspension
How many pairs given World Bank also said that Lebanon is in a Deliberate https://www.eu
above are correctly Depression with Unprecedented Consequences for its roparl.europa.e
matched? Human Capital, Stability, and Prosperity. Hence pair 3 is u/thinktank/en/
(a) Only one pair correctly matched. document/EPRS
(b) Only two pairs In July, 2021, the president of Tunisia had announced _BRI(2022)7293
(c) Only three pairs the suspension of Tunisia’s parliament and fired Prime 69
https://www.w
(d) All four pairs Minister Hicham Mechichi and the cabinet. This
orldbank.org/en
announcement came one day after protesters had
/news/press-
clashed with security forces over how the Tunisian release/2020/1
government had handled the health care sector during 1/30/lebanon-
the COVID-19 pandemic and the deterioration of the is-in-a-
economic situation in the country. In December, 2021, deliberate-
the President announced the extension of the depression-
suspension and finally in March, 2022, he announced with-
the dissolution of the Parliament. Hence pair 4 is unprecedented-
consequences-
correctly matched.
for-its-human-
capital-stability-
and-prosperity
88 Geography Consider the following B Anatolia is the peninsula of land that today constitutes Regions M CA https://www.bri E Monthly Current Affairs
pairs: the Asian portion of Turkey. Because of its location at mentioned tannica.com/pla N March 2022 (Pg. 44)
Region often the point where the continents of Asia and Europe in the news ce/Anatolian-
plateau
mentioned in the news meet, Anatolia was, from the beginnings of civilization, frequently
https://www.th
Country a crossroads for numerous peoples migrating or appeared in
ehindu.com/opi
1. Anatolia – Turkey conquering from either continent. Hence pair 1 is news. nion/op-ed/the-
2. Amhara – Ethiopia correctly matched. path-to-peace-
3. Cabo Delgado – in-
Spain The Amhara Region is located in the northwestern part ethiopia/article
4. Catalonia – Italy of Ethiopia. It is the homeland of Amhara people. 33974219.ece
How many pairs given Amhara is the site of the largest inland body of water in https://econom
above are correctly Ethiopia, Lake Tana. Amhara region is often mentioned ictimes.indiatim
es.com/news/in
matched? in the news due to historical conflicts among Oromo,
dia/india-and-
(a) Only one pair Amhara and Tigrayan groups of Ethiopia. Hence pair 2 is
mozambique-
(b) Only two pairs correctly matched. to-expand-fight-
(c) Only three pairs
80 www.visionias.in ©Vision IAS
(d) All four pairs Cabo Delgado is the northernmost province of against-
Mozambique. It was in news because Mozambique's Al- terror/articlesh
Shabab extremist group, has been carrying out attacks ow/91404902.c
ms
in the country’s most northern province, Cabo Delgado,
https://www.th
since 2017. Hence pair 3 is not correctly matched.
ehindu.com/ne
Catalonia occupies a triangular area in the ws/internationa
northeastern corner of Spain. It is bordered by France l/mass-protests-
and Andorra to the north, the Mediterranean Sea to the across-
east, the autonomous community of Valencia to the catalonia-after-
south, and the autonomous community of Aragon to ex-leaders-
the west. Recently, mass protests broke out across arrest/article61
Spain’s Catalonia region after its former separatist 873462.ece
leader Carles Puigdemont was arrested. Hence pair 4 is
not correctly matched.

81 www.visionias.in ©Vision IAS


89 Environme With reference to A Statement 1 is correct: The Wild Life (Protection) Act, Governmen M FA https://www.th R Vision IAS Test Series FLT:
nt Indian laws about 1972 mentions that every wild animal, other than t had ehindu.com/opi M Test 3500 (Statement 3)
wildlife protection, vermin, which is hunted shall be the property of the recently nion/op- If an animal belonging to the
ed/Leopards-in-
consider the following State Government, and, where such animal is hunted in proposed schedule I of Wildlife
a-
statements: a sanctuary or National Park declared by the Central amendmen Protection Act,1972 turns into
spot/article169
1. Wild animals are the Government, such animal or any animal article shall be ts in the 13392.ece a man eater and poses threat
sole property of the the property of the Central Government. In a significant Wild Life to humans, then which of the
government. verdict in 2012, the Bombay High Court has ruled that (Protection) following authorities is
2. When a wild animal wild animals including tiger should be treated as Act, 1972. permitted to order for its
is declared protected, "government property for all purposes". killing?
such animal is entitled (a) Chief Wildlife Warden of
for equal protection Statement 2 is correct: The law governing the subject of the state concerned
whether it is found in wildlife, the Wildlife (Protection) Act, 1972, does not (b) Deputy Conservator of
protected areas or discriminate between animals found in protected areas Forests of state concerned
outside. and outside. It provides for equal protection for wild (c) Range forest officer of the
3. Apprehension of a animals irrespective of where they are found. The WLPA state concerned
protected wild animal prohibits people from hunting wildlife, provides legal (d) Forest guard of the state
becoming a danger to safeguards for different species based on their threat concerned
human life is sufficient status, regulates trade and commerce in wild species,
ground for its capture imposes penalties for wildlife-related crimes and Vision IAS Test Series: Test
or killing. specifies the terms to declare protected areas. 3501 (Statement 2)
Which of the With reference to the various
statements given Statement 3 is not correct: Only if the wild animal species of seahorses, consider
above is/are correct? becomes a danger to human life or is diseased or the following statements:
(a) 1 and 2 disabled beyond recovery can it be allowed to be 1. They are the flagship
(b) 2 only captured or killed by the competent authority, the Chief species of their habitats.
(c) 1 and 3 Wildlife Warden of the State. This provision is applicable 2. They are among the only
(d) 3 only to wild animals listed in Schedule I of the Wildlife animal species on Earth in
(Protection) Act, 1972, which includes leopards. Mere which the male bears the
apprehension or fear that a wild animal could endanger unborn young.
human life is not a ground for capture or killing. 3. All seahorse species found
in India are protected under
Schedule I of Wildlife
(Protection) Act, 1972.
Which of the statements
given above is/are correct?
(a) 1 only
(b) 2 and 3 only
(c) 3 only
(d) 1, 2 and 3
82 www.visionias.in ©Vision IAS
Explanation mentions: All
seahorse species found in
India are protected under
Schedule I of Wildlife
(Protection) Act, 1972 making
their sale, purchase, or any
other form of utilization of
the species, or their parts and
derivatives, illegal and
punishable under the law.
90 Environme Certain species of A The ant–fungus mutualism is a symbiosis seen Recently, D FA https://www.nd R
nt which one of the between certain ant and fungal species, in which ants US tv.com/world- R
following organisms actively cultivate fungus much like humans farm crops Scientists news/us-
scientists-find-
are well known as as a food source. Find Killer
killer-fungus-to-
cultivators of fungi? Fungus To
tackle-invasive-
(a) Ant Leafcutter ants use leaves as their fertilizer to grow Tackle ants-issue-in-
(b) Cockroach their crop: fungus. They cultivate their fungal gardens Invasive texas-2848991
(c) Crab by providing them with freshly cut leaves, protecting Ants Issue
(d) Spider them from pests and molds, and clearing them of In Texas. https://www.sci
decayed material and garbage. In return, the fungus ence.org/doi/10
acts as a food source for the ants' larvae. .1126/science.1
53.3736.587
91 Ancient Consider the following B The set of 14 major rock edicts (or portions thereof) To test the M F ASHOKA, THE E
History pairs: occur at: basic EMPEROR M
Site of Ashoka's major Kandahar (in Kandahar district, south Afghanistan) (only fundament WHO GAVE UP
WAR-NCERT
rock edicts Location in portions of rock edicts 12 and 13) al
class 7 (Our
the State of Shahbazgarhi (Peshawar district, North-West Frontier knowledge
Past-II) Page no.
1. Dhauli – Odisha Province, Pakistan) 68.
2. Erragudi – Andhra Mansehra (Hazara district, NWFP, Pakistan)
Pradesh Kalsi (Dehradun district, Uttarakhand) Themes in
3. Jaugada – Madhya Girnar (Junagadh district, Gujarat) Indian History :
Pradesh Mumbai–Sopara (originally at Sopara in Thane district, 12th NCERT :
4. Kalsi – Karnataka Maharashtra; now in the Chhatrapati Shivaji Maharaj Chapter 2
How many pairs given Vastu Sangrahalaya, Mumbai; only fragments of rock
above are correctly edicts 8 and 9)
matched? Dhauli (Puri district, Orissa; separate rock edicts 1 and
(a) Only one pair 2 replace major rock edicts 11–13)
(b) Only two pairs Jaugada (Ganjam district, Orissa; separate rock edicts 1
(c) Only three pairs and 2 replace major rock edicts 11–13)
(d) All four pairs Erragudi (Kurnool district, Andhra Pradesh)

83 www.visionias.in ©Vision IAS


Sannati (Gulbarga district, Karnataka; portions of rock
edicts 12 and 14 and separate rock edicts 1 and 2 were
found on a granite slab in a medieval goddess temple.

92 Medieval Consider the following B The Gurjara Pratihara dynasty was founded by To test the M F https://ncert.ni E
History pairs: Nagabhatta I in the region of Malwa in the 8th century basic c.in/ncerts/l/ge M
King Dynasty AD. Gurjara-Pratihara dynasty ruled much of Northern fundament ss102.pdf (Our
past-II Class 7th
1. Nannuka – Chandela India from the mid-8th to the 11th century. al
NCERT ( New
2. jayashakti – The Pratiharas derived their name from the Sanskrit knowledge
Kings and
Paramara meaning doorkeeper, are seen as a tribal group or a clan Kingdom),
3. Nagabhata II ` of the Gurjaras.
gurjara-Prathihara Nagabhata I, Nagabhata Il, Mihir Bhoj or Bhoja and https://nios.ac.i
4. Bhoja – Rashtrakuta Mahendrapala were the other important rulers of the n/media/docum
How many pairs given dynasty. ents/SrSec315N
above are correctly The expansion of the Gurjara-Pratihara kingdom EW/History_Mo
matched? involved constant conflicts with other contemporary dule1.pdf

84 www.visionias.in ©Vision IAS


(a) Only one pair powers such as the Palas and the Rashtrakutas known https://www.kh
(b) Only two pairs as the tripartite struggle over the city of Kannauj. ajuraho-
(c) Only three pairs Dantidurga was the founder of the dynasty called, india.org/chand
ela-
(d) All four pairs Rashtrakuta dynasty (8th century AD). The capital of the
dynasty.html
Rastrakutas was Manyakheta or Malkhed near Sholapur.
Dhruva-I, Krishna-I and Amoghavarsha were the other
important rulers of this dynasty.

The Chandela or Chandel was Indian Rajput clan of


Central India. They were popularly termed as Chandelas
or Jejakabhukti dynasty in Indian history. Nannuk, a
ruler of small kingdom was the founder of Chandela
Dynasty. Khajuraho was a strong hold of Nannuk who
was considered as the chief of his clan. Vakapati
succeeded his father in the first quarter of tenth
century. Vakapati had to often assist Pratiharas in the
battle field, as he was a liegeman of them. Some
inscriptions prove that his territory extended till
Vindhya hills. Jaishakti, the elder son, succeeded the
throne and was successful in extending his empire.
Jaishakti, the elder brother who ruled first, was also
called Jai Jak and from this name the region ruled by the
Chandelas acquired the name of Jaijikbhukti. He was
succeeded on the throne by his younger brother,
Vijayshakti.
93 Art & Which one of the B The Sangam age refers to that period in the early To test the M FA Class XI NCERT, E All India Test Series- Tests
Culture following statements history of south India when large numbers of poems in basic Ancient India, M 3474 & 3481:
about Sangam Tamil were composed by a number of authors. . The fundament Chapter-3, Page Consider the following
No.-17-18
literature in ancient term Sangam refers to an assembly or “meeting al statements with respect to
South India is correct? together” of Tamil poets. Traditionally, three Sangams knowledge Sangam literature:
(a) Sangam poems are or assemblies are believed to have been convened one India Ancient's 1. Sangam texts were
devoid of any after the other. All the three Sangams took place at Past-R.S predominantly secular in
reference to material different places under the patronage of the Pandya Sharma- nature.
culture. kings of Madurai. (Chapter 22)- 2. Sangam poems describe
(b) The social Women in the Sangam period appear to have been The dawn of the military exploits of the
classification of Varna educated. This is testified by many poems contributed history in the kings and chiefs.
was known to Sangam by women poets to the Sangam literature. Women are deep south. 3. Sangam literature provides
poets. also described as engaged in various economic activities information about trade with
(c) Sangam poems such as paddy plantation, cattle rearing, basket-making, Yavanas (foreigners).
have no reference to spinning, etc. However, the cruel practice of Sati was Which of the statements
85 www.visionias.in ©Vision IAS
warrior ethic. also prevalent in Tamil society, and it was known as given above are correct?
(d) Sangam literature tippayadal. But it was not obligatory as there are (a) 1 and 2 only
refers to magical forces references to widows present in society. However, their (b) 2 and 3 only
as irrational. position was miserable as they were prohibited to (c) 1 and 3 only
decorate themselves or participate in any form of (d) 1, 2 and 3
amusement.
The class of warriors was an important element in With reference to the social
society during the Sangam age. Captains of the army structure prevalent in the
were invested with the title of enadi at a formal Sangam age, consider the
ceremony. Civil and military officers were held under following statements:
both the Cholas and the Pandyas by vellalas or rich 1. The practice of Sati was
peasants. The ruling class was called arasar and its prohibited in the Sangam
members had marriage relations with the vellalas. society.
Hence option (b) is the correct answer. 2. Agricultural operations
The rich did not plough land themselves but employed such as ploughing were
labourers for this purpose. Agricultural operations were mainly confined to the
generally carried on by the members of the lowest class members of the lowest class.
known as kadaisiyar. 3. The rich nobles were
We notice sharp social inequalities in the Sangam age. invested with the title of
The rich lived in houses of brick and mortar and the enadi to demarcate the
poor in huts and humbler structures. In the cities, the society.
rich lived in the upper storey of their house. Which of the statements
The Sangam texts refer to many settlements including given above is/are correct?
Kaveripattanam whose flourishing existence is now (a) 1 and 3 only
attested archaeologically. They also speak of the (b) 3 only
Yavanas (foreigners) coming in theri own vessels (c) 2 only
purchasing pepper with gold and supplying wine and (d) 1, 2 and 3
women slaves to the natives. This trade is not known
only from Latin and Greek writings but also from
archaeological records. The Sangam literature is a very
major source of our information for the social,
economic and political life of the people living in deltaic
Tamil Nadu in the early Christian centuries.
Aṇaṅku is a phenomenon of sacred power described in
ancient Tamil literature such as the Sangam literature.
This sacred magical forces were supposed to inhabit
various objects, which eventually determined there
association with the society.

86 www.visionias.in ©Vision IAS


94 Art & "Yogavasistha" was A Yoga Vasistha (also known as Vasistha's Yoga) is a Hindu https://the D F https://religion. R
Culture translated into Persian spiritual text traditionally attributed to Valmiki. It wire.in/poli fandom.com/wi R
by Nizamuddin recounts a discourse of the sage Vasistha to a young tics/narend ki/Yoga_Vasisth
a#Translations
Panipati during the Prince Rama, during a period when the latter is in a ra-modi-
https://www.wi
reign of: dejected state. The contents of Vasistha's teaching to nine-covid-
sdomlib.org/sho
(a) Akbar Rama is associated with Advaita Vedanta, the illusory 19-lights p/books/hinduis
(b) Humayun nature of the manifest world and the principle of non- m/the-
(c) Shahjahan duality. The book has been dated between the 11th and Hidden in yogavasistha-of-
(d) Aurangzeb 14th century AD) and is generally regarded as one of the Plain Sight valmiki-with-
longest texts in Sanskrit (after the Mahabharata) and an in PM commentary/do
important text of Yoga. The book consists of about Modi's New c225484.html
32,000 shlokas (lines), including numerous short stories Coronavirus
and anecdotes used to help illustrate its content. Action Call,
a Pattern of
Originally written in Sanskrit, the Yoga Vasistha has Nines.
been translated into most Indian languages, and the A former
stories are told to children in various forms. The number head of the
of Muslim scholars who collaborated with Hindu pandits Indian
in making Sanskrit works available was considerable. Medical
During the Moghul Dynasty the text was translated into Association,
Persian several times, as ordered by Akbar, Jahangir Dr K.K.
and Darah Shikoh. Aggarwal,
Nizam al-Din Panipati rendered the widely influential took to
Yoga Vasistha into Persian late in the sixteenth century social
at the behest of the Mughal ruler Jahangir while he media to
was still a crown prince. The translation, known as the “explain
Jug-Basisht, has since became popular in Persia among the
intellectuals interested in Indo-Persian culture. science”
Hence option (a) is the correct answer. behind the
prime
minister’s
announcem
ent – but
his
explanation
, if it can be
called that,
led only to
further
confusion.
87 www.visionias.in ©Vision IAS
“… [the
announcem
ent] is
based on
Yoga
Vasistha,
Chapter 6,
the
principle of
collective
consciousn
ess. …This
collective
consciousn
ess has the
power to
heal the
ACE2-
receptor in
our
bodies,” he
said.

“If we all
think
together
that the
coronavirus
not attach
itself to our
ACE2-
receptors,
then our
collective
consciousn
ess will
make sure
this doesn’t
happen.

88 www.visionias.in ©Vision IAS


95 Current The World's second A Recent context: 216-feet tall ‘Statue of Equality 216-feet M FCA https://statueof E Monthly Current Affairs
Affairs tallest statue in sitting commemorating Sri Ramanujacharya was recently tall ‘Statue equality.org/ab N February 2022 (Pg. 107)
pose of Ramanuja was dedicated to the nation. The statue has been of Equality out-bagwad-sri- Abhyaas 2022 Test 3700-
ramanujacharya
inaugurated by the inaugurated at Hyderabad in the backdrop of commemor 118950(Explanation)
-history-
Prime Minister of India celebrations of the ‘Festival of equality’, marking the ating Sri Recently, the "Statue of
philosophy/
at Hyderabad recently. 1000th birth anniversary of the Saint. Ramanujac Equality" has been erected to
Which one of the The statue has been conceptualized by Sri Chinna Jeeyar harya was https://indianex honor Ramanujacharya. In
following statements Swami of Sri Ramanujacharya Ashram. It is made of recently press.com/articl this context, consider the
correctly represents ‘panchaloha’, a combination of five metals: gold, silver, dedicated e/explained/ra following statements:
the teachings of copper, brass, and zinc and is among one of the tallest to the manujacharya- 1. He was the chief exponent
Ramanuja? metallic statues in sitting position, in the world. nation. The statue-of- of the Visishtadvaita school of
(a) The best means of About Sri Ramanujacharya statue has equality- Vedanta.
explained-
salvation was Born in Tamil Nadu in the 11th century, been 2. He was the author of
7754236/
devotion. Ramanujacharya is revered as a Vedic philosopher and inaugurate Brahmasutras.
(b) Vedas are eternal, social reformer. He was deeply influenced by the Alvars d at 3. He was a contemporary of
self-existent and (Vishnu worshippers) and according to him the best Hyderabad Hoysala King
wholly authoritative. means of attaining salvation was through intense in the "Vishnuvardhana".
(c) Logical arguments devotion to Vishnu. He propounded the doctrine of backdrop of Which of the statements
were essential means Vishishtadvaita or qualified oneness. celebration given above are correct?
for the highest bliss. He spoke of universal salvation through devotion to s of the Explanation: Recent context:
(d) Salvation was to be God, compassion, humility, equality, and mutual ‘Festival of 216-feet tall ‘Statue of
obtained through respect, which is known as Sri Vaishnavam Sampradaya. equality’, Equality commemorating Sri
mediation. .He is considered to be the inspiration for Bhakti poets marking the Ramanujacharya was recently
like Annamacharya, Ramdas, Thyagaraja, Kabir, and 1000th dedicated to the nation. The
Meerabai. birth statue has been inaugurated
anniversary at Hyderabad in the backdrop
of the of celebrations of the ‘Festival
Saint. of equality’, marking the
1000th birth anniversary of
the Saint.
The statue has been
conceptualized by Sri Chinna
Jeeyar Swami of Sri
Ramanujacharya Ashram. It is
made of ‘panchaloha’, a
combination of five metals:
gold, silver, copper, brass, and
zinc and is among one of the
tallest metallic statues in
sitting position, in the world.
89 www.visionias.in ©Vision IAS
About Sri Ramanujacharya
Born in Tamil Nadu in the
11th century,
Ramanujacharya is revered as
a Vedic philosopher and social
reformer. He was deeply
influenced by the Alvars
(Vishnu worshippers) and
according to him the best
means of attaining salvation
was through intense
devotion to Vishnu. He
propounded the doctrine of
Vishishtadvaita or qualified
oneness. Hence statement 1
is correct
.He is considered to be the
inspiration for Bhakti poets
like Annamacharya, Ramdas,
Thyagaraja, Kabir, and
Meerabai.
Ramanujacharya started
writing the commentary of
‘Brahmasutra’. It is known as
‘Sri Bhashya’. He wrote nine
scriptures known as the
Navaratnas, including Sri
Bhashya, Gita-Bhasya, and
Vedartha Sangraha, and
composed commentaries on
Vedic scriptures. Hence
statement 2 is not correct.
Badarayana or Vyasa wrote
the Brahma Sutra. It
contained the Upanisadic
teaching dealt in four
chapters, covering 555 sutras.
Both Shankracharya and
Ramanujacharya have written
commentaries called
90 www.visionias.in ©Vision IAS
Brahmasutra Bhasya and Sri
Bhasya respectively on it.
He is also credited with
establishing the correct
procedures for rituals
performed in temples
throughout India, the most
famous being Tirumala and
Srirangam.
He propounded the concept
of “vasudhaiva kutumbakam”
(originally in Maha
Upanishad), which translates
as “all the universe is one
family”.
He spoke of universal
salvation through devotion to
God, compassion, humility,
equality, and mutual respect,
which is known as Sri
Vaishnavam Sampradaya.

91 www.visionias.in ©Vision IAS


96 Current The Prime Minister A Somnath is a magnificent temple the location of PM to D FCA https://girsomn E News Today 14th April 2022,
Affairs recently inaugurated Somanatha was earlier referred to as Prathasa Pattana, inaugurate ath.nic.in/histor N 21st Aug 2021
the new Circuit House a well known Tirtha or place of pilgrimage in Saurashtra. new Circuit y/#:~:text=Som 3489(AITS)
nath's%20first%
near Somnath Temple It is situated on the west coast of Kathiawar and is at House near
20temple%20is
at Veraval. Which of present under Junagadh. It was associated with the Somnath Which of the following are
%20said,temple
the following nearby confluence of three rivers and it adjoined the Temple in %20and%20des Jyotirlingas in India?
statements are correct port of Veraval. One of the 12 holy Jyotirlingas of Lord January troyed%20the% 1. Somnath
regarding Somnath Shiva is in Jyotirlinga here in Somnath. Hence 2022 20temple. 2. Mallikarjun
Temple? statement 1 is correct. http://www.indi 3. Omkareshwar
1. Somnath Temple is Somnath mean “the protector of moon god”. It is ancommunities. 4. Badrinath
one of the Jyotirlinga known as the “shrine external” as although the temple org/2016/10/10 Select the correct answer
shrines. has been destroyed six times it has been rebuilt every /expedition-of- using the code given below:
somnath/
2. A description of single time.
Somnath Temple was Somnath is also mentioned in Rigveda. This temple of
https://www.th
given by Al-Biruni. Somnath has been very attractive in front of many ehindu.com/ne
3. Pran Pratishtha of destructive foreign invaders who want to lure and ws/national/oth
Somnath Temple convert to the temple glory. Whenever the temple has er-states/pm-
(installation of the tried to destroy it, then it has been built again. to-inaugurate-
present day temple) Mahmud began series of seventeen invasions into new-circuit-
was done by President northwestern India at the end of the 10th century his house-near-
S. Radhakrishnan. 16th expedition was the plunder of Somnath temple somnath-
temple/article3
Select the correct (dedicated to Shiva) in 1025 A.D. Mahmud of Ghazni
8296560.ece
answer using the code plundered the Somanath temple and there are multiple
given below: versions of the event in Turko-Persian sources. Al- https://www.th
(a) 1 and 2 only Beruni writes that the temple was built of stone and ehindu.com/ne
(b) 2 and 3 only ‘constructed about 100 years prior to Mahmud’s attack ws/national/pm
(c) 1 and 3 only (which would date it to the 10th century) that it was set -modi-virtual-
(d) 1, 2 and 3 in a fortress surrounded by the sea on 3 sides – inaguration-of-
presumably; its wealth had to be guarded. Al-Beruni few-projects-of-
states that the upper part of the icon was broken at the somnath-
temple-in-
orders of Mahmud and parts of it were taken back as
gujarat/article3
look to Ghazni and placed so that people would walk on
6012049.ece
it. Mahmud’s raids caused economic devastation and
the Turks were hated among the people who suffered
because of these raids. Hence statement 2 is correct.

India’s Iron Man and First Deputy Prime Minister Sardar


Vallabhbhai Patel on November 13, 1947 promised to
rebuild the temple. Today’s Somnath Temple is built on
its original place in seventh place. In 1951, India’s first
92 www.visionias.in ©Vision IAS
President Dr. Rajendra Prasad proposed to purify
Jyotirling, he said, “This temple of Somnath is a symbol
of victory over construction on destruction”. Hence
statement 3 is not correct.

The temple is built under Shri Somnath Trust And this


trust is now monitoring the temple, Sardar Patel was
the first chairman of this trust.
97 Basic Which one of the D The overall ability of the body to fight the disease- Related to E F https://indianex E PT 365 Science and
Science following statements causing organisms (pathogens), conferred by the COVID 19 press.com/articl N Technology (Pg. 3)
best describes the role immune system is called immunity. The primary and (immune e/lifestyle/healt All India Test Series: Test
h/a-cancer-
of B cells and T cells in secondary immune responses are carried out with the system); 3494
treatment-
the human body? help of two special types of lymphocytes present in our CART has Consider the following
makes-
(a) They protect the blood, i.e., B-lymphocytes (B-cells) and T-lymphocytes been in the leukemia- statements regarding Viral
body from (T-cells). Thus, both B-cells and T-cells are responsible news for vanish-but- Vector-based Vaccines:
environmental for acquired immunity. B-cells are responsible for quite a long creates-more- 1. Viral Vector Vaccines use a
allergens. antibody-mediated immunity response. T-cells are time mysteries- weakened form of the deadly
(b) They alleviate the responsible for cell-mediated immunity response. B- 7760489/ virus to create an immune
body's pain and cells produce antibodies to fight with the pathogens response.
inflammation. into our blood. The T-cells themselves do not secrete 2. Immune response involves
(c) They act as antibodies but help B cells produce them. Hence option both B cells and T cells.
immunosuppressants (d) is the correct answer. 3. Previous exposure to the
in the body. vector could reduce the
(d) They protect the effectiveness of the Vaccine.
body from the diseases Which of the statements
caused by pathogens. given above is/are correct?
(a) 1 and 2 only
(b) 2 and 3 only
(c) 1 only (d) 1 and 3 only
2021 Test Series
3192
With reference to the
immunity mechanism in the
human body, which of the
following statements is/are
correct regarding B-cells and
T-cells?
B-cells are responsible for
innate immunity whereas T
cells are responsible for
93 www.visionias.in ©Vision IAS
acquired Immunity.
B-cells produce antibodies
molecule whereas T-cells do
not.
Select the correct answer
using the code given below.
(a) 1 only (b) 2 only
(c) Both 1 and 2
(d) Neither 1 nor 2
98 S&T Consider the following D A nanoparticle is a small particle that ranges between 1 Nanotechn E FCA https://pubs.rsc R PT 365 Science and
statements: to 100 nanometres in size (diameter). Undetectable by ology and .org/en/content R Technology (Pg. 11)
1. Other than those the human eye, nanoparticles can exhibit significantly nanoparticl /articlehtml/20
15/cs/c5cs0023
made by humans, different physical and chemical properties to their larger es have
6b#:~:text=Iron
nanoparticles do not material counterparts. Being much smaller than the always
%20oxides%2Fs
exist in nature. wavelengths of visible light (400-700 nm), nanoparticles been in the ulfides%2C%20s
2. Nanoparticles of cannot be seen with ordinary optical microscopes, news. The ilver%2C,occurri
some metallic oxides requiring the use of electron microscopes or varied ng%20nanopart
are used in the microscopes with laser. functions of icles%20in%20t
manufacture of some Nanoparticles occur widely in nature and are objects of nanoparticl he%20environm
cosmetics. study in many sciences such as chemistry, physics, es e.g. gold ent.&text=Num
3. Nanoparticles of geology and biology. A significant fraction (by number, if nanoparticl erous%20studie
s%20have%20b
some commercial not by mass) of interplanetary dust, that is still falling on es have
een%20conduct
products which enter the Earth at the rate of thousands of tons per year, is in been hot
ed,on%20huma
the environment are the nanoparticle range; and the same is true of topics. ns%20and%20t
unsafe for humans. atmospheric dust particles. Many viruses have he%20ecosyste
Which of the diameters in the nanoparticle range. Iron ms.
statements given oxides/sulfides, silver, and gold are some of the
above is/are correct? representative examples of naturally-occurring https://www.sci
(a) 1 only nanoparticles in the environment. Hence statement 1 is encedirect.com
(b) 3 only not correct. /science/article
/abs/pii/S01665
(c) 1 and 2 Many cosmetic products used in our daily life are made
26X21000209
(d) 2 and 3 with the assistance of nanotechnologies. Among metal
and metal oxide nanoparticles (NPs) potentially present
in cosmetics, those containing titanium dioxide and zinc
oxide are common ingredients added to obtain a
sufficient sun protection. The addition of silver and gold
NPs is also mentioned to increase the antimicrobial and
healing properties of some cosmetic formulations.
Hence statement 2 is correct.
The exposure of nanoparticle to environment and
94 www.visionias.in ©Vision IAS
human can be described through different mechanisms.
Primarily occupational exposures occur to workers
(including engineers, scientists, and technicians) during
the research-scale synthesis and commercial
production of nanomaterial-based products. This
exposure mainly results from handling of raw materials
while carrying out reactions through the equipment.
Characterization of resulting material, packing, and
transportation can be other sources of this type of
exposure. At the second stage, consumers are exposed
to such nanomaterial during usage and application and
it may lead to harmful and toxic effects. Hence
statement 3 is correct.
99 S&T Consider the following D DNA barcoding is the identification of a species by the Owing to D CAA https://www.th R
statements: characteristic sequence of a standard short section of the ehindu.com/ne R
DNA Barcoding can be DNA in its genome. DNA barcoding is done by frequent ws/national/ker
ala/genetic-
a tool to: comparing an individual DNA sequence with a reference discoveries
barcodes-
1. assess the age of a library of such DNA sections (called sequences). The of species
reveal-three-
plant or animal. introduction of DNA barcoding represents a promising belonging frogs-
2. distinguish among approach for food authentication, being broadly to common unreported-in-
species that look alike. applied in fish species. DNA barcoding has been genera e.g. india/article246
3. identify undesirable successful when applied to seafood because the frogs, 67101.ece
animal or plant number of species is high in comparison to other animal fishes, etc.
materials in processed sources, such as cattle, sheep, goat, and horse, https://www.pn
foods. enhancing the effectiveness of the approach. DNA as.org/doi/10.1
073/pnas.05104
Which of the barcoding techniques were used to explore the
66103#:~:text=T
statements given composition of roasted barley tea (Hordeum vulgare),
he%20use%20o
above is/are correct? which was exported to a certain country from China but f%20DNA%20ba
(a) 1 only then returned, due to the detection of other rcodes,rich%20t
(b) 3 only undesirable plant components present. Contaminants ropical%20biota
(c) 1 and 2 were found. Hence option 3 is correct. s%20is%20unkn
(d) 2 and 3 DNA barcoding will allow users to efficiently recognize own.
known species and speed the discovery of species yet
to be found in nature. There have been reports of DNA https://www.sci
encedirect.com
barcodes used in distinguishing species of tropical
/topics/agricult
Lepidoptera (an order of insects that includes butterflies
ural-and-
and moths which are similar). DNA barcodes effectively biological-
distinguish among species in three Lepidoptera families sciences/dna-
from Area de Conservación Guanacaste in northwestern barcoding#:~:te
Costa Rica. Hence option 2 is correct. xt=DNA%20barc

95 www.visionias.in ©Vision IAS


However, DNA barcoding is not known to be used for oding%20is%20
the assessment of age of living organisms. Hence option a%20system,6.1
1 is not correct. Hence (d) is the correct answer. ).

https://www.re
searchgate.net/
publication/272
535748_A_succ
essful_case_of_
DNA_barcoding
_used_in_an_in
ternational_tra
de_dispute

100 Environme Consider the following: B Acid rain results when sulfur dioxide (SO2) and This E F https://ncert.ni E Vision IAS Test Series: Test
nt 1. Carbon monoxide nitrogen oxides (NOX) are emitted into the atmosphere question c.in/textbook/p M 3477
2. Nitrogen oxide and transported by wind and air currents. The SO2 and aims to test df/kech207.pdf
3. Ozone NOX react with water, oxygen and other chemicals to the Which of the following
4. Sulphur dioxide form sulfuric and nitric acids. These then mix with water fundament statements is not correct with
Excess of which of the and other materials before falling to the ground. al respect to acid rain?
above in the knowledge (a) Acid rain is resulted from
environment is/are the emission of oxides of
cause(s) of acid rain? sulphur and nitrogen.
(a) 1, 2 and 3 (b) Acid rain dissolves and
(b) 2 and 4 only washes away nutrients
(c) 4 only needed for the growth of
(d) 1, 3 and 4 crops.
(c) Acid rain has a pH value of
between 6 and 7.
(d) Acid rain corrodes water
pipes resulting in the leaching
of heavy metals into drinking
water.

Explanation mentions: Acid


rain refers to the ways in
which acid from the
atmosphere is deposited on
the earth’s surface. Oxides of
nitrogen and sulfur which are
acidic in nature can be blown

96 www.visionias.in ©Vision IAS


by the wind along with solid
particles in the atmosphere
and finally settle down either
on the ground as dry
deposition or in water, fog
and snow as wet deposition.

97 www.visionias.in ©Vision IAS


TOPIC

Topic Number of Questions


Ancient History 2
Art & Culture 4
Basic Science 2
Current Affairs 15
Economics 14
Environment 14
Geography 11
International Relations 7
Medieval History 4
Modern History 4
Polity & Governance 12
S&T 11
Total 100

98 www.visionias.in ©Vision IAS


DIFFICULTY

Difficulty Level Number of Questions


Easy - E 32
Medium - M 42
Difficult - D 26

NATURE

Nature Number of Questions


F: Fundamental 36
FA: Fundamental Applied 19
CA: Current Affairs 17
CAA: Current Affairs Applied 8
FCA: Fundamental + Current affairs 20

99 www.visionias.in ©Vision IAS


SOURCE TYPE

Source Type Number of Questions


EM = Essential material 25
RM = Reference material 20
EN = Essential News/Current Affairs 45
RR = Random Read 10

Copyright © by Vision IAS


All rights are reserved. No part of this document may be reproduced, stored in a retrieval system or transmitted in any form or by any means, electronic, mechanical,
photocopying, recording or otherwise, without prior permission of Vision IAS.

100 www.visionias.in ©Vision IAS

You might also like